[obm-l] Obtendo limitantes com a Desigualdade de Chebychev e Markov

2004-03-13 Por tôpico Carlos Maçaranduba
Nao estou conseguindo um limitantes superiores para os
2 problemas:

Convençao:
P() ->Probabilidade
X ->Variavel Aleatoria  associado a um espaço amostral
E(X) ->Esperança ou Media
V(x) ->Variancia = Desvio Padrao ao quadrado
e ->numero Real positivo
v ->numero Real positivo
<= ->Menor ou Igual
>= ->Maior ou igual 
> ->Maior
< ->Menor

-Problema 1:
Desigualdade de Chebychev:
P( |X - E(X)| >= e ) <= V(x) / (e^2)

O problema:
Use a desigualdade para estimar uma cota superior para
a probabilidade de que uma variavel aleatoria tendo
media M e desvio padrao P, se desvie de M por menos
que 3P.

Entao acho que o problema quer isso:
P(|X - M| < 3P)

Modificando um pouco a desigualdade:
P( |X - E(X)| >= e ) <= V(x) / (e^2)
=>1 - P( |X - E(X)| >= e ) >= 1 - V(x) / (e^2)
=> P( |X - E(X)| < e ) >= 1 - V(x) / (e^2)

Fazendo-se e = 3P, E(X)= M  e  V(x) = P^2 temos:
=>P( |X - M| < 3P ) >= 1 - P^2 / 9(P^2)
=> P( |X - M| < 3P ) >= 1 - 1/9
=> P( |X - M| < 3P ) >= 8/9

Mas 8/9 seria uma cota inferior e o problema quer uma
cota superior(por se tratar de probabilidades,é logico
que uma cota superior seria 1 e uma inferior seria
0,mas essas nao valem).Mas a unica forma que eu
encontrei de encaixar |X - M| < 3P na desigualdade foi
essa.O que eu faço?

-Problema 2:
Desigualdade de Markov: P(X >= v) <= E(x) / v
Seja E(X) = M, X <= 2M.De um limitante superior para 
P(X <= M/2).

Quando eu tento adaptar , consigo um limitante
inferior, mas nao superior, similar ao problema 1.




__

Yahoo! Mail - O melhor e-mail do Brasil! Abra sua conta agora:
http://br.yahoo.com/info/mail.html
=
Instruções para entrar na lista, sair da lista e usar a lista em
http://www.mat.puc-rio.br/~nicolau/olimp/obm-l.html
=


[obm-l] Obtendo limitantes com a Desigualdade de Chebychev e Markov

2004-03-13 Por tôpico Carlos Maçaranduba
Nao estou conseguindo um limitantes superiores para os
2 problemas:

Convençao:
P() ->Probabilidade
X ->Variavel Aleatoria  associado a um espaço amostral
E(X) ->Esperança ou Media
V(x) ->Variancia = Desvio Padrao ao quadrado
e ->numero Real positivo
v ->numero Real positivo
<= ->Menor ou Igual
>= ->Maior ou igual 
> ->Maior
< ->Menor

-Problema 1:
Desigualdade de Chebychev:
P( |X - E(X)| >= e ) <= V(x) / (e^2)

O problema:
Use a desigualdade para estimar uma cota superior para
a probabilidade de que uma variavel aleatoria tendo
media M e desvio padrao P, se desvie de M por menos
que 3P.

Entao acho que o problema quer isso:
P(|X - M| < 3P)

Modificando um pouco a desigualdade:
P( |X - E(X)| >= e ) <= V(x) / (e^2)
=>1 - P( |X - E(X)| >= e ) >= 1 - V(x) / (e^2)
=> P( |X - E(X)| < e ) >= 1 - V(x) / (e^2)

Fazendo-se e = 3P, E(X)= M  e  V(x) = P^2 temos:
=>P( |X - M| < 3P ) >= 1 - P^2 / 9(P^2)
=> P( |X - M| < 3P ) >= 1 - 1/9
=> P( |X - M| < 3P ) >= 8/9

Mas 8/9 seria uma cota inferior e o problema quer uma
cota superior(por se tratar de probabilidades,é logico
que uma cota superior seria 1 e uma inferior seria
0,mas essas nao valem).Mas a unica forma que eu
encontrei de encaixar |X - M| < 3P na desigualdade foi
essa.O que eu faço?

-Problema 2:
Desigualdade de Markov: P(X >= v) <= E(x) / v
Seja E(X) = M, X <= 2M.De um limitante superior para 
P(X <= M/2).

Quando eu tento adaptar , consigo um limitante
inferior, mas nao superior, similar ao problema 1.




__

Yahoo! Mail - O melhor e-mail do Brasil! Abra sua conta agora:
http://br.yahoo.com/info/mail.html
=
Instruções para entrar na lista, sair da lista e usar a lista em
http://www.mat.puc-rio.br/~nicolau/olimp/obm-l.html
=


Re: [obm-l] RE: [obm-l] Números inteiros e probabilidade

2004-03-02 Por tôpico Carlos Maçaranduba
O que é a funçao Zeta de Riemann e que zeros nao
triviais sao esses??

 --- Paulo Santa Rita <[EMAIL PROTECTED]> escreveu: >
Ola "Rafael" e demais
> colegas desta lista ... OBM-L,
> 
> Se P1 e um numero  primo, para cada P1 numeros na
> sequencia 1, 2, ..., N, 
> ... havera um numero
> divisivel  por P1, isto e, havera um numero que tem
> P1 como fator primo. 
> Vale dizer que entre os
> numeros naturais, ao escolhermos um ao acaso, a
> probabilidade de que ele 
> tenha P1 como fator
> primo e 1/P1 
> 
> Supondo ( o que e razoavel ) que as escolhas sao
> eventos independentes, 
> entao a probabilidade de
> que os tres numeros escolhidos tenham P1 por fator
> primo e :
> 
> (1/P1)*(1/P1)*(1/P1) = 1/(P1^3)
> 
> O que nos interssa e justamente o contrario, isto e,
> queremos que os tres 
> nao tenham o fator
> primo P1 em comum. Portanto, a probabilidade e :
> 
> 1 - [1/(P1^3)]
> 
> Devemos repetir este raciocinio para todos os
> numeros primos. A  
> probabilidade que procuramos sera
> portanto :
> 
> R = {1 - [1/(2^3)]}*{1 - [1/(3^3)]}*{1 -
> [1/(5^3)]}*...*{1 - [1/(P^3)]}*...
> R = 
>
{[(2^3)-1]/(2^3)]}*{[(3^3)-1]/(3^3)]}*{[(5^3)-1]/(5^3)]}*...*{[(P^3)-1]/(P^3)]}*...
> 1/R 
>
={(2^3)/[(2^3)-1]}*{(3^3)/[(3^3)-1]}*{(5^3)/[(5^3)-1]}*...*{(P^3)/[(P^3)-1]}*...
> 1/R 
>
={1/[1-(2^(-3))]}*{1/[1-(3^(-3))]}*{1/[1-(5^(-3))]}*...*{1/[1-(P^(-3))]}*...
> 
> Observe que cada fator e da forma :
> 
> {1/[1-(P^(-3))]}= 1 + (1/P)^3 + (1/P)^6 + (1/P)^9 +
> ... + (1/P)^(3*N) + ...
> 
> Olhando com tranquilidade, se convenca de que para
> qualquer natural N, o 
> valor de 1/R contem
> 1/(N^3), isto e :
> 
> 1/R = 1 + (1/2)^3 + (1/3)^3 + (1/4)^3 + ... +
> (1/N)^3 + ...
> 
> Esta serie e evidentemente convergente. Todavia, se
> voce propor o problema 
> de se determinar
> o seu valor, muito provavelmente, nenhum matematico
> do mundo sabera 
> responder. Euler e Gauss
> se ocuparam dela, sem sucesso. O "valor simbolico" e
>  ZETA(3), onde ZETA e a 
> famoso funcao
> de Riemann sobre a qual  ninguem sabe  provar se
> todos os seus zeros 
> nao-triviais tem realmete
> parte real igual a 1/2.
> 
> Portanto :
> 
> 1/R = ZETA(3) => R = 1/ZETA(3)
> 
> Observe que se fossem escolhidos 2 numeros, teriamos
> R=1/ZETA(2)=6/(pi^2). 
> Esta e tambem
> a probabilidade de se escolher um numero natural de
> forma que ele nao tenha 
> fator primo duplicado
> ( alguem ja provou isso aqui nesta lista ). Dai eu
> concluo que para N 
> numeros bastaria saber a
> probabilidade do numero nao ter fator primo elevado
> a N.
> 
> Um Abraco
> Paulo Santa Rita
> 1,2154,010304
> 
> 
> 
> 
> 
> >From: "Rafael" <[EMAIL PROTECTED]>
> >Reply-To: [EMAIL PROTECTED]
> >To: "OBM-L" <[EMAIL PROTECTED]>
> >Subject: [obm-l] Números inteiros e probabilidade
> >Date: Sat, 28 Feb 2004 18:51:31 -0300
> >
> >Boa noite, pessoal.
> >
> >Por esses dias, deparei-me com o seguinte problema:
> >
> >"Sejam três inteiros escolhidos ao acaso, a
> probabilidade de que não haja
> >fator comum que os divida é...?"
> >
> >Não imagino como isso poderia ser calculado. Alguém
> tem alguma idéia?
> >
> >
> >Obrigado,
> >
> >Rafael de A. Sampaio
> >
> 
>
_
> MSN Hotmail, o maior webmail do Brasil. 
> http://www.hotmail.com
> 
>
=
> Instruções para entrar na lista, sair da lista e
> usar a lista em
> http://www.mat.puc-rio.br/~nicolau/olimp/obm-l.html
>
= 

__

Yahoo! Mail - O melhor e-mail do Brasil! Abra sua conta agora:
http://br.yahoo.com/info/mail.html
=
Instruções para entrar na lista, sair da lista e usar a lista em
http://www.mat.puc-rio.br/~nicolau/olimp/obm-l.html
=


Re: [obm-l] Ideal Maximal

2004-03-02 Por tôpico Carlos Maçaranduba
Como interpreto ???Se fosse somente  era
tranquilo

 --- Cláudio_(Prática)
<[EMAIL PROTECTED]> escreveu: >
Obrigado, Nicolau!
> 
> Eu estava assumindo implicita e erroneamente que
> todo ideal de Z_4[x] eh
> principal, mas checando meus alfarrábios, vejo que
> A[x] só será um PID se A
> for um corpo. Aliás, o mesmo exemplo com Z ao invés
> de Z_4 mostra que mesmo
> que A seja um domínio de integridade (mas não um
> corpo), A[x] não será
> necessariamente um PID (apesar de ser um domínio de
> integridade).
> 
> Aos poucos estas idéias vão se assentando...
> 
> Um abraço,
> Claudio.
> 
> - Original Message -
> From: "Nicolau C. Saldanha" <[EMAIL PROTECTED]>
> To: <[EMAIL PROTECTED]>
> Sent: Monday, March 01, 2004 1:11 PM
> Subject: Re: [obm-l] Ideal Maximal
> 
> 
> > On Mon, Mar 01, 2004 at 11:09:44AM -0300, Claudio
> Buffara wrote:
> > > Sejam:
> > > Z_4 = anel dos inteiros mod 4
> > > e
> > > Z_4[x] = anel dos polinomios com coeficientes em
> Z_4.
> > > O ideal  de Z_4[x] eh maximal?
> > >
> > > Eu diria que sim, dado que x^2 + 1 eh
> irredutival sobre Z_4, mas nesse
> caso,
> > > Z_4[x]/ seria um corpo, o que nao eh
> verdade, pois contem o
> > > elemento 2 + , o qual eh um divisor de
> zero.
> > >
> > > Onde estah o meu erro?
> >
> > O ideal J que você descreveu não é maximal, ele
> está contido no ideal
> > J1 = . Aliás J1 também não é maximal,
> ele está contido
> > em J2 =  (pois x^2+1 = (x+1)^2 - 2x); J2
> sim é maximal, e o
> > quociente é Z/(2).
> >
> > O fato do polinômio p em A[x] ser irredutível não
> prova que o ideal 
> > é maximal se A for um anel, isto só dá certo se A
> for um corpo.
> >
> > []s, N.
> >
>
=
> > Instruções para entrar na lista, sair da lista e
> usar a lista em
> >
> http://www.mat.puc-rio.br/~nicolau/olimp/obm-l.html
> >
>
=
> 
>
=
> Instruções para entrar na lista, sair da lista e
> usar a lista em
> http://www.mat.puc-rio.br/~nicolau/olimp/obm-l.html
>
= 

__

Yahoo! Mail - O melhor e-mail do Brasil! Abra sua conta agora:
http://br.yahoo.com/info/mail.html
=
Instruções para entrar na lista, sair da lista e usar a lista em
http://www.mat.puc-rio.br/~nicolau/olimp/obm-l.html
=


[obm-l] Orgia de livros

2004-02-19 Por tôpico Carlos Maçaranduba
Varios livros free no formato pdf em:
http://br.endernet.org/~drini/books/


__

Yahoo! Mail - O melhor e-mail do Brasil! Abra sua conta agora:
http://br.yahoo.com/info/mail.html
=
Instruções para entrar na lista, sair da lista e usar a lista em
http://www.mat.puc-rio.br/~nicolau/olimp/obm-l.html
=


[obm-l] Caminhadas em Grafos

2004-02-08 Por tôpico Carlos Maçaranduba
esse problema nao faço a minima ideia como se faz:

-Dado  um grafo de e arestas e v vertives , sendo m e
n 2 vertices desse grafo,tal que existe uma aresta
entre eles, começando pelo vertice m e escolhendo
aleatoriamente uma das arestas que possui m como
vertice, qual a probabilidade dessa aresta conter n
como outro vertice

A resposta é no minimo 2/3 , mas eu nao faço ideia
como se chega nesta resposta.



__

Yahoo! GeoCities: 15MB de espaço grátis para criar seu web site!
http://br.geocities.yahoo.com/
=
Instruções para entrar na lista, sair da lista e usar a lista em
http://www.mat.puc-rio.br/~nicolau/olimp/obm-l.html
=


RE: [obm-l] Cubo de Rubik

2004-02-08 Por tôpico Carlos Maçaranduba
e eu que pensava que era somente eu que me sentia
assimvaleu pela observação..

 --- Artur Costa Steiner <[EMAIL PROTECTED]>
escreveu: > 
> 
> -Original Message-
> From: [EMAIL PROTECTED]
> [mailto:[EMAIL PROTECTED] On
> Behalf Of [EMAIL PROTECTED]
> Sent: Saturday, February 07, 2004 2:36 PM
> To: [EMAIL PROTECTED]
> Subject: Re: [obm-l] Cubo de Rubik
> 
> Ola, 
> 
> >Fui eu quem enviou o problema dos bispos, resolvido
> brilhantemente pelo
> Nicolau. Eh uma >pena que pouquissimas pessoas nesta
> lista se interessam por
> puzzles mecanicos de >matematica. Haja vista que
> enviei um site de um cubo
> Rubick em 4 dimensoes e eu pedia para >alguem me
> explicar como seria um de 5
> dimensoes e ninguem quis responder.
> 
> Eh sem duvida um assunto muito bonito. O que
> acontece e que a matematica eh
> muito extensa eh a tendencia eh que cada um responda
> ou envie mensagens
> novas quando se trata de assunto de que goste mais
> ou domine mais. Ateh
> mesmo por uma questao de tempo. Acho que a grande
> maioria dos participantes
> desta lista nao podem se dedicar aa matematica da
> forma como gostariam.
> Mesmo os maiores experts em Analise Complexa, Teoria
> dos Numeros, Integral
> de Lebesgue, etc tem que se preocupar em fazer valer
> a prosaica desigualdae
> Orcamento >= Despesas, o que pode ser mais dificil
> do que provar a hipotese
> de Riemann.
> 
> Eu mesmo jah enviei a esta lista uma 20 mensagens
> que ninguem comentou. Eh
> um tanto frustrante, mas o assunto, certamente, nao
> pareceu interessante aos
> outros participantes. 
> O que eu realmente acho muito desagradavel eh quando
> alguem pede ajuda com
> algum problema, voce tenta ajudar e o principal
> interessado nao mais se
> manifesta. Fica-se com a impressao de se ter perdido
> tempo para nada.
> 
> Artur
> 
> 
> 
> 
>
=
> Instruções para entrar na lista, sair da lista e
> usar a lista em
> http://www.mat.puc-rio.br/~nicolau/olimp/obm-l.html
>
= 

__

Yahoo! GeoCities: 15MB de espaço grátis para criar seu web site!
http://br.geocities.yahoo.com/
=
Instruções para entrar na lista, sair da lista e usar a lista em
http://www.mat.puc-rio.br/~nicolau/olimp/obm-l.html
=


RE: [obm-l] Teorema das raizes racionais.

2004-01-29 Por tôpico Carlos Maçaranduba
A reciproca neste caso,nao é verdadeira, mas o que se
faz é pegar o p e q devidos e testar se é raiz.Mas vc
sabe a probabilidade de ser raiz???Aproveitando, como
é que é aquela historia da probabilidade da reciproca
do pequeno teorema de Fermat??
Melhor ainda ,alguem tem alguma referencia(de
preferencia na internet) sobre probabilidades e Teoria
dos Numeros???Ou referencias da probabilidade de
caminhadas em grafos???



 --- Artur Costa Steiner <[EMAIL PROTECTED]>
escreveu: > Naum sei se este eh o teorema ao qual vc
se refere,
> mas o que eu conheco por
> este nome diz o seguinte: Seja P um polinomio de
> coeficientes inteiros dado
> por P(x) = a_0 + a_1x.+a_n x^n (a_n<>0). Se a
> fracao irredutivel p/q, p
> e q inteiros, q<>0, for raiz de P, entao p divide
> a_0 e q divide a_n.
> Uma forma de vermos isto comeca observando o fato de
> que, se r eh raiz de P,
> entao, para todo real x, P(x) = (x-r)* Q(x), onde Q
> eh um polinomio de grau
> n-1. Para facilitar, consideremos inicialmente o
> caso particular em que q=1
> e p, consequentemente, eh raiz de P. Como os
> coeficientes de P sao inteiros
> e os do binomio x-p sao 1 e -1, o algoritmo da
> divisao de polinomios
> acarreta que os coeficientes de Q sejam inteiros.
> Temos entao que P(x) =
> (x-p)* Q(x) e, portanto, P(0) = a_0 = -p * Q(0).
> Como Q(0), o termo
> independente de Q, eh inteiro, segue-se que p divide
> a_0. E como q=1 eh
> divisor de a_n, concluimos que o teorema vale neste
> caso particular.
> 
> No caso geral, observamos que, se p/q eh raiz de P,
> entao a_0 + a_1
> *(p/q)...+ a_n*(p/q)^n = 0. Logo, a_0*q^n +
> a_1*p*q^(n-1)  + a_n*p^n =0.
> Temos portanto que p eh raiz do polinomio P1 de
> coeficientes (do termo
> independente para o do termo de grau n) a_0*q^n,
> a_1*q^(n-1),...a_n e q eh
> raiz do polinomio P2 de coeficientes (mesma
> convencao) a_n*p^n,
> a_(n-1)*p^(n-1),...a_0. Eh imediato que os
> coeficientes de P1 e de P2 sao
> inteiros. Aplicando-se o caso particular do teorema,
> jah demonstrado, a P1,
> concluimos que p divide a_0*q^n. Mas como p/q eh uma
> fracao irredutivel,
> segue-se necessariamente que p divide a_0. De modo
> similar,  aplicando-se o
> caso particular do teorema a P2 concluimos que q
> divide a_n*p^n e que,
> portanto, q divide a_n. Isto completa a demonstracao
> do teorema.  
> 
> Exemplo simples: os racionais 3/2 e 1/2 sao raizes 
> do polinomio do 2o grau,
> de coeficientes inteiros, P(x) = 4x^2 - 8x + 3.
> Verificamos facilmente que
> as condicoes especificadas no teorema sao validas.
> Outra aplicacao: podemos
> afirmar que P(x) = x^579 - 785*x^273 + 4297*x^198 +
> 1 nao admite raizes
> racionais. Segundo o teorema, se a fracao
> irredutivel p/q for raiz de P,
> entao p divide 1 e q divide 1. Para que isto seja
> possivel, temos que p=q =1
> e p/q =1, o que faz de 1 o unico racional candidato
> a raiz de P. Mas 1,
> decididamente, naum eh raiz de P.
> 
> Finalmente, eh interessante observar que a reciproca
> do teorema nao eh
> verdadeira.  
> Abracos
> Artur   
>  
> 
> >-Original Message-
> >From: [EMAIL PROTECTED]
> [mailto:[EMAIL PROTECTED] On
> >Behalf Of Victor Machado
> >Sent: Tuesday, January 27, 2004 7:15 PM
> >To: [EMAIL PROTECTED]
> >Subject: [obm-l] Teorema das raizes racionais.
> >
> >Olá pessoal.
> >Gostaria de saber como é o Teorema das Raízes
> Racionais, como prová-la e um
> >exemplo de aplicação.
> >Muita coisa ? :)
> >Obrigado.
> >Víctor.
> >
>
>_
> >--->Get your free email @godisdead.com
> >Made possible by Fade to Black Comedy Magazine
> >
>
>=
> >Instru es para entrar na lista, sair da lista e
> usar a lista em
> >http://www.mat.puc-rio.br/~nicolau/olimp/obm-l.html
>
>=
> 
> 
>
=
> Instruções para entrar na lista, sair da lista e
> usar a lista em
> http://www.mat.puc-rio.br/~nicolau/olimp/obm-l.html
>
= 

__

Yahoo! GeoCities: a maneira mais fácil de criar seu web site grátis!
http://br.geocities.yahoo.com/
=
Instruções para entrar na lista, sair da lista e usar a lista em
http://www.mat.puc-rio.br/~nicolau/olimp/obm-l.html
=


[obm-l] Dilema dos prisioneiros-Probabilidade

2004-01-21 Por tôpico Carlos Maçaranduba
Um colega me deu esse problema para mim e eu nao soube
responder.


-Tres prisioneiros X,Y,Z sao informados por seu
carcereiro que um deles foi escolhido aleatoriamente
para ser eliminado e os outros 2 liberados.O
prisioneiro X pede ao carcereiro para que diga
confidencialmente para ele qual dos 2 prisioneiros(Y
ou Z) foi escolhido para ser liberado, dizendo ao
carcereiro que não ha problema nisso, pois ele já sabe
que um dos 2(Y ou Z) sera liberado.O carcereiro
recusa-se a dizer, argumentando que se ele(o
prisioneiro X) souber se Y ou Z foi liberado, entao a
sua probabilidade de ser eliminado aumentaria de 1/3
para 1/2.O que voce acha do raciocinio do carcereiro?? 

__

Yahoo! GeoCities: a maneira mais fácil de criar seu web site grátis!
http://br.geocities.yahoo.com/
=
Instruções para entrar na lista, sair da lista e usar a lista em
http://www.mat.puc-rio.br/~nicolau/olimp/obm-l.html
=


Re: [obm-l] Uma belissima demonstracao

2004-01-21 Por tôpico Carlos Maçaranduba
nao é de Euclides nao, observe que na prova de
euclides , ele faz o produto dos i-esimos primos
p1p2...pi + 1 enquanto que na prova apresentada pelo
colega ele faz N! + 1.Alem do mais, a prova de Kumer
apresentada nao é a mesma que esta no
site.Aproveitando a oportunidade, ja vi uma prova que
usa o anel Z((-5)^1/2) mas nunca havia entendido
direito, se alguem souber explicar por favor fique a
vontade.

> 
>   Eu sempre aprendi que essa prova foi feita pelo
> Euclides,
> e o site do Wolfram parece confirmar isso:
> 
>   http://mathworld.wolfram.com/EuclidsTheorems.html
> 
> --
> Ricardo Bittencourt
> 


__

Yahoo! GeoCities: a maneira mais fácil de criar seu web site grátis!
http://br.geocities.yahoo.com/
=
Instruções para entrar na lista, sair da lista e usar a lista em
http://www.mat.puc-rio.br/~nicolau/olimp/obm-l.html
=


Re: [obm-l] Problema de fatoriais

2004-01-11 Por tôpico Carlos Maçaranduba
bem consegui resolver é (n - S_p(n))/(p - 1) o que
condiz para o caso que p = 2 como mostrado por
Nicolau.Basta ver que [n / p^i] = a_d*p^(d - i) + 
a_d-1*p^(d-1-i) + ... 
sendo n=(a_d,a_d-1, a_d-2 ... a_0)base p, com d + 1
digitos na base p e maos a massa...



 --- Carlos Maçaranduba <[EMAIL PROTECTED]>
escreveu: > 
> 
> 
> > > Ache a formula geral para a potencia do
> > > primo p que divide n! em funçao de n,p,S_b(n).
> > > ->não fiz :(
> > 
> > Este eu deixo para você. É parecido. []s, N.
>  
> Desculpe nao é S_b(n) é  S_p(n)
> 
> 
> 
>
__
> 
> Conheça a nova central de informações anti-spam do
> Yahoo! Mail:
> http://www.yahoo.com.br/antispam
>
=
> Instruções para entrar na lista, sair da lista e
> usar a lista em
> http://www.mat.puc-rio.br/~nicolau/olimp/obm-l.html
>
= 

__

Conheça a nova central de informações anti-spam do Yahoo! Mail:
http://www.yahoo.com.br/antispam
=
Instruções para entrar na lista, sair da lista e usar a lista em
http://www.mat.puc-rio.br/~nicolau/olimp/obm-l.html
=


Re: [obm-l] Problema de fatoriais

2004-01-11 Por tôpico Carlos Maçaranduba



> > Ache a formula geral para a potencia do
> > primo p que divide n! em funçao de n,p,S_b(n).
> > ->não fiz :(
> 
> Este eu deixo para você. É parecido. []s, N.
 
Desculpe nao é S_b(n) é  S_p(n)



__

Conheça a nova central de informações anti-spam do Yahoo! Mail:
http://www.yahoo.com.br/antispam
=
Instruções para entrar na lista, sair da lista e usar a lista em
http://www.mat.puc-rio.br/~nicolau/olimp/obm-l.html
=


[obm-l] Problema de fatoriais

2004-01-10 Por tôpico Carlos Maçaranduba
Alguem se habilita a fazer a letra c da questao???A a
e a b eu ja fiz...

1a)Mostre que a potencia de um primo p que exatamente
divide n! é igual a [n/p]+ [n/p^2] +
[n/p^3]+...[n/p^f]
sendo p^f <= n < p^(f+1).
->beleza :)

b)Usando a letra a ,escreva a fatoraçao de 100!.
->beleza :)

c)Sendo S_b(n) indicando a soma dos digitos de n na
base b(ex: 3 na base 2 é igual a 11. Entao S_2(3) =
2).
Mostre que a potencia de 2 que divide n! é igual a 
n - S_2(n).Ache a formula geral para a potencia do
primo p que divide n! em funçao de n,p,S_b(n).
->não fiz :(

__

Conheça a nova central de informações anti-spam do Yahoo! Mail:
http://www.yahoo.com.br/antispam
=
Instruções para entrar na lista, sair da lista e usar a lista em
http://www.mat.puc-rio.br/~nicolau/olimp/obm-l.html
=


Re:_[obm-l]_característica_de_um_corpo/dúvida

2004-01-07 Por tôpico Carlos Maçaranduba
Porque a conclusão de que é infinito???Essa prova não
mostra somente que se a caracteristica é zero, os
elementos são distintos 2 a 2 no corpo

 --- Felipe Pina <[EMAIL PROTECTED]> escreveu: > Vou
resolver a 1)
> 
> olhe para os seguintes elementos do corpo
> 
> 1
> 1 + 1
> 1 + 1 + 1
> 1 + 1 + 1 + 1
> ..
> e assim por diante. Afirmo que estes infinitos
> elementos sao distintos 2 a 2.
> Chame de n*, a soma de n 1`s.
> Suponha que 2 sao iguais. Digamos n* = m*
> 
> Se n != m,  existem mais 1`s de um lado da equacao
> do que de outro. 
> Subtraindo os 1`s em comum dos dois lados,
> descobrimos que a caracteristica 
> do corpo nao eh zero (contradicao)
> Portanto estes elementos sao distintos 2 a 2 e o
> corpo nao pode ser finito.
> 
> At 12:12 PM 1/6/2004, you wrote:
> >Olá amigos!
> >
> >1)Como provo que todo corpo de característica zero
> possui um número infinito
> >de elementos.
> >
> >2) mostre que se p não é primo, então Zp não é um
> corpo.
> 
> 
> >--
> >Use o melhor sistema de busca da Internet
> >Radar UOL - http://www.radaruol.com.br
> >
> >
> >
> >
>
>=
> >Instruções para entrar na lista, sair da lista e
> usar a lista em
> >http://www.mat.puc-rio.br/~nicolau/olimp/obm-l.html
>
>=
> 
> 
> 
>
=
> Instruções para entrar na lista, sair da lista e
> usar a lista em
> http://www.mat.puc-rio.br/~nicolau/olimp/obm-l.html
>
= 

__

Conheça a nova central de informações anti-spam do Yahoo! Mail:
http://www.yahoo.com.br/antispam
=
Instruções para entrar na lista, sair da lista e usar a lista em
http://www.mat.puc-rio.br/~nicolau/olimp/obm-l.html
=


Re: [obm-l] RE:_[obm-l]_sites_sobre_matemática

2003-12-24 Por tôpico Carlos Maçaranduba
Se voce quer em pdf, va em www.google.com coloque na 
o assunto que voce quer e ao lado coloque
filetype:pdf.
Ex:  , isto funciona
para outros tipos de arquivos tambem.

 --- Artur Coste Steiner <[EMAIL PROTECTED]>
escreveu: > Um muito conhecido eh o da MathWorld,
> http://mathworld.wolfram.com/  Eh
> muitro bom. Mas nao estou certo se os arquivos estao
> em pdf.
> Artur
> 
> 
> Subject: [obm-l] sites sobre matemática
> 
> vcs conhecem algum site onde haja arquivos no
> formato pdf
> sobre assuntos como algebra,trigonometria, teoria
> dos conjuntos e etc, pode
> ser em ingles.
> 
> 
>
=
> Instruções para entrar na lista, sair da lista e
> usar a lista em
> http://www.mat.puc-rio.br/~nicolau/olimp/obm-l.html
>
= 

__

Conheça a nova central de informações anti-spam do Yahoo! Mail:
http://www.yahoo.com.br/antispam
=
Instruções para entrar na lista, sair da lista e usar a lista em
http://www.mat.puc-rio.br/~nicolau/olimp/obm-l.html
=


Re: [obm-l] MDC de Impares

2003-12-10 Por tôpico Carlos Maçaranduba
porque a questao assim pedena verdade ela descreve
um metodo de se calcular mdc de 2 numeros e pede para
provar que ele realmente calcula o mdc, assim diz:

*Se a e b pares, sendo D = mdc(a,b) e faça d =
mdc(a/2,b/2) tal que D = 2d.
*Se um dos dois é par , digamos b, entao faça D =
mdc(a,b) e d = mdc(a ,b/2) tal que D = d.
 
Essas 2 partes eu ja demonstrei, faltou a terceira
parte no qual eu mandei para lista.

--- Anderson <[EMAIL PROTECTED]> escreveu: > Pq
da restricao a e b impares?  Parece que a 
> demonstracao vale tambem para pares.
> 
> --------
> > Carlos Maçaranduba wrote:
> > 
> > >  Como provo que , dado a e b tais que a e b
> impares
> > > positivos e a > b, sendo d = mdc(a,b) , entao d 
> tambem
> > > poderá ser 
> > > d = mdc(a - b , b)
> > 
> > Se d=mdc(a,b), então a=Ad e b=Bd, e mdc(A,B)=1.
> > 
> > Logo mdc(a-b,b)=mdc(Ad-Bd,Bd)=d.mdc(A-B,B)
> > 
> > Vamos agora por contradição:
> > Suponha que mdc(A-B,B)=k, com k diferente de 1.
> > 
> > Então A-B=rk e B=sk. Mas isso implica em
> > 
> > A=rk+B=rk+sk=(r+s)k. Logo k é fator comum de A 
> e B,
> > portanto mdc(A,B)=k, o que contradiz a hipótese de
> 
> mdc(A,B)=1.
> > Logo mdc(A-B,B)=1 e com isso concluímos que mdc(a-
> b,b)=d.1=d
> 
>  
>
__
> Acabe com aquelas janelinhas que pulam na sua tela.
> AntiPop-up UOL - É grátis!
> http://antipopup.uol.com.br/
> 
> 
> 
>
=
> Instruções para entrar na lista, sair da lista e
> usar a lista em
> http://www.mat.puc-rio.br/~nicolau/olimp/obm-l.html
>
= 

__

Yahoo! Mail: 6MB, anti-spam e antivírus gratuito! Crie sua conta agora:
http://mail.yahoo.com.br
=
Instruções para entrar na lista, sair da lista e usar a lista em
http://www.mat.puc-rio.br/~nicolau/olimp/obm-l.html
=


[obm-l] MDC de Impares

2003-12-08 Por tôpico Carlos Maçaranduba
 Como provo que , dado a e b tais que a e b impares
positivos e a > b, sendo d = mdc(a,b) , entao d tambem
poderá ser 
d = mdc(a - b , b)

__

Yahoo! Mail: 6MB, anti-spam e antivírus gratuito! Crie sua conta agora:
http://mail.yahoo.com.br
=
Instruções para entrar na lista, sair da lista e usar a lista em
http://www.mat.puc-rio.br/~nicolau/olimp/obm-l.html
=


Re: [obm-l] Mais problemas Sobre Grupos-Ajudem -me

2003-10-24 Por tôpico Carlos Maçaranduba
pessoal estou dando uma de autodidata e estudando
Teoria dos Grupos.Mesmo que os exercicios de Paulo
Santa Rita sejam trivias, pediria que me mostrassem
como faze-los porque só assim eu posso captar a
essencia da teoria e ser capaz de fazer outros mais
complicados.:)



 --- Paulo Santa Rita <[EMAIL PROTECTED]> escreveu: >
Ola Carlos e demais colegas
> desta lista ... OBM-L,
> 
> Vou contribuir um pouquinho ...
> Observe que este resultado tem uma consequencia
> imediata, qual seja : "Todo 
> Grupo de ordem menor ou igual a 5 e ciclico". Prove
> isso !
> 
> Dois outros problemas elementares sobre Grupos :
> 
> 1) Seja G um grupo e G' o subgrupo dos comutadores.
> Prove que o quociente 
> G/G' e abeliano.
> 
> 2) Seja G um grupo de ordem p^n, p primo e n >=3.
> Mostre que se o centro de 
> G tem ordem p entao existe uma classe de conjugacao
> de ordem p.
> 
> Um Abraco a Todos
> Paulo Santa Rita
> 2,1012,201003
> 




> >Seja Z conjunto dos inteiros e  o subgrupo
> gerado
> >por x e Zm um grupo qualquer mod m. Mostre que a,b
> e m
> >inteiros( m>= 2):
> >
> >e)Mostre  que se (G , *) é um grupo multiplicativo
> de
> >ordem 2 entao G é ciclico.
> >
> >f)Mostre  que se (G , *) é um grupo multiplicativo
> de
> >ordem 3 entao G é ciclico.(Sugestao: Sendo G =
> >{x,a,b}, x o elemento neutrode G, pense sobre o que
> >poderia ser o elemento ab)
> >


Yahoo! Mail - o melhor webmail do Brasil
http://mail.yahoo.com.br
=
Instruções para entrar na lista, sair da lista e usar a lista em
http://www.mat.puc-rio.br/~nicolau/olimp/obm-l.html
=


Re: [obm-l] equação diofantina

2003-10-24 Por tôpico Carlos Maçaranduba
Olhe a equaçao possui soluçao para x, y e K inteiros
se somente se MDC(a, b) dividir K.Vamos provar:


SE x, y e K inteiros => MDC(a,b) divide K.
Seja d = mdc(a,b) .Pegando ax+by = k e dividindo por d
em ambos os membros => (ax+by)/d = k/d.Observe o
primeiro membro.Como d é mdc de a e b ,ele divide ax
+by porque ele divide a e b ao mesmo tempo.Essa
divisao resulta num numero inteiro e como x e y sao
inteiros entao (ax + by) /d é um numero inteiro.Mas
(ax + by) /d é igual a k/d entao k/d deve ser um
numero inteiro.Entao para que isso ocorra d divide k,
portanto SE x, y e K inteiros => MDC(a,b) divide K. 

Provar a reciproca agora:
Se MDC(a,b) divide K => x, y e K inteiros.

Por bezout, Se MDC(a,b) = d => d = aw + bt, w e t
inteiros.Mas como d divide k => k = d*f , f inteiro.
Pegando d = aw + bt e multiplicando ambos os membros
por f => d*f = a*(w*f) + b*(t*f), mas d*f =k =>
k = a*(w*f) + b*(t*f) = ax +by => x=(w*f) e y = (t*f)
e como t, w e f sao inteiros => x e y sao
inteiros.Como k = ax +by e a,b,x,y é inteiro => k é
inteiro.
CQ:D1 

Observando sua equaçao como mdc(a,b) = 1 e x,y e K
inteiros ,mdc(a,b) divide K, pois mdc(a,b) =1.
Portanto, pelo que eu provei acima, como mdc(a,b) =1
=> ax +by = k tem soluçao para qualquer k inteiro
escolhido porque sempre 1 divide k. 
CQ:D2


Para saber as soluçoes, ai ja é outra historia.



--- luiz frança <[EMAIL PROTECTED]> escreveu: > 
> 
>  se  (a,b)=1 
> 
>  ax +by = k  , x, y e k inteiros
>  
>  porvar que sempre existe uma soluma solução x,y
> que satisfaça a equação para qualquer k escolhido.
> 
> será mesmo verdade?  bom... a principio se
>  
> ax +by = 1 tiver solução, então terá pra qualquer K.
> pois basta pegarmos Kx e Ky. Mas como provar que
> vale
> pra k=1 ???
> 
> __
> Do you Yahoo!?
> The New Yahoo! Shopping - with improved product
> search
> http://shopping.yahoo.com
>
=
> Instruções para entrar na lista, sair da lista e
> usar a lista em
> http://www.mat.puc-rio.br/~nicolau/olimp/obm-l.html
>
= 

Yahoo! Mail - o melhor webmail do Brasil
http://mail.yahoo.com.br
=
Instruções para entrar na lista, sair da lista e usar a lista em
http://www.mat.puc-rio.br/~nicolau/olimp/obm-l.html
=


Re: [obm-l] equação diofantina

2003-10-24 Por tôpico Carlos Maçaranduba
Olhe a equaçao possui soluçao para x, y e K inteiros
se somente se MDC(a, b) dividir K.Vamos provar:


SE x, y e K inteiros => MDC(a,b) divide K.
Seja d = mdc(a,b) .Pegando ax+by = k e dividindo por d
em ambos os membros => (ax+by)/d = k/d.Observe o
primeiro membro.Como d é mdc de a e b ,ele divide ax
+by porque ele divide a e b ao mesmo tempo.Essa
divisao resulta num numero inteiro e como x e y sao
inteiros entao (ax + by) /d é um numero inteiro.Mas
(ax + by) /d é igual a k/d entao k/d deve ser um
numero inteiro.Entao para que isso ocorra d divide k,
portanto SE x, y e K inteiros => MDC(a,b) divide K. 

Provar a reciproca agora:
Se MDC(a,b) divide K => x, y e K inteiros.

Por bezout, Se MDC(a,b) = d => d = aw + bt, w e t
inteiros.Mas como d divide k => k = d*f , f inteiro.
Pegando d = aw + bt e multiplicando ambos os membros
por f => d*f = a*(w*f) + b*(t*f), mas d*f =k =>
k = a*(w*f) + b*(t*f) = ax +by => x=(w*f) e y = (t*f)
e como t, w e f sao inteiros => x e y sao
inteiros.Como k = ax +by e a,b,x,y é inteiro => k é
inteiro.
CQ:D1 

Observando sua equaçao como mdc(a,b) = 1 e x,y e K
inteiros ,mdc(a,b) divide K, pois mdc(a,b) =1.
Portanto, pelo que eu provei acima, como mdc(a,b) =1
=> ax +by = k tem soluçao para qualquer k inteiro
escolhido porque sempre 1 divide k. 
CQ:D2


Para saber as soluçoes, ai ja é outra historia.



--- luiz frança <[EMAIL PROTECTED]> escreveu: > 
> 
>  se  (a,b)=1 
> 
>  ax +by = k  , x, y e k inteiros
>  
>  porvar que sempre existe uma soluma solução x,y
> que satisfaça a equação para qualquer k escolhido.
> 
> será mesmo verdade?  bom... a principio se
>  
> ax +by = 1 tiver solução, então terá pra qualquer K.
> pois basta pegarmos Kx e Ky. Mas como provar que
> vale
> pra k=1 ???
> 
> __
> Do you Yahoo!?
> The New Yahoo! Shopping - with improved product
> search
> http://shopping.yahoo.com
>
=
> Instruções para entrar na lista, sair da lista e
> usar a lista em
> http://www.mat.puc-rio.br/~nicolau/olimp/obm-l.html
>
= 

Yahoo! Mail - o melhor webmail do Brasil
http://mail.yahoo.com.br
=
Instruções para entrar na lista, sair da lista e usar a lista em
http://www.mat.puc-rio.br/~nicolau/olimp/obm-l.html
=


Re: [obm-l] Sobre a Revista Eureka

2003-10-23 Por tôpico Carlos Maçaranduba
eu ja conversei com Nelly. Obrigado :).

 --- "Nicolau C. Saldanha"
<[EMAIL PROTECTED]> escreveu: > On Wed,
Oct 22, 2003 at 05:48:22PM -0300, Carlos
> Maçaranduba wrote:
> > E eu que sempre achava que alguns desta lista lhe
> > interpretavam voce mal(Dirichlet)..Pra algumas
> > pessoas que estao interessadas apenas nos artigos
> > talvez seja mais interessante os artigos em
> > separadocomo vi alguem aqui mandar um e-mail
> > representando a revista Eureka, achei que o
> > responsavel leria este e-mail que na minha
> opiniao,
> > nao tem nada demais.Quanto as palavras, se for
> pela
> > matematica, eu abstraio :).
> 
> Deixando de lado os mal entendidos, este trabalho de
> botar
> os artigos individualmente na home page era feito
> sim,
> e talvez tenha sido deixado de lado por baixa
> demanda.
> Talvez seja melhor você escrever direto para os
> editores
> ([EMAIL PROTECTED], [EMAIL PROTECTED]) e/ou para a
> Nelly
> ([EMAIL PROTECTED]) dizendo que sim, você gosta de poder
> baixar
> os artigos individualmente.
> 
> []s, N.
>
=
> Instruções para entrar na lista, sair da lista e
> usar a lista em
> http://www.mat.puc-rio.br/~nicolau/olimp/obm-l.html
>
= 

Yahoo! Mail - o melhor webmail do Brasil
http://mail.yahoo.com.br
=
Instruções para entrar na lista, sair da lista e usar a lista em
http://www.mat.puc-rio.br/~nicolau/olimp/obm-l.html
=


Re: [obm-l] Sobre a Revista Eureka

2003-10-22 Por tôpico Carlos Maçaranduba
E eu que sempre achava que alguns desta lista lhe
interpretavam voce mal(Dirichlet)..Pra algumas
pessoas que estao interessadas apenas nos artigos
talvez seja mais interessante os artigos em
separadocomo vi alguem aqui mandar um e-mail
representando a revista Eureka, achei que o
responsavel leria este e-mail que na minha opiniao,
nao tem nada demais.Quanto as palavras, se for pela
matematica, eu abstraio :).

 --- Johann Peter Gustav Lejeune Dirichlet
<[EMAIL PROTECTED]> escreveu: >
NOSSA!Nao precisa ser tao estupido e rispido.Ja
> faz um tempo que esta parte tem sido deixada
> parada.E por um motivo simples:e mais facil
> pegar a revista inteira para ler na rede.
> Qualquer coisa fale com o pessoal por carta,oras!Ou
> diretamente por e-mail.
> 
> Carlos Maçaranduba <[EMAIL PROTECTED]> wrote:
> Quem for responsavel pela divulgaçao onde esta
> presente os artigos em separado da Revista Eureka,
> poderia pelo menos dar uma atualizadinha e por os
> artigos mais recentes...:)
> 
> Yahoo! Mail - o melhor webmail do Brasil
> http://mail.yahoo.com.br
>
=
> Instruções para entrar na lista, sair da lista e
> usar a lista em
> http://www.mat.puc-rio.br/~nicolau/olimp/obm-l.html
>
=
> 
> 
> -
> Yahoo! Mail - o melhor webmail do Brasil. Saiba
mais! 

Yahoo! Mail - o melhor webmail do Brasil
http://mail.yahoo.com.br
=
Instruções para entrar na lista, sair da lista e usar a lista em
http://www.mat.puc-rio.br/~nicolau/olimp/obm-l.html
=


[obm-l] Sobre a Revista Eureka

2003-10-21 Por tôpico Carlos Maçaranduba
 Quem for responsavel pela divulgaçao onde esta
presente os artigos em separado da Revista Eureka,
poderia pelo menos dar uma atualizadinha e por os
artigos mais recentes...:)

Yahoo! Mail - o melhor webmail do Brasil
http://mail.yahoo.com.br
=
Instruções para entrar na lista, sair da lista e usar a lista em
http://www.mat.puc-rio.br/~nicolau/olimp/obm-l.html
=


Re: [obm-l] Mais problemas Sobre Grupos

2003-10-20 Por tôpico Carlos Maçaranduba
contribua mais um pouco:) Me mostre as 3 questoes que
vc propos...

 --- Paulo Santa Rita <[EMAIL PROTECTED]> escreveu: >
Ola Carlos e demais colegas
> desta lista ... OBM-L,
> 
> Vou contribuir um pouquinho ...
> 
> G) Sendo "e" a identidade, de  Y^2= e para todo Y em
> G concluimos que Y^-1 = 
> Y ( Voce saberia dizer porque posso fazer esta
> afirmacao ? ). Sejam "a" e 
> "b" dois elementos quaisquer do Grupo. Entao ab e
> (ab)^-1 estao em G e, pelo 
> que vimos :
> 
> ab=(ab)-1 => ab=(b^-1)(a^-1)  mas b^-1=b e a^-1 = a.
> Segue que :
> ab=ba, para quaisquer "a" e "b" em G. O grupo e
> portanto abeliano.
> 
> Observe que este resultado tem uma consequencia
> imediata, qual seja : "Todo 
> Grupo de ordem menor ou igual a 5 e ciclico". Prove
> isso !
> 
> Dois outros problemas elementares sobre Grupos :
> 
> 1) Seja G um grupo e G' o subgrupo dos comutadores.
> Prove que o quociente 
> G/G' e abeliano.
> 
> 2) Seja G um grupo de ordem p^n, p primo e n >=3.
> Mostre que se o centro de 
> G tem ordem p entao existe uma classe de conjugacao
> de ordem p.
> 
> Um Abraco a Todos
> Paulo Santa Rita
> 2,1012,201003
> 
> >From: Carlos Maçaranduba <[EMAIL PROTECTED]>
> >Reply-To: [EMAIL PROTECTED]
> >To: [EMAIL PROTECTED]
> >Subject: [obm-l] Mais problemas Sobre Grupos
> >Date: Sun, 19 Oct 2003 20:32:19 -0300 (ART)
> >MIME-Version: 1.0
> >Received: from mc5-f8.hotmail.com ([65.54.252.15])
> by mc5-s21.hotmail.com 
> >with Microsoft SMTPSVC(5.0.2195.5600); Mon, 20 Oct
> 2003 04:15:02 -0700
> >Received: from sucuri.mat.puc-rio.br
> ([139.82.27.7]) by mc5-f8.hotmail.com 
> >with Microsoft SMTPSVC(5.0.2195.5600); Mon, 20 Oct
> 2003 04:15:01 -0700
> >Received: (from [EMAIL PROTECTED])by
> sucuri.mat.puc-rio.br (8.9.3/8.9.3) 
> >id VAA05070for obm-l-MTTP; Sun, 19 Oct 2003
> 21:00:41 -0300
> >Received: from web21109.mail.yahoo.com
> (web21109.mail.yahoo.com 
> >[216.136.227.111])by sucuri.mat.puc-rio.br
> (8.9.3/8.9.3) with SMTP id 
> >UAA04957for <[EMAIL PROTECTED]>; Sun, 19 Oct
> 2003 20:59:41 -0300
> >Received: from [200.164.247.30] by
> web21109.mail.yahoo.com via HTTP; Sun, 
> >19 Oct 2003 20:32:19 ART
> >X-Message-Info:
> NDMZeIBu+soqT/9tqALIbVX3Lxac9UkwSv5iQMq7xO4=
> >Message-ID:
> <[EMAIL PROTECTED]>
> >In-Reply-To:
> <[EMAIL PROTECTED]>
> >Sender: [EMAIL PROTECTED]
> >Precedence: bulk
> >Return-Path: [EMAIL PROTECTED]
> >X-OriginalArrivalTime: 20 Oct 2003 11:15:01.0949
> (UTC) 
> >FILETIME=[67D536D0:01C396FB]
> >
> >Eu consegui provar a letra a o resto nao) ai
> vão:
> >
> >
> >Seja Z conjunto dos inteiros e  o subgrupo
> gerado
> >por x e Zm um grupo qualquer mod m. Mostre que a,b
> e m
> >inteiros( m>= 2):
> >
> >a)sendo B = b (mod m) e A = a (mod m) se a divide b
> >entao, como subgrupos de Zm,
> > esta contido em .(Esse eu consegui provar o
> >resto nao)
> >
> >b)sendo A = a (mod m) se mdc(a,m) = 1 , entao  =
> >Zm.
> >
> >c)sendo A = a (mod m) e D = d (mod m) se mdc(a,m) =
> d
> >, entao  = .
> >
> >d) De posse das informacoes acima, determine todos
> os
> >subgrupos de (Z36 , +).
> >
> >e)Mostre  que se (G , *) é um grupo multiplicativo
> de
> >ordem 2 entao G é ciclico.
> >
> >f)Mostre  que se (G , *) é um grupo multiplicativo
> de
> >ordem 3 entao G é ciclico.(Sugestao: Sendo G =
> >{x,a,b}, x o elemento neutrode G, pense sobre o que
> >poderia ser o elemento ab)
> >
> >g)Mostre  que se (G , *) é um grupo multiplicativo
> de
> >elemento neutro x , mostre que se y^2 =x, para cada
> y
> >em G, entao G é abeliano.(Sugestao:Note que y^2 = x
> >implica que y^-1 = x .Tome 2 elementos quaisquer a
> e b
> >em G e comece escrevendo ab = (ab)^-1 = ...)
> >
> >Yahoo! Mail - o melhor webmail do Brasil
> >http://mail.yahoo.com.br
>
>=
> >Instruções para entrar na lista, sair da lista e
> usar a lista em
> >http://www.mat.puc-rio.br/~nicolau/olimp/obm-l.html
>
>=
> 
>
_
> MSN Hotmail, o maior webmail do Brasil. 
> http://www.hotmail.com
> 
>
=
> Instruções para entrar na lista, sair da lista e
> usar a lista em
> http://www.mat.puc-rio.br/~nicolau/olimp/obm-l.html
>
= 

Yahoo! Mail - o melhor webmail do Brasil
http://mail.yahoo.com.br
=
Instruções para entrar na lista, sair da lista e usar a lista em
http://www.mat.puc-rio.br/~nicolau/olimp/obm-l.html
=


[obm-l] Mais problemas Sobre Grupos

2003-10-20 Por tôpico Carlos Maçaranduba
Eu consegui provar a letra a o resto nao) ai vão:


Seja Z conjunto dos inteiros e  o subgrupo gerado
por x e Zm um grupo qualquer mod m. Mostre que a,b e m
inteiros( m>= 2):

a)sendo B = b (mod m) e A = a (mod m) se a divide b
entao, como subgrupos de Zm,
 esta contido em .(Esse eu consegui provar o
resto nao)

b)sendo A = a (mod m) se mdc(a,m) = 1 , entao  =
Zm.

c)sendo A = a (mod m) e D = d (mod m) se mdc(a,m) = d
, entao  = .

d) De posse das informacoes acima, determine todos os
subgrupos de (Z36 , +).

e)Mostre  que se (G , *) é um grupo multiplicativo de
ordem 2 entao G é ciclico.

f)Mostre  que se (G , *) é um grupo multiplicativo de
ordem 3 entao G é ciclico.(Sugestao: Sendo G =
{x,a,b}, x o elemento neutrode G, pense sobre o que
poderia ser o elemento ab)

g)Mostre  que se (G , *) é um grupo multiplicativo de
elemento neutro x , mostre que se y^2 =x, para cada y
em G, entao G é abeliano.(Sugestao:Note que y^2 = x
implica que y^-1 = x .Tome 2 elementos quaisquer a e b
em G e comece escrevendo ab = (ab)^-1 = ...)

Yahoo! Mail - o melhor webmail do Brasil
http://mail.yahoo.com.br
=
Instruções para entrar na lista, sair da lista e usar a lista em
http://www.mat.puc-rio.br/~nicolau/olimp/obm-l.html
=


[obm-l] Mais problemas Sobre Grupos

2003-10-20 Por tôpico Carlos Maçaranduba
Eu consegui provar a letra a o resto nao) ai vão:


Seja Z conjunto dos inteiros e  o subgrupo gerado
por x e Zm um grupo qualquer mod m. Mostre que a,b e m
inteiros( m>= 2):

a)sendo B = b (mod m) e A = a (mod m) se a divide b
entao, como subgrupos de Zm,
 esta contido em .(Esse eu consegui provar o
resto nao)

b)sendo A = a (mod m) se mdc(a,m) = 1 , entao  =
Zm.

c)sendo A = a (mod m) e D = d (mod m) se mdc(a,m) = d
, entao  = .

d) De posse das informacoes acima, determine todos os
subgrupos de (Z36 , +).

e)Mostre  que se (G , *) é um grupo multiplicativo de
ordem 2 entao G é ciclico.

f)Mostre  que se (G , *) é um grupo multiplicativo de
ordem 3 entao G é ciclico.(Sugestao: Sendo G =
{x,a,b}, x o elemento neutrode G, pense sobre o que
poderia ser o elemento ab)

g)Mostre  que se (G , *) é um grupo multiplicativo de
elemento neutro x , mostre que se y^2 =x, para cada y
em G, entao G é abeliano.(Sugestao:Note que y^2 = x
implica que y^-1 = x .Tome 2 elementos quaisquer a e b
em G e comece escrevendo ab = (ab)^-1 = ...)

Yahoo! Mail - o melhor webmail do Brasil
http://mail.yahoo.com.br
=
Instruções para entrar na lista, sair da lista e usar a lista em
http://www.mat.puc-rio.br/~nicolau/olimp/obm-l.html
=


[obm-l] Problemas de Teoria dos Grupos

2003-10-17 Por tôpico Carlos Maçaranduba
1-Seja G um grupo finito e seja H um subconjunto nao
vazio de G.Mostre que H é subgrupo de G se e somente
se H é fechado na operaçao de G.[Sugestao: Mostre que,
para cada elemento "a" pertencente a H, existe um
inteiro positivo n tal que a^n = e(elemento neutro)
].Mostre que esta propriedade nao se mantem para G
infinito.

2-Sejam G um grupo multiplicativo e seja H um subgrupo
de G.Mostre que se x pertence a G entao xHy(y é o
inverso de x em G) é tambem um subgrupo de G, sendo
xHy = 
{xhy tal que h pertence a H}.

Yahoo! Mail - o melhor webmail do Brasil
http://mail.yahoo.com.br
=
Instruções para entrar na lista, sair da lista e usar a lista em
http://www.mat.puc-rio.br/~nicolau/olimp/obm-l.html
=


[obm-l] Problemas de Teoria dos Grupos

2003-10-17 Por tôpico Carlos Maçaranduba
1-Seja G um grupo finito e seja H um subconjunto nao
vazio de G.Mostre que H é subgrupo de G se e somente
se H é fechado na operaçao de G.[Sugestao: Mostre que,
para cada elemento "a" pertencente a H, existe um
inteiro positivo n tal que a^n = e(elemento neutro)
].Mostre que esta propriedade nao se mantem para G
infinito.

2-Sejam G um grupo multiplicativo e seja H um subgrupo
de G.Mostre que se x pertence a G entao xHy(y é o
inverso de x em G) é tambem um subgrupo de G, sendo
xHy = 
{xhy tal que h pertence a H}.

Yahoo! Mail - o melhor webmail do Brasil
http://mail.yahoo.com.br
=
Instruções para entrar na lista, sair da lista e usar a lista em
http://www.mat.puc-rio.br/~nicolau/olimp/obm-l.html
=


Re:_[obm-l]_Problemas_de_Divisibilidade

2003-10-15 Por tôpico Carlos Maçaranduba
e pode fazer isso numa induçao???Supor que para uma
sequencia é valida e provar seu sucessor???
O que ate hoje eu sabia, era que primeiro era o caso
base.Depois vc supunha para um x e provava para um
x+1(ou se quiser um x-1 e provava para um x) e com
isso provado vc mostrava que era valido para qualquer
x.Sendo x=n vc esta dizendo que ele é valido para
qualquer sequencia de x -1 termos mas como vc pode
dizer que uma sequencia é valida se vc nem provou que
dado um termo n, seu sucessor é valido
Fica aqui minha duvida.

 --- "claudio.buffara" <[EMAIL PROTECTED]>
escreveu: > Bom, o do (3+raiz(5))^n + (3-raiz(5))^n
ser
> divisivel por 2^n sai por inducao.
> Pra n = 0 e n = 1 eh obvio.
> Suponha que o resultado valha para 0 <= k <= n-1.
> Sejam a = 3 + raiz(5) e b = 3 - raiz(5) ==> a+b = 6 
> e  a*b= 4.
> Alem disso:
> a^n + b^n = 
> a*a^(n-1) + b*b^(n-1) + a*b^(n-1) + b*a^(n-1) -
> a*b^(n-1) - b*a^(n-1) =
> (a + b)*(a^(n-1) + b^(n-1)) - a*b*(a^(n-2) +
> b^(n-2)) =
> 6*(a^(n-1) + b^(n-1)) - 4*(a^(n-2) + b^(n-2)) =
> 6*(p*2^(n-1)) - 4*(q*2^(n-2)) =
> 2^n*(3*p - q) ==>
> 2^n divide a^n + b^n.
> 
> Nao tive nenhuma boa ideia pro outro a nao ser usar
> forca bruta e deduzir a formula pra 1^5 + 2^5 + ...
> + n^5 (o que eh meio sacal, mas certamente
> funciona).
> 
> Um abraco,
> Claudio.
> 
> De:[EMAIL PROTECTED]
> 
> Para:[EMAIL PROTECTED]
> 
> Cópia:
> 
> Data:Tue, 14 Oct 2003 14:17:37 -0300 (ART)
> 
> Assunto:Re: [obm-l] Problemas de Divisibilidade
> 
>   
> 
> > nao consegui demonstrar..
> > --- Claudio Buffara 
> > escreveu: > Pergunta:
> > > Voce quer saber como se demonstra ou jah conhece
> uma
> > > demeonstracao e estah
> > > propondo o problema pra lista?
> > > 
> > > on 13.10.03 16:58, Carlos Maçaranduba at
> > > [EMAIL PROTECTED] wrote:
> > > 
> > > > essa da congruencia foi legal..Valeu.Tente o
> resto
> > > que
> > > > eu enviei...
> > > > 
> > > > --- Cláudio_(Prática)
> > > > escreveu: >
> > > >> - Original Message -
> > > >> From: "Carlos Maçaranduba"
> > > >> 
> > > >> To: 
> > > >> Sent: Sunday, October 12, 2003 6:32 PM
> > > >> Subject: [obm-l] Problemas de Divisibilidade
> > > >> 
> > > >> 
> > > >>> II-Se n >1 e impar => 1^n + 2^n + ... (n
> -1)^n é
> > > >>> divisivel por n.
> > > >>> 
> > > >> Usando congruências mod n, teremos:
> > > >> 1 == -(n-1)
> > > >> 2 == -(n-2)
> > > >> ...
> > > >> (n-1)/2 == -(n+1)/2
> > > >> 
> > > >> Elevando essas (n-1)/2 congruências ao
> expoente n
> > > >> (que é ímpar), obteremos:
> > > >> 1^n == -(n-1)^n
> > > >> 2^n == -(n-2)^n
> > > >> ...
> > > >> ((n-1)/2)^n == -((n+1)/2)^n
> > > >> 
> > > >> Somando tudo, ficaremos com:
> > > >> 1^n + 2^n + ... + ((n-1)/2)^n == -(n-1)^n -
> > > (n-2)^n
> > > >> - ... - ((n+1)/2)^n
> > > >> 
> > > >> Ou seja:
> > > >> 1^n + 2^n + ... + (n-2)^n + (n-1)^n == 0 (mod
> n)
> > > >> 
> > > >> O que quer dizer que:
> > > >> n divide 1^n + 2^n + ... + (n-1)^n.
> > > >> 
> > > >> Um abraço,
> > > >> Claudio.
> > > >> 
> > > >> 
> > > >
> > >
> >
>
=
> > > >> Instruções para entrar na lista, sair da
> lista e
> > > >> usar a lista em
> > > >>
> > >
> http://www.mat.puc-rio.br/~nicolau/olimp/obm-l.html
> > > >> 
> > > >
> > >
> >
>
=
> > > > 
> > > > Yahoo! Mail - o melhor webmail do Brasil
> > > > http://mail.yahoo.com.br
> > > >
> > >
> >
>
=
> > > > Instruções para entrar na lista, sair da lista
> e
> > > usar a lista em
> > > >
> > >
> http://www.mat.puc-rio.br/~nicolau/olimp/obm-l.html
> > > >
> > >
> >
>
=
> > > > 
> > > 
> > >
> >
>
=
> > > Instruções para entrar na lista, sair da lista e
> > > usar a lista em
> > >
> http://www.mat.puc-rio.br/~nicolau/olimp/obm-l.html
> > >
> >
>
=
> 
> > 
> > Yahoo! Mail - o melhor webmail do Brasil
> > http://mail.yahoo.com.br
> >
>
=
> > Instruções para entrar na lista, sair da lista e
> usar a lista em
> >
> http://www.mat.puc-rio.br/~nicolau/olimp/obm-l.html
> >
>
=
> > 
> >  

Yahoo! Mail - o melhor webmail do Brasil
http://mail.yahoo.com.br
=
Instruções para entrar na lista, sair da lista e usar a lista em
http://www.mat.puc-rio.br/~nicolau/olimp/obm-l.html
=


Re: [obm-l] TEORIA DOS JOGOS

2003-10-15 Por tôpico Carlos Maçaranduba
Procurando por teoria dos jogos no google achei:
http://www.fesppr.br/~inacio/ELAVIO/TEORIA%20DOS%20JOGOS.doc
http://c2.com/cgi/wiki?TeoriaUtilidade

quem quiser continuar minha pesquisa nas paginas em
portugues(parei na 41): 
http://www.google.com.br/search?q=%22teoria+dos+jogos&hl=pt-BR&lr=lang_pt&ie=UTF-8&oe=UTF-8&start=400&sa=N


--- Alexandre Daibert <[EMAIL PROTECTED]>
escreveu: > Olá Jorge,
> 
> Olha, não está explícito se C "DISSE" q valia muito
> mais. Vou considerar 
> que disse.
> Se B quer uma compensação para não fazer lances, ele
> não tem tanto 
> interesse no quadro. Provalvelmente ele quer tirar
> vantagem da situação 
> e ganhar algum dinheiro em cima.
> Devemos considerar o seguinte. Se C se dispõe
> prontamente a pagar uma 
> quantia muito alta, B saberá q ele pode pagar ainda
> mais e pedirá mais. 
> C (ao menos se deve mostrar disposto a) pagar uma
> quantia baixa. Outro 
> fato a considerar é que mesmo que B ganhe 1 dólar, a
> princípio seria 
> vantajoso para ele, pois ele não tirou nenhum dólar
> do bolso e obteve 
> lucro sem riscos. Agora veja que B para recuperar um
> x em dinheiro deve 
> comprar o quadro por um preço e posteriormente
> vendê-lo por um preço x 
> maior do que comprou. Repare que ele não conseguirá
> comprar o quadro por 
> 15, pois C disse que o quadro valia muito mais. C
> tende, em uma disputa 
> fazer lances altos em contrapartida aos lances de B.
> Visto isso, B 
> estaria em "desvantagem" na disputa, a princípio.
> logo, se houver leilão:
> -se B chegar a comprar o quadro, pagará um alto
> preço por este, pois C 
> está disposto a pagar caro e a disputa será intensa.
> Em uma posterior 
> revenda, deveria vendê-lo por este preço caro pelo
> qual comprou mais o 
> x, que é o que C ofereceu, mais um k (o k
> representaria a vantagem em 
> relação a ter aceitado a proposta indecorosa...)
> -se C comprar o quadro, ou pagará um alto preço (o q
> para ele parece 
> razoável) ou pagará um preço baixo, pois a atividade
> para B pode lhe 
> parecer não interessante logo no princípio do leilão
> (o que é o mais 
> provável). Repare que o preço mínimo, neste caso,
> seria um pouco acima 
> de 15.
> 
> Visto isso, percebemos que a compra do quadro por B
> se revelará 
> provavelmente desvantajosa. É vantagem para B fazer
> negócios e ganhar 
> algo em cima (ao menos é mais vantagem que entrar na
> disputa com a real 
> intenção de comprar o quadro)
> A compra do quadro por C a um preço baixo é o evento
> mais provável, 
> visto que B não estaria muito disposto a fazer
> lances altos.
> 
> Depois de estudado tudo isto, poderia afirmar que C
> deve estar disposto 
> a pagar uma quantia menor que 5 dólares. Uma quantia
> igual a 5 dólares 
> se revelaria excepcionalmente vantajosa para B e nem
> tão vantajosa para 
> C, pois ele "ganharia" com isso alguns poucos
> dólares (menos que cinco 
> provavelmente) que é a diferença que ocorreria em um
> provável lance 
> final do leilão (pouco mais de $15 como visto) - $15
> O razoável aos dois seria algo em torno de $3, pois
> B ganharia de graça 
> $3, "ganhando" C algo a mais que $3 [lance final -
> 13]. Repare que os $3 
> são iguais ao X a que nos referimos no início do
> problema. Quanto maior, 
> menor a vantagem de B entrar no leilão
> Quantias mais baixas poderiam ser oferecidas a B.
> Como em toda a 
> negociação, nunca começamos mostrando todo o nosso
> potencial, até aonde 
> podemos chegar. Logicamente, C deveria começar
> propondo $1,5 ou $2 
> dólares para tentar persuadir B a aceitar um valor
> menor, tendo como 
> limite de valor de aceitação os $3 dólares.
> 
> Não sei se está correto, mas sinceramente é o que eu
> faria em uma 
> situação dessas
> 
> Quanto as desculpas pelo envio deste problema e
> considerá-lo muito off, 
> sinceramente eu discordo e acho q problemas deste
> tipo são muito 
> enriquecedores para a lista. Aliás, vc teria algum
> endereço com material 
> em português sobre este assunto? Se tiver eu
> agradeço enormemente.
> 
> Abraços,
> Alexandre Daibert
> 
> 
> 
> [EMAIL PROTECTED] escreveu:
> 
> >Ok! Nicolau, obrigado mais uma vez pelo Tira-Teima,
> pois estava aguardando uma 
> >resposta que coincidisse com a enviada pelo prof.
> André Toom-UFPE (CAMPEÃO!)
> >
> >Nobres Colegas! Este assunto que estou enviando,
> apesar de um pouco indigesto, 
> >trata-se de moderníssima disciplina com
> propriedades matemáticas inéditas. Caso 
> >haja algum interessado, gostaria da sua opinião,
> pois não há resposta no livro!
> >
> >Um quadro deve ser vendido em leilão e os lances
> começam com $10. B diz que o 
> >quadro vale $15; C acha que o quadro vale muito
> mais. B e C são os únicos 
> >interessados potenciais, e B pede a C alguma
> compensação para deixar de fazer 
> >lances. Ignorando os problemas éticos, que quantia
> poderia C dispor-se a pagar?
> >
> >
> >Prometo! não mais trazer à lista este assunto um
> tanto offResposta
> >
> >
> >
> >___

Re: [obm-l] TEORIA DOS JOGOS

2003-10-14 Por tôpico Carlos Maçaranduba
on de encontrar boas referencias sobre este assunto???

 --- [EMAIL PROTECTED] escreveu: > Ok! Nicolau,
obrigado mais uma vez pelo Tira-Teima,
> pois estava aguardando uma 
> resposta que coincidisse com a enviada pelo prof.
> André Toom-UFPE (CAMPEÃO!)
> 
> Nobres Colegas! Este assunto que estou enviando,
> apesar de um pouco indigesto, 
> trata-se de moderníssima disciplina com propriedades
> matemáticas inéditas. Caso 
> haja algum interessado, gostaria da sua opinião,
> pois não há resposta no livro!
> 
> Um quadro deve ser vendido em leilão e os lances
> começam com $10. B diz que o 
> quadro vale $15; C acha que o quadro vale muito
> mais. B e C são os únicos 
> interessados potenciais, e B pede a C alguma
> compensação para deixar de fazer 
> lances. Ignorando os problemas éticos, que quantia
> poderia C dispor-se a pagar?
> 
> 
> Prometo! não mais trazer à lista este assunto um
> tanto offResposta
> 
> 
> 
> 
> WebMail UNIFOR - http://www.unifor.br
>
=
> Instruções para entrar na lista, sair da lista e
> usar a lista em
> http://www.mat.puc-rio.br/~nicolau/olimp/obm-l.html
>
= 

Yahoo! Mail - o melhor webmail do Brasil
http://mail.yahoo.com.br
=
Instruções para entrar na lista, sair da lista e usar a lista em
http://www.mat.puc-rio.br/~nicolau/olimp/obm-l.html
=


Re: [obm-l] Problemas de Divisibilidade

2003-10-14 Por tôpico Carlos Maçaranduba
nao consegui demonstrar..
 --- Claudio Buffara <[EMAIL PROTECTED]>
escreveu: > Pergunta:
> Voce quer saber como se demonstra ou jah conhece uma
> demeonstracao e estah
> propondo o problema pra lista?
> 
> on 13.10.03 16:58, Carlos Maçaranduba at
> [EMAIL PROTECTED] wrote:
> 
> > essa da congruencia foi legal..Valeu.Tente o resto
> que
> > eu enviei...
> > 
> > --- Cláudio_(Prática)
> > <[EMAIL PROTECTED]> escreveu: >
> >> - Original Message -
> >> From: "Carlos Maçaranduba"
> >> <[EMAIL PROTECTED]>
> >> To: <[EMAIL PROTECTED]>
> >> Sent: Sunday, October 12, 2003 6:32 PM
> >> Subject: [obm-l] Problemas de Divisibilidade
> >> 
> >> 
> >>> II-Se n >1 e impar => 1^n + 2^n + ... (n -1)^n é
> >>> divisivel por n.
> >>> 
> >> Usando congruências mod n, teremos:
> >> 1 == -(n-1)
> >> 2 == -(n-2)
> >> ...
> >> (n-1)/2 == -(n+1)/2
> >> 
> >> Elevando essas (n-1)/2 congruências ao expoente n
> >> (que é ímpar), obteremos:
> >> 1^n == -(n-1)^n
> >> 2^n == -(n-2)^n
> >> ...
> >> ((n-1)/2)^n == -((n+1)/2)^n
> >> 
> >> Somando tudo, ficaremos com:
> >> 1^n + 2^n + ... + ((n-1)/2)^n == -(n-1)^n -
> (n-2)^n
> >> - ... - ((n+1)/2)^n
> >> 
> >> Ou seja:
> >> 1^n + 2^n + ... + (n-2)^n + (n-1)^n == 0 (mod n)
> >> 
> >> O que quer dizer que:
> >> n divide 1^n + 2^n + ... + (n-1)^n.
> >> 
> >> Um abraço,
> >> Claudio.
> >> 
> >> 
> >
>
=
> >> Instruções para entrar na lista, sair da lista e
> >> usar a lista em
> >>
> http://www.mat.puc-rio.br/~nicolau/olimp/obm-l.html
> >> 
> >
>
=
> > 
> > Yahoo! Mail - o melhor webmail do Brasil
> > http://mail.yahoo.com.br
> >
>
=
> > Instruções para entrar na lista, sair da lista e
> usar a lista em
> >
> http://www.mat.puc-rio.br/~nicolau/olimp/obm-l.html
> >
>
=
> > 
> 
>
=
> Instruções para entrar na lista, sair da lista e
> usar a lista em
> http://www.mat.puc-rio.br/~nicolau/olimp/obm-l.html
>
= 

Yahoo! Mail - o melhor webmail do Brasil
http://mail.yahoo.com.br
=
Instruções para entrar na lista, sair da lista e usar a lista em
http://www.mat.puc-rio.br/~nicolau/olimp/obm-l.html
=


Re: [obm-l] Re:_[obm-l]_Interpretaçao_do_corpo_R[x]/(x^2_+_1)

2003-10-13 Por tôpico Carlos Maçaranduba
Nao consegui ver essa magica que vc diz vê na mensagem
abaixo.
> agora a mágica da coisa... tome o elemento x +  1> em R[x]/,
> veja que esse elemento é raiz do polinômio x² + 1,
> pois (x + )² =
> (x² + 1) +  = 0!

Isso aqui"x + "nao é x+f(x)*(x^2+1)??? Pq vc
eleva ao quadrado e como chega em (x² + 1)+ ??? 

> (que na verdade é um corpo pois x²+1 é irredutível)
> são representados por
> polinômios de grau 1 em x, logo são da forma ax + b,
> sabendo que o x e o i
> são a mesma coisa, vemos que os elementos desse
> corpo são da forma ai + b,
> com a e b reais... preciso ser mais formal que isso?

Vc define que x e i sao a mesma coisa???Eu entendi que
os elementos representantes devem ser os restos
possiveis da divisao por x^2 + 1 que no caso ,a
cardinalidade é igual a dos Reais. 
O que vinha antes dessa mensagem eu entendi
direitinhoValeu pela explicacao
simples.Aproveitando, o Teorema de Lagrange para
Grupos vale tambem para polinomios, como o grupo
aditivo de R[x]/(x^2 + 1)??E se x^2 + 1 nao fosse
irredutivel em R[x] quais subgrupos existiriam???




Yahoo! Mail - o melhor webmail do Brasil
http://mail.yahoo.com.br
=
Instruções para entrar na lista, sair da lista e usar a lista em
http://www.mat.puc-rio.br/~nicolau/olimp/obm-l.html
=


Re: [obm-l] Re:_[obm-l]_Interpretaçao_do_corpo_R[x]/(x^2_+_1)

2003-10-13 Por tôpico Carlos Maçaranduba
Nao consegui ver essa magica que vc diz vê na mensagem
abaixo.
> agora a mágica da coisa... tome o elemento x +  1> em R[x]/,
> veja que esse elemento é raiz do polinômio x² + 1,
> pois (x + )² =
> (x² + 1) +  = 0!

Isso aqui"x + "nao é x+f(x)*(x^2+1)??? Pq vc
eleva ao quadrado e como chega em (x² + 1)+ ??? 

> (que na verdade é um corpo pois x²+1 é irredutível)
> são representados por
> polinômios de grau 1 em x, logo são da forma ax + b,
> sabendo que o x e o i
> são a mesma coisa, vemos que os elementos desse
> corpo são da forma ai + b,
> com a e b reais... preciso ser mais formal que isso?

Vc define que x e i sao a mesma coisa???Eu entendi que
os elementos representantes devem ser os restos
possiveis da divisao por x^2 + 1 que no caso ,a
cardinalidade é igual a dos Reais. 
O que vinha antes dessa mensagem eu entendi
direitinhoValeu pela explicacao
simples.Aproveitando, o Teorema de Lagrange para
Grupos vale tambem para polinomios, como o grupo
aditivo de R[x]/(x^2 + 1)??E se x^2 + 1 nao fosse
irredutivel em R[x] quais subgrupos existiriam???




Yahoo! Mail - o melhor webmail do Brasil
http://mail.yahoo.com.br
=
Instruções para entrar na lista, sair da lista e usar a lista em
http://www.mat.puc-rio.br/~nicolau/olimp/obm-l.html
=


Re: [obm-l] Problemas de Divisibilidade

2003-10-13 Por tôpico Carlos Maçaranduba
essa da congruencia foi legal..Valeu.Tente o resto que
eu enviei...

 --- Cláudio_(Prática)
<[EMAIL PROTECTED]> escreveu: > 
> - Original Message -----
> From: "Carlos Maçaranduba"
> <[EMAIL PROTECTED]>
> To: <[EMAIL PROTECTED]>
> Sent: Sunday, October 12, 2003 6:32 PM
> Subject: [obm-l] Problemas de Divisibilidade
> 
> 
> > II-Se n >1 e impar => 1^n + 2^n + ... (n -1)^n é
> > divisivel por n.
> >
> Usando congruências mod n, teremos:
> 1 == -(n-1)
> 2 == -(n-2)
> ...
> (n-1)/2 == -(n+1)/2
> 
> Elevando essas (n-1)/2 congruências ao expoente n
> (que é ímpar), obteremos:
> 1^n == -(n-1)^n
> 2^n == -(n-2)^n
> ...
> ((n-1)/2)^n == -((n+1)/2)^n
> 
> Somando tudo, ficaremos com:
> 1^n + 2^n + ... + ((n-1)/2)^n == -(n-1)^n - (n-2)^n
> - ... - ((n+1)/2)^n
> 
> Ou seja:
> 1^n + 2^n + ... + (n-2)^n + (n-1)^n == 0 (mod n)
> 
> O que quer dizer que:
> n divide 1^n + 2^n + ... + (n-1)^n.
> 
> Um abraço,
> Claudio.
> 
>
=
> Instruções para entrar na lista, sair da lista e
> usar a lista em
> http://www.mat.puc-rio.br/~nicolau/olimp/obm-l.html
>
= 

Yahoo! Mail - o melhor webmail do Brasil
http://mail.yahoo.com.br
=
Instruções para entrar na lista, sair da lista e usar a lista em
http://www.mat.puc-rio.br/~nicolau/olimp/obm-l.html
=


[obm-l] Interpretaçao do corpo R[x]/(x^2 + 1)

2003-10-12 Por tôpico Carlos Maçaranduba
 Qual sao os elementos de R[x]/(x^2 + 1)???Sao todos
os restos de polinomios de coeficientes reais que sao
divididos por x^2 + 1???Entao esse resto poderá ser um
polinomio???Pq se diz que ele é isomorfo ao corpo dos
complexos???É pq a raiz de x^2 + 1 é i(unidade
imaginaria)???Quantos elementos possui este corpo??

Yahoo! Mail - o melhor webmail do Brasil
http://mail.yahoo.com.br
=
Instruções para entrar na lista, sair da lista e usar a lista em
http://www.mat.puc-rio.br/~nicolau/olimp/obm-l.html
=


[obm-l] Problemas de Divisibilidade

2003-10-12 Por tôpico Carlos Maçaranduba
I-Para todo n:

a)Mostrar que 3*(1^5 + 2^5 + ... n^5) é divisivel por
1^3 + 2^3 + ... n^3.

b)Sendo raiz = 5^(1/2) mostrar que
(3+ raiz)^n + (3 - raiz)^n é divisivel por 2^n. 

II-Se n >1 e impar => 1^n + 2^n + ... (n -1)^n é
divisivel por n.



Yahoo! Mail - o melhor webmail do Brasil
http://mail.yahoo.com.br
=
Instruções para entrar na lista, sair da lista e usar a lista em
http://www.mat.puc-rio.br/~nicolau/olimp/obm-l.html
=


Re: [obm-l] Algoritmo de Graham

2003-10-01 Por tôpico Carlos Maçaranduba

para nota de esclarecimento ,"scheduling" em portugues
é escalonamento..esse problema é interessante para
computaçao pois a CPU dos computadores possui uma
politica de escalonamento para os programas em
execuçao(mais neste caso é uma maquina só, a CPU).Se
eu nao me engano esse problema é NP e o que a turma
tenta fazer é se aproximar polinomialmente da soluçao
otima.Nesse caso, o algoritmo de Graham é 33% mais
custoso. 

 --- Claudio Buffara <[EMAIL PROTECTED]>
escreveu: > Oi, Pessoal:
> 
> O Domingos Jr. me falou de um problema interessante,
> envolvendo o algoritmo
> de Graham para "scheduling" (qual a palavra em
> portugues?):
> 
> Suponha que vc tem m máquinas idênticas e n tarefas
> (cujo tempo de execução
> está num vetor v[1..n] de reais positivos).
> O algoritmo de Graham é assim:
> - ordene v de forma decrescente (v(1) >= v(2) >= ...
> >= v(n));
> - na i-ésima (1 <= i <= n) iteração aloque a i-ésima
> tarefa na máquina mais
> livre (aquela cuja soma dos respectivos v(k)'s é a
> menor de todas)
> (se há empate coloque na máquina de índice menor,
> por exemplo).
>  
> Mostre que o algoritmo acima sempre dá uma solução
> cujo tempo de execução é
> no máximo igual a 4/3 do ótimo.
>  
> Um abraco,
> Claudio.
> 
>
=
> Instruções para entrar na lista, sair da lista e
> usar a lista em
> http://www.mat.puc-rio.br/~nicolau/olimp/obm-l.html
>
= 

Yahoo! Mail - o melhor webmail do Brasil
http://mail.yahoo.com.br
=
Instruções para entrar na lista, sair da lista e usar a lista em
http://www.mat.puc-rio.br/~nicolau/olimp/obm-l.html
=


Re: [obm-l] Metodo Geral de Racionalizaçao

2003-09-28 Por tôpico Carlos Maçaranduba
Pessoal esse metodo  que Dirichlet "quase" mostrou(nao
se preocupe Dirichlet, eu entendo sua falta de
tempo..hehe), eu entendi , mas parece que existe outro
mais elegante , que usa teorema do isomorfismo entre
aneis e extensao de corpos conhecido como metodo de
Cauchy-Kronecker de achar inversos multiplicativos.Eu
estou tentando entender isso, tentando encaixar todas
essas ideias mais ainda nao vi a luz.Inclusive a
sugestao da questao abaixo tem tudo a ver com esse
metodo.Tentem fazer pela sugestao: 
 
PROBLEMA
Racionalizar o denominador da fraçao  
(1 - 2^1/3) / (1 + 2^1/3 + 4^1/3), isto é,escrever a
fraçao dada na forma "a + b*(2^1/3) + c*(4^1/3)" com
a, b,c pertencente aos racionais.
(Sugestão: Determinar o polinomio minimo de 2^1/3
sobre os Racionais e usar o algoritmo de divisao
euclidiana apropriadamente.)







--- Claudio Buffara <[EMAIL PROTECTED]>
escreveu: > on 24.09.03 15:02, Carlos Maçaranduba at
> [EMAIL PROTECTED] wrote:
> 
> > --- Johann Peter Gustav Lejeune Dirichlet
> > <[EMAIL PROTECTED]> escreveu: > Esse
> > assunto ja foi muito discutido ha um ano nessa
> >> lista e entao nao vou falar muito.
> >> Basicamente a ideia e obter o polinomio minimal
> do
> >> denominador e fazer o numerador inteiro.Por
> exemplo
> >> pegue 1/(2^1/2+2^1/3).
> >> Se x e o denominador entao x-2^1/3=2^1/2 ou
> >> (x-2^1/3)^2=2, e assim sendo x^2
> -2*2^1/3*x+2^2/3=2
> >> A partir dai voce tenta destruir as potencias uma
> a
> >> uma:isola de um lado e eleva loucamente!
> >  
> > sim ai eu acho uma equacao e como concluo???
> > O artigo de shine esta em latex e eu nao tenho
> > visualizador
> >  
> >> Enfim e isso...
> >> PS:se voce estudar um pouco de polinomios no
> atrigo
> >> do Shine na Semana Olimpica,vai entender um pouco
> >> disso.
> > 
> Oi, Macaranduba:
> 
> Como sempre, somos obrigados a aguentar as mensagens
> cripticas e pela metade
> do Dirichlet...
> 
> O artigo do Shine tem um exercicio que pede para:
> i) achar o polinomio minimal de a = 2^(1/2) +
> 3^(1/3);
> ii) racionalizar o denominador de 1/(2^(1/2) +
> 3^(1/3))
> 
> Esse exercicio ilustra bem a tecnica.
> 
> i) O polinomio minimal pedido eh obtido elevando-se
> ao cubo a equacao:
> x - 2^(1/2) = 3^(1/3),
> depois agrupando os termos com 2^(1/2) de um lado e
> elevando-se ao quadrado.
> No fim, voce chega em:
> x^6 - 6x^4 - 6x^3 + 12x^2 - 36x + 1 = 0, ou seja, o
> polinomio minimal eh:
> p(x) = x^6 - 6x^4 - 6x^3 + 12x^2 - 36x + 1
> 
> ii) a eh raiz desse polinomio. Logo:
> a^6 - 6a^4 - 6a^3 + 12a^2 - 36a + 1 = 0 ==>
> 
> 1/a = -a^5 + 6a^3 + 6a^2 - 12a + 36
> 
> Repare que o lado esquerdo eh justamente o que
> queremos racionalizar e o
> lado direito eh uma FUNCAO RACIONAL de a (de fato,
> um polinomio) COM
> DENOMINADOR RACIONAL (de fato, igual a 1).
> 
> Dah um pouco de trabalho pra calcular, mas resolve o
> problema...
> 
> 
> Um abraco,
> Claudio.
> 
>
=
> Instruções para entrar na lista, sair da lista e
> usar a lista em
> http://www.mat.puc-rio.br/~nicolau/olimp/obm-l.html
>
= 


=
Instruções para entrar na lista, sair da lista e usar a lista em
http://www.mat.puc-rio.br/~nicolau/olimp/obm-l.html
=


Re: [obm-l] Metodo Geral de Racionalizaçao

2003-09-24 Por tôpico Carlos Maçaranduba
 --- Johann Peter Gustav Lejeune Dirichlet
<[EMAIL PROTECTED]> escreveu: > Esse
assunto ja foi muito discutido ha um ano nessa
> lista e entao nao vou falar muito.
> Basicamente a ideia e obter o polinomio minimal do
> denominador e fazer o numerador inteiro.Por exemplo
> pegue 1/(2^1/2+2^1/3).
> Se x e o denominador entao x-2^1/3=2^1/2 ou
> (x-2^1/3)^2=2, e assim sendo x^2 -2*2^1/3*x+2^2/3=2
> A partir dai voce tenta destruir as potencias uma a
> uma:isola de um lado e eleva loucamente!


sim ai eu acho uma equacao e como concluo???
O artigo de shine esta em latex e eu nao tenho
visualizador







> Enfim e isso...
> PS:se voce estudar um pouco de polinomios no atrigo
> do Shine na Semana Olimpica,vai entender um pouco
> disso.


___
Desafio AntiZona: participe do jogo de perguntas e respostas que vai
dar um Renault Clio, computadores, câmeras digitais, videogames e muito
mais! www.cade.com.br/antizona
=
Instruções para entrar na lista, sair da lista e usar a lista em
http://www.mat.puc-rio.br/~nicolau/olimp/obm-l.html
=


[obm-l] Metodo Geral de Racionalizaçao

2003-09-23 Por tôpico Carlos Maçaranduba
Alguem sabe um metodo geral de racionalizaçao de denominadores??

___
Desafio AntiZona: participe do jogo de perguntas e respostas que vai
dar um Renault Clio, computadores, câmeras digitais, videogames e muito
mais! www.cade.com.br/antizona
=
Instruções para entrar na lista, sair da lista e usar a lista em
http://www.mat.puc-rio.br/~nicolau/olimp/obm-l.html
=


[obm-l] Kolmogorov

2003-09-16 Por tôpico Carlos Maçaranduba
Alguem que entenda de complexidade computacional pode
fazer um paralelo entre maquinas de Turing ,
Complexidade de Kolmogorov, Entropia.Em outras
palavras explicar  a relação desses conceitos entre
si.
Pelo que entendi de inicio,são diferentes formas de se
encarar um dado problema.




 --- Johann Peter Gustav Lejeune Dirichlet
<[EMAIL PROTECTED]> escreveu: > Depois
de muito tempo eu nao deveria mandar um
> comentario desse tipo,ja que "o Dirichlet nunca
> mandou uma demonstraçao completa de qualquer
> problema proposto nesta lista,so manda
> referencias inuteis e dicas que nao levam a lugar
> nenhum...",entre muitos outros,mas eu nao resisto
> em te falar que a demonstraçao de que existem
> infinitos primos nas PAs de termo inicial 1 e
> razao qualquer pode ser achada no artigo
> "polinomios ciclotomicos" do Antonio Caminha
> Muniz Neto,do Ceara,no link Semana Olimpica da
> OBM,ou mesmo em  
> www.teorema.mat.br/ciclotomico.pdf
> Espero que lhe seja menos inutil...
> 
>  --- Frederico Reis Marques de Brito
> <[EMAIL PROTECTED]> escreveu: > 
> > 
> > Pessoal, como todos devem saber dada em toda 
> > progressão aritméticaem 
> > que a razão e o termo inicial são coprimos
> > existe uma quantidade infinita de 
> > primos. Este é o conhecido Teorema de
> > Dirichlet, cuja demonstração  é 
> > bastante complexa. Alguns casos especiais são
> > facilmente demonstrados como  
> > 4k+3   ou   6k+5 e já foram tratados nesta
> > lista.  Proponho então a 
> > demonstração  dos seguintes casos:
> > 10K +1e4k +1 , especialmente o primeiro
> > deles, poias embora conheça 
> > as demonstrações gostaria de obter provas mais
> > simples das de que tenho 
> > conhecimento.
> > Se alguém tiver uma idéia, por favor
> > escreva-me.
> > 
> > Abraços,
> > Frederico.
> > 
> >
>
_
> > MSN Messenger: converse com os seus amigos
> > online.  
> > http://messenger.msn.com.br
> > 
> >
>
=
> > Instruções para entrar na lista, sair da lista
> > e usar a lista em
> >
> http://www.mat.puc-rio.br/~nicolau/olimp/obm-l.html
> >
>
=
> 
> 
>
___
> Conheça o novo Cadê? - Mais rápido, mais fácil e
> mais preciso.
> Toda a web, 42 milhões de páginas brasileiras e nova
> busca por imagens!
> http://www.cade.com.br
>
=
> Instruções para entrar na lista, sair da lista e
> usar a lista em
> http://www.mat.puc-rio.br/~nicolau/olimp/obm-l.html
>
= 

___
Desafio AntiZona: participe do jogo de perguntas e respostas que vai
dar um Renault Clio, computadores, câmeras digitais, videogames e muito
mais! www.cade.com.br/antizona
=
Instruções para entrar na lista, sair da lista e usar a lista em
http://www.mat.puc-rio.br/~nicolau/olimp/obm-l.html
=


[obm-l] Classes de Complexidade Computacional e Metodos Probabilisticos

2003-09-07 Por tôpico Carlos Maçaranduba
ei pessoal estou estudando complexidade computacional
de problemas e uma coisa no qual não entendi é a
relação da classe rp(random polinomial) com o metodo
de probabilistico Monte Carlo e a relação  da classe
zpp(zero probabilistic polinomial) com o metodo
probabilistico de Las Vegas.

quem souber fico grato.

___
Desafio AntiZona: participe do jogo de perguntas e respostas que vai
dar um Renault Clio, computadores, câmeras digitais, videogames e muito
mais! www.cade.com.br/antizona
=
Instruções para entrar na lista, sair da lista e usar a lista em
http://www.mat.puc-rio.br/~nicolau/olimp/obm-l.html
=


Re: [obm-l] Interpolaçao num corpo geral

2003-06-06 Por tôpico Carlos Maçaranduba
Dirichlet ,bem que vc poderia facilitar e dizer qual
ano e mes vc falou sobre esse assunto pois não existe
nada para buscar por palavra chave em
http://www.mat.puc-rio.br/~nicolau/olimp/obm-l.html.

Valeu.


 --- Johann Peter Gustav Lejeune Dirichlet
<[EMAIL PROTECTED]> escreveu: > Eu me
lembro de um artigo da Mathematical
> Excalibur...
> 
> Carlos Maçaranduba <[EMAIL PROTECTED]>
> wrote:Parece que isso tem a ver com o problema da
> interpolação em um corpo..
> 
> --- Carlos Maçaranduba 
> escreveu: > Fui perguntar como resolver este
> problema(abaixo) e
> > meu professor só disse que eu deveria dar uma
> olhada
> > no teorema chines do resto para inteiros e
> > considerar
> > para polinomios usando homomorfismo entre
> > aneis.Confesso que minha cabeça entrou em parafuso
> e
> > por isso eu perguntei a vcs qual a relaçao entre
> > teorema chines do resto e a interpolaçao de
> > Lagrange.Ai vai o problema:
> > NOTAÇÃO:
> > * -> multiplicaçao.
> > y_i -> o i-esimo y.
> > a/b -> a dividido por b
> > PROD_i=m,h,(X - a_i) -> Produtorio de todos os
> > fatores
> > 
> > (X - a_i), sendo i começando de m e indo ate h. 
> > 
> > (Interpolaçao de Lagrange)Seja k um corpo e n>=1
> um
> > numero inteiro.Provar que dados dois subconjuntos
> de
> > elementos de k ,{a_1, ... a_n+1} e {b_1, ...
> b_n+1},
> > onde a_1,...,a_n+1 sao distintos, existe um unico
> > polinomio f pertencente a k[x] de grau <=n tal que
> 
> > f(a_i) = b_i , i = 1, ... , n+1.
> > (Sugestao: Tomar q = PROD_i=1,n+1,(X - a_i) e
> > f = b_1*p_1 + ... b_n+1*p_n+1 , onde:
> > p_i = q_i /(q_i*a_i) e q_i = q /(X -a_i) )
> > 
> > 
> > 
> > 
> > 
> > 
> > 
> > 
> > 
> > 
> > 
> > 
> > 
> > 
> >
>
___
> > Yahoo! Mail
> > Mais espaço, mais segurança e gratuito: caixa
> postal
> > de 6MB, antivírus, proteção contra spam.
> > http://br.mail.yahoo.com/
> >
>
=
> > Instruções para entrar na lista, sair da lista e
> > usar a lista em
> >
> http://www.mat.puc-rio.br/~nicolau/olimp/obm-l.html
> >
>
=
> 
> 
>
___
> Yahoo! Mail
> Mais espaço, mais segurança e gratuito: caixa postal
> de 6MB, antivírus, proteção contra spam.
> http://br.mail.yahoo.com/
>
=
> Instruções para entrar na lista, sair da lista e
> usar a lista em
> http://www.mat.puc-rio.br/~nicolau/olimp/obm-l.html
>
=
> 
> 
> 
> -
> Yahoo! Mail 
> Mais espaço, mais segurança e gratuito: caixa postal
> de 6MB, antivírus, proteção contra spam. 

___
Yahoo! Mail
Mais espaço, mais segurança e gratuito: caixa postal de 6MB, antivírus, proteção 
contra spam.
http://br.mail.yahoo.com/
=
Instruções para entrar na lista, sair da lista e usar a lista em
http://www.mat.puc-rio.br/~nicolau/olimp/obm-l.html
=


Re: [obm-l] Interpolaçao num corpo geral

2003-06-05 Por tôpico Carlos Maçaranduba
onde issoqual o link??
 
--- Johann Peter Gustav Lejeune Dirichlet
<[EMAIL PROTECTED]> escreveu: > Eu me
lembro de um artigo da Mathematical
> Excalibur...
> 
> Carlos Maçaranduba <[EMAIL PROTECTED]>
> wrote:Parece que isso tem a ver com o problema da
> interpolação em um corpo..
> 
> --- Carlos Maçaranduba 
> escreveu: > Fui perguntar como resolver este
> problema(abaixo) e
> > meu professor só disse que eu deveria dar uma
> olhada
> > no teorema chines do resto para inteiros e
> > considerar
> > para polinomios usando homomorfismo entre
> > aneis.Confesso que minha cabeça entrou em parafuso
> e
> > por isso eu perguntei a vcs qual a relaçao entre
> > teorema chines do resto e a interpolaçao de
> > Lagrange.Ai vai o problema:
> > NOTAÇÃO:
> > * -> multiplicaçao.
> > y_i -> o i-esimo y.
> > a/b -> a dividido por b
> > PROD_i=m,h,(X - a_i) -> Produtorio de todos os
> > fatores
> > 
> > (X - a_i), sendo i começando de m e indo ate h. 
> > 
> > (Interpolaçao de Lagrange)Seja k um corpo e n>=1
> um
> > numero inteiro.Provar que dados dois subconjuntos
> de
> > elementos de k ,{a_1, ... a_n+1} e {b_1, ...
> b_n+1},
> > onde a_1,...,a_n+1 sao distintos, existe um unico
> > polinomio f pertencente a k[x] de grau <=n tal que
> 
> > f(a_i) = b_i , i = 1, ... , n+1.
> > (Sugestao: Tomar q = PROD_i=1,n+1,(X - a_i) e
> > f = b_1*p_1 + ... b_n+1*p_n+1 , onde:
> > p_i = q_i /(q_i*a_i) e q_i = q /(X -a_i) )
> > 
> > 
> > 
> > 
> > 
> > 
> > 
> > 
> > 
> > 
> > 
> > 
> > 
> > 
> >
>
___
> > Yahoo! Mail
> > Mais espaço, mais segurança e gratuito: caixa
> postal
> > de 6MB, antivírus, proteção contra spam.
> > http://br.mail.yahoo.com/
> >
>
=
> > Instruções para entrar na lista, sair da lista e
> > usar a lista em
> >
> http://www.mat.puc-rio.br/~nicolau/olimp/obm-l.html
> >
>
=
> 
> 
>
___
> Yahoo! Mail
> Mais espaço, mais segurança e gratuito: caixa postal
> de 6MB, antivírus, proteção contra spam.
> http://br.mail.yahoo.com/
>
=
> Instruções para entrar na lista, sair da lista e
> usar a lista em
> http://www.mat.puc-rio.br/~nicolau/olimp/obm-l.html
>
=
> 
> 
> 
> -
> Yahoo! Mail 
> Mais espaço, mais segurança e gratuito: caixa postal
> de 6MB, antivírus, proteção contra spam. 

___
Yahoo! Mail
Mais espaço, mais segurança e gratuito: caixa postal de 6MB, antivírus, proteção 
contra spam.
http://br.mail.yahoo.com/
=
Instruções para entrar na lista, sair da lista e usar a lista em
http://www.mat.puc-rio.br/~nicolau/olimp/obm-l.html
=


Re: [obm-l] Interpolaçao num corpo geral

2003-06-04 Por tôpico Carlos Maçaranduba
Parece que isso tem a ver com o problema da
interpolação em um corpo..

 --- Carlos Maçaranduba <[EMAIL PROTECTED]>
escreveu: > Fui perguntar como resolver este
problema(abaixo) e
> meu professor só disse que eu deveria dar uma olhada
> no teorema chines do resto para inteiros e
> considerar
> para polinomios usando homomorfismo entre
> aneis.Confesso que minha cabeça entrou em parafuso e
> por isso eu perguntei a vcs qual a relaçao entre
> teorema chines do resto e a interpolaçao de
> Lagrange.Ai vai o problema:
> NOTAÇÃO:
> * -> multiplicaçao.
> y_i -> o i-esimo y.
> a/b -> a dividido por b
> PROD_i=m,h,(X - a_i) -> Produtorio de todos os
> fatores
> 
> (X - a_i), sendo i começando de   m e indo ate h. 
> 
> (Interpolaçao de Lagrange)Seja k um corpo e n>=1 um
> numero inteiro.Provar que dados dois subconjuntos de
> elementos de k ,{a_1, ... a_n+1} e {b_1, ... b_n+1},
> onde a_1,...,a_n+1 sao distintos, existe um unico
> polinomio f pertencente a k[x] de grau <=n tal que 
> f(a_i) = b_i ,  i = 1, ... , n+1.
> (Sugestao: Tomar q = PROD_i=1,n+1,(X - a_i)  e
>  f = b_1*p_1 + ... b_n+1*p_n+1 , onde:
>  p_i = q_i /(q_i*a_i)  e q_i = q /(X -a_i) )
> 
> 
> 
>  
> 
> 
> 
> 
> 
> 
> 
> 
> 
> 
>
___
> Yahoo! Mail
> Mais espaço, mais segurança e gratuito: caixa postal
> de 6MB, antivírus, proteção contra spam.
> http://br.mail.yahoo.com/
>
=
> Instruções para entrar na lista, sair da lista e
> usar a lista em
> http://www.mat.puc-rio.br/~nicolau/olimp/obm-l.html
>
= 

___
Yahoo! Mail
Mais espaço, mais segurança e gratuito: caixa postal de 6MB, antivírus, proteção 
contra spam.
http://br.mail.yahoo.com/
=
Instruções para entrar na lista, sair da lista e usar a lista em
http://www.mat.puc-rio.br/~nicolau/olimp/obm-l.html
=


Re: [obm-l] Problema de aneis de polinomios

2003-06-04 Por tôpico Carlos Maçaranduba
Ei Carlos realmente ele é bom vc possui outras
referencias boas sobre algebra e afins??? 

 --- Carlos_César_de_Araújo
<[EMAIL PROTECTED]> escreveu: > Prezado
Domingos,
> 
> > Eu estudei álgebra II pelo livro do Fraghley (acho
> que a grafia não é
> esta)
> > e a definição de polinômio dele é um pouco
> diferente.
> 
> Não há nada de "errado" com a definição ali
> proposta. É a mais comum, embora
> algo restrita para CERTOS propósitos.
> 
> > Tentei buscar no google o nome do autor e o nome
> do livro, mas não
> > encontrei, por favor, indique o endereço.
> 
> Herbert Wilf é um matemático bem conhecido, autor de
> livros e inúmeros
> artigos maravilhosamente escritos, muitos publicados
> no American
> Mathematical Monthly. Deixei um link para a página
> dele numa seção do meu
> site, que acabo de consultar. O endereço é
> 
> http://www.cis.upenn.edu/~wilf/
> 
> 
> > Gostaria de conhecer mais sobre isso (topologia,
> análise, álgebra...),
> minha
> > área é computação, mas eu me interesso mto pela
> matemática... Dicas de
> > material de estudo são bem vindas (as férias estão
> chegando e vai ser o
> > período que eu vou poder me dedicar a esse tipo de
> diversão, entre outras
> > coisas!).
> 
> OK. Quando tiver tempo, enviarei a você alguns dados
> sobre isso.
> (Infelizmente, o tempo anda escasso por aqui...)
> 
> Carlos César de Araújo
> Matemática para Gregos & Troianos
> www.gregosetroianos.mat.br
> Belo Horizonte, MG
> 
>
=
> Instruções para entrar na lista, sair da lista e
> usar a lista em
> http://www.mat.puc-rio.br/~nicolau/olimp/obm-l.html
>
= 

___
Yahoo! Mail
Mais espaço, mais segurança e gratuito: caixa postal de 6MB, antivírus, proteção 
contra spam.
http://br.mail.yahoo.com/
=
Instruções para entrar na lista, sair da lista e usar a lista em
http://www.mat.puc-rio.br/~nicolau/olimp/obm-l.html
=


[obm-l] Interpolaçao de Lagrange para polinomios

2003-06-03 Por tôpico Carlos Maçaranduba
Fui perguntar como resolver este problema(abaixo) e
meu professor só disse que eu deveria dar uma olhada
no teorema chines do resto para inteiros e considerar
para polinomios usando homomorfismo entre
aneis.Confesso que minha cabeça entrou em parafuso e
por isso eu perguntei a vcs qual a relaçao entre
teorema chines do resto e a interpolaçao de
Lagrange.Ai vai o problema:
NOTAÇÃO:
* -> multiplicaçao.
y_i -> o i-esimo y.
a/b -> a dividido por b
PROD_i=m,h,(X - a_i) -> Produtorio de todos os fatores

(X - a_i), sendo i começando de   m e indo ate h. 

(Interpolaçao de Lagrange)Seja k um corpo e n>=1 um
numero inteiro.Provar que dados dois subconjuntos de
elementos de k ,{a_1, ... a_n+1} e {b_1, ... b_n+1},
onde a_1,...,a_n+1 sao distintos, existe um unico
polinomio f pertencente a k[x] de grau <=n tal que 
f(a_i) = b_i ,  i = 1, ... , n+1.
(Sugestao: Tomar q = PROD_i=1,n+1,(X - a_i)  e
 f = b_1*p_1 + ... b_n+1*p_n+1 , onde:
 p_i = q_i /(q_i*a_i)  e q_i = q /(X -a_i) )



 










___
Yahoo! Mail
Mais espaço, mais segurança e gratuito: caixa postal de 6MB, antivírus, proteção 
contra spam.
http://br.mail.yahoo.com/
=
Instruções para entrar na lista, sair da lista e usar a lista em
http://www.mat.puc-rio.br/~nicolau/olimp/obm-l.html
=


[obm-l] Interpolaçao de Lagrange para polinomios

2003-06-03 Por tôpico Carlos Maçaranduba
Fui perguntar como resolver este problema(abaixo) e
meu professor só disse que eu deveria dar uma olhada
no teorema chines do resto para inteiros e considerar
para polinomios usando homomorfismo entre
aneis.Confesso que minha cabeça entrou em parafuso e
por isso eu perguntei a vcs qual a relaçao entre
teorema chines do resto e a interpolaçao de
Lagrange.Ai vai o problema:
NOTAÇÃO:
* -> multiplicaçao.
y_i -> o i-esimo y.
a/b -> a dividido por b
PROD_i=m,h,(X - a_i) -> Produtorio de todos os fatores

(X - a_i), sendo i começando de   m e indo ate h. 

(Interpolaçao de Lagrange)Seja k um corpo e n>=1 um
numero inteiro.Provar que dados dois subconjuntos de
elementos de k ,{a_1, ... a_n+1} e {b_1, ... b_n+1},
onde a_1,...,a_n+1 sao distintos, existe um unico
polinomio f pertencente a k[x] de grau <=n tal que 
f(a_i) = b_i ,  i = 1, ... , n+1.
(Sugestao: Tomar q = PROD_i=1,n+1,(X - a_i)  e
 f = b_1*p_1 + ... b_n+1*p_n+1 , onde:
 p_i = q_i /(q_i*a_i)  e q_i = q /(X -a_i) )



 










___
Yahoo! Mail
Mais espaço, mais segurança e gratuito: caixa postal de 6MB, antivírus, proteção 
contra spam.
http://br.mail.yahoo.com/
=
Instruções para entrar na lista, sair da lista e usar a lista em
http://www.mat.puc-rio.br/~nicolau/olimp/obm-l.html
=


Re: [obm-l] Problema de aneis de polinomios

2003-06-02 Por tôpico Carlos Maçaranduba
Concordo plenamente com o que vc e Morgado disse  mas 
esta afirmação em um corpo infinito não seria falsa já
que eu posso representar este polinomio como um
produtorio infinito (x -w) para todo w pertencente ao
corpo e ele não ser necessariamente o polinomio nulo??



 --- Carlos_César_de_Araújo
<[EMAIL PROTECTED]> escreveu: > Prezado
Carlos Maçaranduba e demais colegas,
> 
> Um contra-exemplo de uma afirmação P é um exemplo da
> negação de P. No seu
> caso, P é a firmação
> 
> "Se f pertence a k[x] é tal que f(w) = 0 para todo
> elemento w pertencente a
> k, então f = 0."
> 
> Pelas regras que governam os sinais lógicos, a
> negação disto é:
> 
> "Existe f em k[x] tal que f(w)=0 para todo w em k,
> MAS f<>0."
> 
> (O "mas", aqui, é meramente enfático; do ponto de
> vista lógico, significa
> "e".) Portanto, construir um contra-exemplo para K
> finito consiste em
> apresentar:
> 
> (a) um corpo finito específico. A. C. Morgado
> ofereceu K=Z(2)={0,1}, que é
> um corpo porque 2 é primo.
> (b) um polinômio f em K[X] que se anule em todo w
> pertencente a K. A. C.
> Morgado considerou f = X^2+X. Este polinômio é nulo
> em todo w em Z(2)? Sim,
> pois f(0)=0^2+0=0 e f(1)=1^2+1=1+1=2=0. (Suponho,
> naturalmente, que você
> compreende essas passagens sem maiores explicações.)
> Portanto, f(w)=w^2+w=0
> para todo w em Z(2). Certo?
> (c) um polinômio f que satisfaça (b) e que seja
> NÃO-NULO. Isto quer dizer,
> que tenha PELO MENOS UM coeficiente diferente do
> zero (do corpo). Ora, o
> polinômio apresentado por Morgado tem como
> coeficientes 1, 1 e 0. Isto é,
> X^2+X=aX^2+bX+c, com (a,b,c)=(1,1,0). Como 1<>0 (não
> é?), este polinômio é
> não-nulo.
> 
> Em parte, foi justamente a percepção de fatos como
> em (c) que estimularam os
> matemáticos a estabelecer uma diferença entre
> "funções polinomiais" e
> "formas polinomiais". Esses conceitos deixam de
> coincidir precisamente
> quando se trabalha com corpos finitos.
> 
> Abraços,
> 
> Carlos César de Araújo
> Matemática para Gregos & Troianos
> www.gregosetroianos.mat.br
> Belo Horizonte, MG
> 
> - Original Message -
> From: "Carlos Maçaranduba"
> <[EMAIL PROTECTED]>
> To: <[EMAIL PROTECTED]>
> Sent: Saturday, May 31, 2003 6:20 PM
> Subject: Re: [obm-l] Problema de aneis de polinomios
> 
> 
> > Pode ser que seja problema de interpretação, mas
> eu
> > acho que isto não é contra-exemplo PORQUE:
> >
> > ->Pela hipotese, eu nao deveria considerar, mesmo
> para
> > um corpo finito que f(w) = 0 ,para todo elemento w
> > pertencente ao corpo finito e  CONCLUIR QUE f =0 É
> > FALSO NESTE CASO
> >
> > ->UM CONTRA-EXEMPLO BOM NAO SERIA RESPEITANDO O
> QUE EU
> > DISSE ACIMA
> >
> >
> >
> >
> > --- "A. C. Morgado" <[EMAIL PROTECTED]>
> > escreveu: > f(x) = x^2 + x  em Z(2) eh um
> > contraexemplo.
> > >
> > > Carlos Maçaranduba wrote:
> > >
> > > >
> > > >Seja k um corpo infinito.Se f pertence a k[x] é
> tal
> > > >que f(w) = 0 para todo elemento w pertencente a
> k,
> > > >então f = 0.Mostrar por exemplo que esta
> > > propriedade é
> > > >falha se k é finito.
> > > >
> > >
> >
>
>___
> > > >Yahoo! Mail
> > > >Mais espaço, mais segurança e gratuito: caixa
> > > postal de 6MB, antivírus, proteção contra spam.
> > > >http://br.mail.yahoo.com/
> > >
> >
>
>=
> > > >Instruções para entrar na lista, sair da lista
> e
> > > usar a lista em
> > >
> >http://www.mat.puc-rio.br/~nicolau/olimp/obm-l.html
> > >
> >
>
>=
> > > >
> > > >
> > > >
> > > >
> > >
> > >
> > >
> >
>
=
> > > Instruções para entrar na lista, sair da lista e
> > > usar a lista em
> > >
> http://www.mat.puc-rio.br/~nicolau/olimp/obm-l.html
> > >
> >
>
=
> >
> >
>
___
> > Yahoo! Mail
> > Mais espaço, mais segurança e gratuito:

Re: [obm-l] Problema

2003-06-02 Por tôpico Carlos Maçaranduba
Olhe eu sei quem ele é e não tive nenhuma intenção de
ofende-lo.Confesso que fiquei até surpreso ao notar a
reação de algumas para UM SIMPLES E-MAIL.Aposto que
ele possui coisas mais importantes para se preocupar.
Vamos encerrar este assunto por aqui e como conselho
,acho que eu voce e todos desta lista deveriam ignorar
dessas bobagens.


 --- Frederico Reis Marques de Brito
<[EMAIL PROTECTED]> escreveu: > Não quero fazer
patrulha ideológica, mas a sucessão
> de emails sobre esse 
> tema me causou profunda estranheza. Sou obrigado a
> dizer que além de não 
> saber o que vem a ser um polinômio, o Maçaranduba tb
> não deve ter noção de 
> quem é o professor Morgado. É evidente que todos
> somos passíveis de erros, 
> mas para se afirmar que o MOrgado deu um
> contra-exemplo "furado" é 
> necessário pensar um bocado e ter muita coragem(
> acho que eu não teria )  
> sobre isto antes, sobretudo em se tratando de um
> assunto tão simples, como o 
> que motivou todas essas mensagens.
> 
> Enfim, acho que alguns partici´pantes da lista
> deveriam sintonizar-se.
> 
> Frederico.
> 
> 
> >From: "A. C. Morgado" <[EMAIL PROTECTED]>
> >Reply-To: [EMAIL PROTECTED]
> >To: [EMAIL PROTECTED]
> >Subject: Re: [obm-l] Problema de aneis de
> polinomios
> >Date: Sat, 31 May 2003 20:27:03 -0300
> >
> >Voce sabe o que eh um polinomio? Isso que voce esta
> indicando aih eu nao 
> >sei o que eh, mas polinomio nao eh. Imagine se o
> seu corpo for R: 
> >produtorio de (x-w), w percorrendo os reais.
> >Voce sabe o que eh um polinomio nulo? Voce ja se
> deu ao trabalho de olhar a 
> >resposta do Carlos Cesar a sua pergunta?
> >Esta eh minha ultima manifestaçao a respeito.
> >
> >Carlos Maçaranduba wrote:
> >
> >>usando o seu argumento eu poderia dizer que no
> caso de
> >>um corpo infinito , eu poderia construir um
> produtorio
> >>de (x - w) infinitos para todo w que pertence ao
> >>corpo.Isto é possivel pelo teorema das raizes de
> um
> >>polinomio num corpo.Entao eu obteria um polinomio
> não
> >>nulo de infinitos fatoresNão é um polinomio
> nulo
> >>como (x - 1)(x - 0)mod2 do seu exemplo..
> >>
> >>--- "A. C. Morgado" <[EMAIL PROTECTED]>
> >>escreveu: > Um polinomio eh  nulo quando os seus
> >>coeficientes
> >>
> >>
> >>>sao nulos, o que nao eh o caso do polinomio f(x)
> = x^2 + x no corpo dos
> >>>inteiros modulo 2; dois dos coeficientes desse
> polinomio sao iguais a
> >>>1.
> >>>Entao, eh falso que f = 0.
> >>>Mas f(0) = f(1) = 0, ou seja, f(w) = 0 para todo
> w
> >>>em Z2.
> >>>
> >>>Leia a resposta que lhe foi mandada por Carlos
> >>>César de Araújo e convença-se de que isso eh um
> contra-exemplo sim..
> >>>
> >>>Carlos Maçaranduba wrote:
> >>>
> >>>
> >>>
> >>>>Pode ser que seja problema de interpretação, mas
> eu
> >>>>acho que isto não é contra-exemplo PORQUE:
> >>>>
> >>>>->Pela hipotese, eu nao deveria considerar,
> mesmo
> >>>>
> >>>>
> >>>para
> >>>
> >>>
> >>>>um corpo finito que f(w) = 0 ,para todo elemento
> w
> >>>>pertencente ao corpo finito e  CONCLUIR QUE f =0
> É
> >>>>FALSO NESTE CASO
> >>>>
> >>>>->UM CONTRA-EXEMPLO BOM NAO SERIA RESPEITANDO O
> QUE
> >>>>
> >>>>
> >>>EU
> >>>
> >>>
> >>>>DISSE ACIMA
> >>>>
> >>>>
> >>>>
> >>>>
> >>>>--- "A. C. Morgado" <[EMAIL PROTECTED]>
> >>>>escreveu: > f(x) = x^2 + x  em Z(2) eh um
> >>>>contraexemplo.
> >>>>
> >>>>
> >>>>
> >>>>
> >>>>>Carlos Maçaranduba wrote:
> >>>>>
> >>>>>
> >>>>>
> >>>>>
> >>>>>
> >>>>>>Seja k um corpo infinito.Se f pertence a k[x]
> é
> >>>>>>
> >>>>>>
> >>>tal
> >>>
> >>>
> >>>>>>que f(w) = 0 para todo elemento w pertencente
> a
> >>>>>>
> >>>>>>
> >>>k,
> >>>
> >>&

Re: [obm-l] Problema de aneis de polinomios

2003-06-01 Por tôpico Carlos Maçaranduba
usando o seu argumento eu poderia dizer que no caso de
um corpo infinito , eu poderia construir um produtorio
de (x - w) infinitos para todo w que pertence ao
corpo.Isto é possivel pelo teorema das raizes de um
polinomio num corpo.Entao eu obteria um polinomio não
nulo de infinitos fatoresNão é um polinomio nulo
como (x - 1)(x - 0)mod2 do seu exemplo..

 --- "A. C. Morgado" <[EMAIL PROTECTED]>
escreveu: > Um polinomio eh  nulo quando os seus
coeficientes
> sao nulos, o que nao 
> eh o caso do polinomio f(x) = x^2 + x no corpo dos
> inteiros modulo 2; 
> dois dos coeficientes desse polinomio sao iguais a
> 1.
> Entao, eh falso que f = 0.
> Mas f(0) = f(1) = 0, ou seja, f(w) = 0 para todo w
> em Z2.
> 
>  Leia a resposta que lhe foi mandada por Carlos
> César de Araújo e 
> convença-se de que isso eh um contra-exemplo sim..
> 
> Carlos Maçaranduba wrote:
> 
> >Pode ser que seja problema de interpretação, mas eu
> >acho que isto não é contra-exemplo PORQUE:
> >
> >->Pela hipotese, eu nao deveria considerar, mesmo
> para
> >um corpo finito que f(w) = 0 ,para todo elemento w
> >pertencente ao corpo finito e  CONCLUIR QUE f =0 É
> >FALSO NESTE CASO
> >
> >->UM CONTRA-EXEMPLO BOM NAO SERIA RESPEITANDO O QUE
> EU
> >DISSE ACIMA  
> >
> >
> > 
> >
> >--- "A. C. Morgado" <[EMAIL PROTECTED]>
> >escreveu: > f(x) = x^2 + x  em Z(2) eh um
> >contraexemplo.
> >  
> >
> >>Carlos Maçaranduba wrote:
> >>
> >>
> >>
> >>>Seja k um corpo infinito.Se f pertence a k[x] é
> tal
> >>>que f(w) = 0 para todo elemento w pertencente a
> k,
> >>>então f = 0.Mostrar por exemplo que esta
> >>>  
> >>>
> >>propriedade é
> >>
> >>
> >>>falha se k é finito.
> >>>
> >>>  
> >>>
>
>>___
> >>
> >>
> >>>Yahoo! Mail
> >>>Mais espaço, mais segurança e gratuito: caixa
> >>>  
> >>>
> >>postal de 6MB, antivírus, proteção contra spam.
> >>
> >>
> >>>http://br.mail.yahoo.com/
> >>>  
> >>>
>
>>=
> >>
> >>
> >>>Instruções para entrar na lista, sair da lista e
> >>>  
> >>>
> >>usar a lista em
> >>
> >>
>
>>>http://www.mat.puc-rio.br/~nicolau/olimp/obm-l.html
> >>>  
> >>>
>
>>=
> >>
> >>
> >>> 
> >>>
> >>>  
> >>>
> >>
> >>
> >>
>
>=
> >  
> >
> >>Instruções para entrar na lista, sair da lista e
> >>usar a lista em
>
>>http://www.mat.puc-rio.br/~nicolau/olimp/obm-l.html
> >>
> >>
> >>
>
>=
> 
> >
>
>___
> >Yahoo! Mail
> >Mais espaço, mais segurança e gratuito: caixa
> postal de 6MB, antivírus, proteção contra spam.
> >http://br.mail.yahoo.com/
>
>=
> >Instruções para entrar na lista, sair da lista e
> usar a lista em
> >http://www.mat.puc-rio.br/~nicolau/olimp/obm-l.html
>
>=
> >
> >
> >  
> >
> 
>  

___
Yahoo! Mail
Mais espaço, mais segurança e gratuito: caixa postal de 6MB, antivírus, proteção 
contra spam.
http://br.mail.yahoo.com/
=
Instruções para entrar na lista, sair da lista e usar a lista em
http://www.mat.puc-rio.br/~nicolau/olimp/obm-l.html
=


Re: [obm-l] Problema de aneis de polinomios

2003-06-01 Por tôpico Carlos Maçaranduba
Pode ser que seja problema de interpretação, mas eu
acho que isto não é contra-exemplo PORQUE:

->Pela hipotese, eu nao deveria considerar, mesmo para
um corpo finito que f(w) = 0 ,para todo elemento w
pertencente ao corpo finito e  CONCLUIR QUE f =0 É
FALSO NESTE CASO

->UM CONTRA-EXEMPLO BOM NAO SERIA RESPEITANDO O QUE EU
DISSE ACIMA  


 

--- "A. C. Morgado" <[EMAIL PROTECTED]>
escreveu: > f(x) = x^2 + x  em Z(2) eh um
contraexemplo.
> 
> Carlos Maçaranduba wrote:
> 
> > 
> >Seja k um corpo infinito.Se f pertence a k[x] é tal
> >que f(w) = 0 para todo elemento w pertencente a k,
> >então f = 0.Mostrar por exemplo que esta
> propriedade é
> >falha se k é finito.
> >
>
>___
> >Yahoo! Mail
> >Mais espaço, mais segurança e gratuito: caixa
> postal de 6MB, antivírus, proteção contra spam.
> >http://br.mail.yahoo.com/
>
>=
> >Instruções para entrar na lista, sair da lista e
> usar a lista em
> >http://www.mat.puc-rio.br/~nicolau/olimp/obm-l.html
>
>=
> >
> >
> >  
> >
> 
> 
>
=
> Instruções para entrar na lista, sair da lista e
> usar a lista em
> http://www.mat.puc-rio.br/~nicolau/olimp/obm-l.html
>
= 

___
Yahoo! Mail
Mais espaço, mais segurança e gratuito: caixa postal de 6MB, antivírus, proteção 
contra spam.
http://br.mail.yahoo.com/
=
Instruções para entrar na lista, sair da lista e usar a lista em
http://www.mat.puc-rio.br/~nicolau/olimp/obm-l.html
=


Re: [obm-l] Duvida de polinomios

2003-06-01 Por tôpico Carlos Maçaranduba
essa interpolação é para polinomios..

 --- "A. C. Morgado" <[EMAIL PROTECTED]>
escreveu: > Salvo melhor juizo, nenhuma. Talvez,
longinquamente,
> na demonstraçao de 
> ambos (as demonstraçoes sao construtivas!) se use a
> ideia de escrever o 
> que se deseja como uma soma de coisas mais simples.
> 
> Carlos Maçaranduba wrote:
> 
> > 
> >Qual a relação entre a interpolação de Lagrange e o
> 
> >teorema chines do resto??
> >
>
>___
> >Yahoo! Mail
> >Mais espaço, mais segurança e gratuito: caixa
> postal de 6MB, antivírus, proteção contra spam.
> >http://br.mail.yahoo.com/
>
>=
> >Instruções para entrar na lista, sair da lista e
> usar a lista em
> >http://www.mat.puc-rio.br/~nicolau/olimp/obm-l.html
>
>=
> >
> >
> >  
> >
> 
> 
>
=
> Instruções para entrar na lista, sair da lista e
> usar a lista em
> http://www.mat.puc-rio.br/~nicolau/olimp/obm-l.html
>
= 

___
Yahoo! Mail
Mais espaço, mais segurança e gratuito: caixa postal de 6MB, antivírus, proteção 
contra spam.
http://br.mail.yahoo.com/
=
Instruções para entrar na lista, sair da lista e usar a lista em
http://www.mat.puc-rio.br/~nicolau/olimp/obm-l.html
=


[obm-l] Duvida de polinomios

2003-05-31 Por tôpico Carlos Maçaranduba
 
Qual a relação entre a interpolação de Lagrange e o 
teorema chines do resto??

___
Yahoo! Mail
Mais espaço, mais segurança e gratuito: caixa postal de 6MB, antivírus, proteção 
contra spam.
http://br.mail.yahoo.com/
=
Instruções para entrar na lista, sair da lista e usar a lista em
http://www.mat.puc-rio.br/~nicolau/olimp/obm-l.html
=


[obm-l] Problema de aneis de polinomios

2003-05-31 Por tôpico Carlos Maçaranduba
 
Seja k um corpo infinito.Se f pertence a k[x] é tal
que f(w) = 0 para todo elemento w pertencente a k,
então f = 0.Mostrar por exemplo que esta propriedade é
falha se k é finito.

___
Yahoo! Mail
Mais espaço, mais segurança e gratuito: caixa postal de 6MB, antivírus, proteção 
contra spam.
http://br.mail.yahoo.com/
=
Instruções para entrar na lista, sair da lista e usar a lista em
http://www.mat.puc-rio.br/~nicolau/olimp/obm-l.html
=


[obm-l] Provar congruencia

2003-03-26 Por tôpico Carlos Maçaranduba
Ei pessoal talvez esse não seja tão trivial:

Seja a^b("a" elevado a "b") , a==b(mod n)("a" é
congruente a "b" modulo n) e "j mod c" o resto da
divisão de "j" por "c".

Seja x,y,p,q e n inteiros ,"n=p*q"  e "p" e "q" são
primos.

Prove que:
(x^y)==(x^( y mod[p-1]*[q-1] ) )(mod n)




___
Yahoo! Mail
O melhor e-mail gratuito da internet: 6MB de espaço, antivírus, acesso POP3, filtro 
contra spam. 
http://br.mail.yahoo.com/
=
Instruções para entrar na lista, sair da lista e usar a lista em
http://www.mat.puc-rio.br/~nicolau/olimp/obm-l.html
O administrador desta lista é <[EMAIL PROTECTED]>
=


[obm-l] Provar primo impar

2003-03-22 Por tôpico Carlos Maçaranduba
ei pessoal, como é que eu provo que qualquer número
primo impar pode ser escrito ou da forma 4n + 1 ou 
4n - 1 ??



>
= 

___
Yahoo! Mail
O melhor e-mail gratuito da internet: 6MB de espaço, antivírus, acesso POP3, filtro 
contra spam. 
http://br.mail.yahoo.com/
=
Instruções para entrar na lista, sair da lista e usar a lista em
http://www.mat.puc-rio.br/~nicolau/olimp/obm-l.html
O administrador desta lista é <[EMAIL PROTECTED]>
=


[obm-l] Qual o intuito da topologia?

2003-03-20 Por tôpico Carlos Maçaranduba
Ei pessoal, qual a motivação do estudo da topologia?Um
colega meu disse rapidamente que era o estudo das
caracteristicas que não mudavam de um objeto.Entao ele
completou afirmando que se eu pegasse uma esfera e
amassasse , haveriam caracteristicas nela no qual
seriam preservadas.Que caracteristicas são essas??Quem
puder dar uma explanação geral dessa ciencia ...me
ajude.

Agradecido.

___
Busca Yahoo!
O serviço de busca mais completo da Internet. O que você pensar o Yahoo! encontra.
http://br.busca.yahoo.com/
=
Instruções para entrar na lista, sair da lista e usar a lista em
http://www.mat.puc-rio.br/~nicolau/olimp/obm-l.html
O administrador desta lista é <[EMAIL PROTECTED]>
=


Re: [obm-l] Problema: N rainhas

2003-01-31 Por tôpico Carlos Maçaranduba
boa pergunta.Eu sei até como achar uma solução mais
não quantas

 --- Helder Suzuki <[EMAIL PROTECTED]>
escreveu: > De quantas formas podemos colocar N
rainhas em um
> tabuleiro NxN tal que nenhuma rainha possa enxergar
> outra?
> 
> obs: uma rainha enxerga outra se ambas estiverem na
> mesma coluna, linha ou diagonal.
> 
>
___
> Yahoo! GeoCities
> Tudo para criar o seu site: ferramentas fáceis de
> usar, espaço de sobra e acessórios.
> http://br.geocities.yahoo.com/
>
=
> Instruções para entrar na lista, sair da lista e
> usar a lista em
> http://www.mat.puc-rio.br/~nicolau/olimp/obm-l.html
> O administrador desta lista é
> <[EMAIL PROTECTED]>
>
= 

___
Busca Yahoo!
O serviço de busca mais completo da Internet. O que você pensar o Yahoo! encontra.
http://br.busca.yahoo.com/
=
Instruções para entrar na lista, sair da lista e usar a lista em
http://www.mat.puc-rio.br/~nicolau/olimp/obm-l.html
O administrador desta lista é <[EMAIL PROTECTED]>
=



[obm-l] Curvas de Bezier

2003-01-13 Por tôpico Carlos Maçaranduba
Alguem poderia explicar o que é issoNão entendi
para que o polinomio de Bernstein.Donde esse polinomio surgiu?

___
Busca Yahoo!
O melhor lugar para encontrar tudo o que você procura na Internet
http://br.busca.yahoo.com/
=
Instruções para entrar na lista, sair da lista e usar a lista em
http://www.mat.puc-rio.br/~nicolau/olimp/obm-l.html
O administrador desta lista é <[EMAIL PROTECTED]>
=



Re: [obm-l] Re: [obm-l]_Somatório_de_Fibonacci_com_binomio_de_Newton

2003-01-11 Por tôpico Carlos Maçaranduba
Eu sei que isso ja prova mas não tem como provar para
aquele caso particular não??
 

--- "Nicolau C. Saldanha"
<[EMAIL PROTECTED]> escreveu: > On Fri,
Jan 10, 2003 at 03:14:00PM -0300, Carlos
> Maçaranduba wrote:
> > Alguem poderia fazer a questão abaixo?
> > 
> >  Seja F_n o enésimo número de fibonacci.Seja C_x,y
> a
> > combinação de x elementos tomados y a y(x maior ou
> > igual a y).Prove o somatório abaixo:
> > 
> > C_n,0*(F_1) + C_n,1*(F_2) +C_n,n*(F_n+1) =
> F_2n+1.
> 
> O bom é provar uma identidade bem mais geral:
> 
> C_n,0 * F_m + C_n,1 * F_m+1 + ... + C_n,n * F_m+n =
> F_2n+m
> 
> que pode ser provada por indução em n. O caso n = 0
> é trivial:
> 
> C_0,0 * F_m = F_0+m
> 
> e o caso n = 1 é fácil:
> 
> C_1,0 * F_m + C_1,1 * F_m+1 = F_m+2
> 
> Supondo o caso n temos
> 
> C_n,0 * F_m   + C_n,1 * F_m+1 + C_n,2 * F_m+2 + ...
> + C_n,n * F_m+n = F_2n+m
> C_n,0 * F_m+1 + C_n,1 * F_m+2 + ...
> + C_n,n-1 * F_m+n + C_n,n * F_m+n+1 = F_2n+m+1
> 
> e somando as duas equações casando do lado esquerdo
> termos
> onde o F_* tem o mesmo índice
> (na vertical para quem a minha diagramação
> funcionar)
> temos
> 
> C_n+1,0 * F_m + C_n+1,1 * F_m+1 + C_n+1,2 * F_m+2 +
> ... + C_n+1,n * F_m+n + C_n+1,n+1 * F_m+n+1 =
> F_2n+m+2
> 
> que é o caso n+1.
> 
> []s, N.
>
=
> Instruções para entrar na lista, sair da lista e
> usar a lista em
> http://www.mat.puc-rio.br/~nicolau/olimp/obm-l.html
> O administrador desta lista é
> <[EMAIL PROTECTED]>
>
= 

___
Busca Yahoo!
O melhor lugar para encontrar tudo o que você procura na Internet
http://br.busca.yahoo.com/
=
Instruções para entrar na lista, sair da lista e usar a lista em
http://www.mat.puc-rio.br/~nicolau/olimp/obm-l.html
O administrador desta lista é <[EMAIL PROTECTED]>
=



Re: [obm-l] Re: [obm-l]_Somatório_de_Fibonacci_com_binomio_de_Newton

2003-01-11 Por tôpico Carlos Maçaranduba
Eu sei que isso ja prova mas não tem como provar para
aquele caso particular não??
 

--- "Nicolau C. Saldanha"
<[EMAIL PROTECTED]> escreveu: > On Fri,
Jan 10, 2003 at 03:14:00PM -0300, Carlos
> Maçaranduba wrote:
> > Alguem poderia fazer a questão abaixo?
> > 
> >  Seja F_n o enésimo número de fibonacci.Seja C_x,y
> a
> > combinação de x elementos tomados y a y(x maior ou
> > igual a y).Prove o somatório abaixo:
> > 
> > C_n,0*(F_1) + C_n,1*(F_2) +C_n,n*(F_n+1) =
> F_2n+1.
> 
> O bom é provar uma identidade bem mais geral:
> 
> C_n,0 * F_m + C_n,1 * F_m+1 + ... + C_n,n * F_m+n =
> F_2n+m
> 
> que pode ser provada por indução em n. O caso n = 0
> é trivial:
> 
> C_0,0 * F_m = F_0+m
> 
> e o caso n = 1 é fácil:
> 
> C_1,0 * F_m + C_1,1 * F_m+1 = F_m+2
> 
> Supondo o caso n temos
> 
> C_n,0 * F_m   + C_n,1 * F_m+1 + C_n,2 * F_m+2 + ...
> + C_n,n * F_m+n = F_2n+m
> C_n,0 * F_m+1 + C_n,1 * F_m+2 + ...
> + C_n,n-1 * F_m+n + C_n,n * F_m+n+1 = F_2n+m+1
> 
> e somando as duas equações casando do lado esquerdo
> termos
> onde o F_* tem o mesmo índice
> (na vertical para quem a minha diagramação
> funcionar)
> temos
> 
> C_n+1,0 * F_m + C_n+1,1 * F_m+1 + C_n+1,2 * F_m+2 +
> ... + C_n+1,n * F_m+n + C_n+1,n+1 * F_m+n+1 =
> F_2n+m+2
> 
> que é o caso n+1.
> 
> []s, N.
>
=
> Instruções para entrar na lista, sair da lista e
> usar a lista em
> http://www.mat.puc-rio.br/~nicolau/olimp/obm-l.html
> O administrador desta lista é
> <[EMAIL PROTECTED]>
>
= 

___
Busca Yahoo!
O melhor lugar para encontrar tudo o que você procura na Internet
http://br.busca.yahoo.com/
=
Instruções para entrar na lista, sair da lista e usar a lista em
http://www.mat.puc-rio.br/~nicolau/olimp/obm-l.html
O administrador desta lista é <[EMAIL PROTECTED]>
=



[obm-l] Somatório de Fibonacci com binomio de Newton

2003-01-10 Por tôpico Carlos Maçaranduba
Alguem poderia fazer a questão abaixo?

 Seja F_n o enésimo número de fibonacci.Seja C_x,y a
combinação de x elementos tomados y a y(x maior ou
igual a y).Prove o somatório abaixo:

C_n,0*(F_1) + C_n,1*(F_2) +C_n,n*(F_n+1) = F_2n+1.

___
Busca Yahoo!
O melhor lugar para encontrar tudo o que você procura na Internet
http://br.busca.yahoo.com/
=
Instruções para entrar na lista, sair da lista e usar a lista em
http://www.mat.puc-rio.br/~nicolau/olimp/obm-l.html
O administrador desta lista é <[EMAIL PROTECTED]>
=



[obm-l] Somatório de Fibonacci com binomio de Newton

2003-01-10 Por tôpico Carlos Maçaranduba
Alguem poderia fazer a questão abaixo?

 Seja F_n o enésimo número de fibonacci.Seja C_x,y a
combinação de x elementos tomados y a y(x maior ou
igual a y).Prove o somatório abaixo:

C_n,0*(F_1) + C_n,1*(F_2) +C_n,n*(F_n+1) = F_2n+1.

___
Busca Yahoo!
O melhor lugar para encontrar tudo o que você procura na Internet
http://br.busca.yahoo.com/
=
Instruções para entrar na lista, sair da lista e usar a lista em
http://www.mat.puc-rio.br/~nicolau/olimp/obm-l.html
O administrador desta lista é <[EMAIL PROTECTED]>
=



[obm-l] Somatório de Fibonacci com binomio de Newton

2003-01-10 Por tôpico Carlos Maçaranduba
Alguem poderia fazer a questão abaixo?

 Seja F_n o enésimo número de fibonacci.Seja C_x,y a
combinação de x elementos tomados y a y(x maior ou
igual a y).Prove o somatório abaixo:

C_n,0*(F_1) + C_n,1*(F_2) +C_n,n*(F_n+1) = F_2n+1.

___
Busca Yahoo!
O melhor lugar para encontrar tudo o que você procura na Internet
http://br.busca.yahoo.com/
=
Instruções para entrar na lista, sair da lista e usar a lista em
http://www.mat.puc-rio.br/~nicolau/olimp/obm-l.html
O administrador desta lista é <[EMAIL PROTECTED]>
=



[obm-l] CONSTRUCAO COMPUTACIONAL DE POLIGONO.

2002-12-29 Por tôpico Carlos Maçaranduba
Saudações ao pessoal da lista, quem poder ajudar
ficarei grato.

Preciso construir um poligono fechado da seguinte
forma:
-Vou definindo cada ponto no plano.
-Uma aresta é definida como sendo o segmento formando 
entre o ponto que se esta definindo atualmente e o
ponto definido anteriormente.
-O ultimo ponto liga-se ao primeiro ponto.
Ex:
P_1 LIGA-SE A P_2 , P_3 LIGA-SE A P_2, P_4 LIGA-SE A
P_3 E ASSIM SUCESSIVAMENTE ATE P_n QUE SE LIGARÁ 
A P_n-1 E P_1.(quem ler faça no papel para entender).

PROBLEMA: ESSA FORMA DE CONSTRUÇAO PODE NAO FORMAR UM
POLIGONO CASO DUAS ARESTAS SE CRUZEM.

QUESTÃO: QUE ALGORITMO(SE É QUE ELE
EXISTE)PERMITIRIA-ME SABER QUE SE EU POR UM
DETERMINADO PONTO EM UM DETERMINADO LOCAL,A ARESTA
FORMADA POR ESSE PONTO E O ANTERIOR NAO CRUZARIA COM
NENHUMA DAS ARESTAS JA FORMADAS DO POLIGONO?A
UNICA COISA QUE SE SABE É A COORDENADA X,Y DE CADA
PONTO.
OBs:Que fique claro , a construção é em TEMPO REAL ,
se a posição do ponto atual for invalida ele teria que
por o ponto em uma posicao válida(que sua aresta nao
cruze com ninguem).

Obrigado pela atenção.


___
Busca Yahoo!
O melhor lugar para encontrar tudo o que você procura na Internet
http://br.busca.yahoo.com/
=
Instruções para entrar na lista, sair da lista e usar a lista em
http://www.mat.puc-rio.br/~nicolau/olimp/obm-l.html
O administrador desta lista é <[EMAIL PROTECTED]>
=



[obm-l] CONSTRUCAO COMPUTACIONAL DE POLIGONO.

2002-12-29 Por tôpico Carlos Maçaranduba
Saudações ao pessoal da lista, quem poder ajudar
ficarei grato.

Preciso construir um poligono fechado da seguinte
forma:
-Vou definindo cada ponto no plano.
-Uma aresta é definida como sendo o segmento formando 
entre o ponto que se esta definindo atualmente e o
ponto definido anteriormente.
-O ultimo ponto liga-se ao primeiro ponto.
Ex:
P_1 LIGA-SE A P_2 , P_3 LIGA-SE A P_2, P_4 LIGA-SE A
P_3 E ASSIM SUCESSIVAMENTE ATE P_n QUE SE LIGARÁ 
A P_n-1 E P_1.(quem ler faça no papel para entender).

PROBLEMA: ESSA FORMA DE CONSTRUÇAO PODE NAO FORMAR UM
POLIGONO CASO DUAS ARESTAS SE CRUZEM.

QUESTÃO: QUE ALGORITMO(SE É QUE ELE
EXISTE)PERMITIRIA-ME SABER QUE SE EU POR UM
DETERMINADO PONTO EM UM DETERMINADO LOCAL,A ARESTA
FORMADA POR ESSE PONTO E O ANTERIOR NAO CRUZARIA COM
NENHUMA DAS ARESTAS JA FORMADAS DO POLIGONO?A
UNICA COISA QUE SE SABE É A COORDENADA X,Y DE CADA
PONTO.
OBs:Que fique claro , a construção é em TEMPO REAL ,
se a posição do ponto atual for invalida ele teria que
por o ponto em uma posicao válida(que sua aresta nao
cruze com ninguem).

Obrigado pela atenção.


___
Busca Yahoo!
O melhor lugar para encontrar tudo o que você procura na Internet
http://br.busca.yahoo.com/
=
Instruções para entrar na lista, sair da lista e usar a lista em
http://www.mat.puc-rio.br/~nicolau/olimp/obm-l.html
O administrador desta lista é <[EMAIL PROTECTED]>
=



[obm-l] CONSTRUCAO COMPUTACIONAL DE POLIGONO.

2002-12-29 Por tôpico Carlos Maçaranduba
Saudações ao pessoal da lista, quem poder ajudar
ficarei grato.

Preciso construir um poligono fechado da seguinte
forma:
-Vou definindo cada ponto no plano.
-Uma aresta é definida como sendo o segmento formando 
entre o ponto que se esta definindo atualmente e o
ponto definido anteriormente.
-O ultimo ponto liga-se ao primeiro ponto.
Ex:
P_1 LIGA-SE A P_2 , P_3 LIGA-SE A P_2, P_4 LIGA-SE A
P_3 E ASSIM SUCESSIVAMENTE ATE P_n QUE SE LIGARÁ 
A P_n-1 E P_1.(quem ler faça no papel para entender).

PROBLEMA: ESSA FORMA DE CONSTRUÇAO PODE NAO FORMAR UM
POLIGONO CASO DUAS ARESTAS SE CRUZEM.

QUESTÃO: QUE ALGORITMO(SE É QUE ELE
EXISTE)PERMITIRIA-ME SABER QUE SE EU POR UM
DETERMINADO PONTO EM UM DETERMINADO LOCAL,A ARESTA
FORMADA POR ESSE PONTO E O ANTERIOR NAO CRUZARIA COM
NENHUMA DAS ARESTAS JA FORMADAS DO POLIGONO?A
UNICA COISA QUE SE SABE É A COORDENADA X,Y DE CADA
PONTO.
OBs:Que fique claro , a construção é em TEMPO REAL ,
se a posição do ponto atual for invalida ele teria que
por o ponto em uma posicao válida(que sua aresta nao
cruze com ninguem).

Obrigado pela atenção.


___
Busca Yahoo!
O melhor lugar para encontrar tudo o que você procura na Internet
http://br.busca.yahoo.com/
=
Instruções para entrar na lista, sair da lista e usar a lista em
http://www.mat.puc-rio.br/~nicolau/olimp/obm-l.html
O administrador desta lista é <[EMAIL PROTECTED]>
=



Re: [obm-l] Re:_[obm-l]_Re:_[obm-l]_RAIZ_CÚBICA_DE_7

2002-12-18 Por tôpico Carlos Maçaranduba
Como seria entao a raiz n-ésinma de um número primo qualquer?

___
Busca Yahoo!
O melhor lugar para encontrar tudo o que você procura na Internet
http://br.busca.yahoo.com/
=
Instruções para entrar na lista, sair da lista e usar a lista em
http://www.mat.puc-rio.br/~nicolau/olimp/obm-l.html
O administrador desta lista é <[EMAIL PROTECTED]>
=



[obm-l] Off topic: Agentes X Algoritmos

2002-11-13 Por tôpico Carlos Maçaranduba
Como sei que há pessoas aqui nesta lista que mexem com
IA(apesar de ser apenas para matemática), estou
começando a estuda-la e gostaria de saber uma
coisa.Qual a diferença basica entre  resolver um
problema algoritmicamente  e resolver um problema
usando agentesO que mudaria no problema de sair de
um labirinto  usando as duas abordagens

___
Yahoo! GeoCities
Tudo para criar o seu site: ferramentas fáceis de usar, espaço de sobra e acessórios.
http://br.geocities.yahoo.com/
=
Instruções para entrar na lista, sair da lista e usar a lista em
http://www.mat.puc-rio.br/~nicolau/olimp/obm-l.html
O administrador desta lista é <[EMAIL PROTECTED]>
=



Re: [obm-l] P e NP

2002-11-10 Por tôpico Carlos Maçaranduba
Deixa eu ver se entendi bem.Os problemas P são
resolvidos em tempo aceitavel(porque é da ordem de um
polinomio)e fornece a resposta procurada com exatidao
, por isso são deterministicos.Os NP são de ordem
exponencial e os computadores atuais levariam muito
tempo para achar a resposta e o que se faz é o uso de
técnicas probabilisticas(portanto nao deterministicas)
em tempo polinomial para se achar a resposta.Um
problema  x NP completo ,é o representante de uma
classe  de problemas que pódem ser reduzidos a x e
portanto seriam NP.
Agora uma coisa que nao ficou clara é por que se
define NP como uma verificação de resposta e não como
uma busca de resposta.È porque como a solução é
probabilistica , dado que eu a achei, devo verificar a
resposta???Se assim for,toda verificação de resposta é
em tempo polinomial???Como seria??

 

--- "Domingos Jr." <[EMAIL PROTECTED]> escreveu: >
Basicamente problemas da classe P são aqueles para
> os quais existe um
> algoritmo que determina a(s) solução(ões) em tempo
> polinomial, problemas NP
> são aqueles problemas considerados difíceis pois não
> existe solução
> polinomial, só exponencial.
> 
> Não-determinístico quer dizer que envolve algo
> aleatório, o que você leu
> provavelmente não tem muito a ver com a definição de
> NP, mas talvez com
> algum comentário a respeito de um problema
> específico.
> 
> Até pouco tempo atrás o problema de verificar se um
> número é primo (PRIMES)
> só era possível em tempo polinomial usando
> algoritmos não determinísticos
> (são algoritmos que dizem que o número é primo com
> um certo grau de
> confiança, mas não uma certeza absoluta). O
> algoritmo dos indianos, o AKS
> definitivamente colocou PRIMES em P, pois é um
> algoritmo determinístico (te
> dá absoluta certeza se o número é ou não primo) em
> tempo polinomial.
> 
> Para exemplos de problemas NP-completo temos o caso
> do caxeiro viajante
> (muito famoso), problemas de grafos, otimização
> inteira etc... uma pequena
> busca na internet vai te retornar muitos links para
> esses assuntos.
> 
> esse aqui parece ser interessante:
> http://www.dcc.ufmg.br/~wesley/aeds3/relatorio.html
> 
> - Original Message -
> From: "Carlos Maçaranduba"
> <[EMAIL PROTECTED]>
> To: <[EMAIL PROTECTED]>
> Sent: Saturday, November 09, 2002 8:26 PM
> Subject: [obm-l] P e NP
> 
> 
> > Já vi várias definiçoes sobre problemas P e NP e
> não
> > consegui entender direito.Afinal estas estimativas
> > estão relacionadas a o tempo de ACHAR UMA RESPOSTA
> QUE
> > SATISFAÇA O PROBLEMA ou COM UMA SUPOSTA RESPOSTA
> EM
> > MÂOS,VERIFICAR SE ELA É VÁLIDAO que seria
> entao
> > problemas NP-COMPLETOS???Qual o sentido do
> > "não-deterministico" do NP O que significa
> > P=NP
> > Enfim quem puder esclarecer junto com exemplos
> ficarei
> > grato.
> >
> >
> >
>
___
> > Yahoo! GeoCities
> > Tudo para criar o seu site: ferramentas fáceis de
> usar, espaço de sobra e
> acessórios.
> > http://br.geocities.yahoo.com/
> >
>
=
> > Instruções para entrar na lista, sair da lista e
> usar a lista em
> >
> http://www.mat.puc-rio.br/~nicolau/olimp/obm-l.html
> > O administrador desta lista é
> <[EMAIL PROTECTED]>
> >
>
=
> 
>
=
> Instruções para entrar na lista, sair da lista e
> usar a lista em
> http://www.mat.puc-rio.br/~nicolau/olimp/obm-l.html
> O administrador desta lista é
> <[EMAIL PROTECTED]>
>
= 

___
Yahoo! GeoCities
Tudo para criar o seu site: ferramentas fáceis de usar, espaço de sobra e acessórios.
http://br.geocities.yahoo.com/
=
Instruções para entrar na lista, sair da lista e usar a lista em
http://www.mat.puc-rio.br/~nicolau/olimp/obm-l.html
O administrador desta lista é <[EMAIL PROTECTED]>
=



[obm-l] P e NP

2002-11-09 Por tôpico Carlos Maçaranduba
 Já vi várias definiçoes sobre problemas P e NP e não
consegui entender direito.Afinal estas estimativas
estão relacionadas a o tempo de ACHAR UMA RESPOSTA QUE
SATISFAÇA O PROBLEMA ou COM UMA SUPOSTA RESPOSTA EM
MÂOS,VERIFICAR SE ELA É VÁLIDAO que seria entao
problemas NP-COMPLETOS???Qual o sentido do
"não-deterministico" do NP O que significa
P=NP
Enfim quem puder esclarecer junto com exemplos ficarei
grato.


___
Yahoo! GeoCities
Tudo para criar o seu site: ferramentas fáceis de usar, espaço de sobra e acessórios.
http://br.geocities.yahoo.com/
=
Instruções para entrar na lista, sair da lista e usar a lista em
http://www.mat.puc-rio.br/~nicolau/olimp/obm-l.html
O administrador desta lista é <[EMAIL PROTECTED]>
=



[obm-l] Grafos(novamente)

2002-10-24 Por tôpico Carlos Maçaranduba
 


 Um grafo pode ser hamiltoniano e euleriano ao mesmo
 tempo??Ou seja ter caminho hamiltoniano e caminho
 euleriano ao mesmo tempoe quanto aos ciclos???
 Podem coexistir em harmonia???
 
 ___

___
Yahoo! GeoCities
Tudo para criar o seu site: ferramentas fáceis de usar, espaço de sobra e acessórios.
http://br.geocities.yahoo.com/
=
Instruções para entrar na lista, sair da lista e usar a lista em
http://www.mat.puc-rio.br/~nicolau/olimp/obm-l.html
O administrador desta lista é <[EMAIL PROTECTED]>
=



[obm-l] Grafos

2002-09-28 Por tôpico Carlos Maçaranduba

Um grafo pode ser hamiltoniano e euleriano ao mesmo
tempo??Ou seja ter caminho hamiltoniano e caminho
euleriano ao mesmo tempoe quanto aos ciclos???
Podem coexistir em harmonia???

___
Yahoo! GeoCities
Tudo para criar o seu site: ferramentas fáceis de usar, espaço de sobra e acessórios.
http://br.geocities.yahoo.com/
=
Instruções para entrar na lista, sair da lista e usar a lista em
http://www.mat.puc-rio.br/~nicolau/olimp/obm-l.html
O administrador desta lista é <[EMAIL PROTECTED]>
=



Re: [obm-l] Caos & Mercado Financeiro

2002-07-12 Por tôpico Carlos Maçaranduba

Por que nao disse antes ??

--- Bruno <[EMAIL PROTECTED]> escreveu: > Caros
colegas,
> 
> Estou criando um grupo de estudo interdisciplinar
> afim de aplicar a abordagem de sistemas complexos ao
> mercado financeiro. Uma Asset Management norte
> americana que usa a Teoria do Caos na gestão de
> alguns de seus fundos, já mostrou interesse em
> financiar a criação desse grupo. Gostaria de saber
> se alguém se interessa pelo tema.
> 
> abraços,
> Bruno
>  

___
Yahoo! Encontros
O lugar certo para encontrar a sua alma gêmea.
http://br.encontros.yahoo.com/
=
Instruções para entrar na lista, sair da lista e usar a lista em
http://www.mat.puc-rio.br/~nicolau/olimp/obm-l.html
O administrador desta lista é <[EMAIL PROTECTED]>
=



Re: [obm-l] Progressálise_Combitmética + Duvidas sobre Logica Matematica

2002-05-31 Por tôpico Carlos Maçaranduba

 > 
> 2) Simplificar a seguinte proposicao e indicar em
> cima de cada simbolo de
> equivalencia a propriedade logica utilizada.
> 
> ~((~P -> ~Q) OU ((Q E P) <-> ~P))
  ~((P OU ~Q) OU ( ( (Q E P)-> ~P ) E (~P->(Q E P) ) )

  ~((P OU ~Q) OU ( ( ~(Q E P) OU ~P ) E ( P OU (Q E P)
) ) )  

~((P OU ~Q) OU  ( (  ~Q OU ~P OU ~P ) E ( P OU ( Q E
P)))

~((P OU ~Q) OU  ( ~Q OU ~P E ( P OU ( Q E P) ) ) ) 

~((P OU ~Q) OU  ( ~Q OU FALSO OU ~P E Q E P ) ) 
~((P OU ~Q) OU  ( ~Q E Q E FALSO ) )
~((P OU ~Q) OU  FALSO )

 ~( P OU ~Q )
   
 ~P E Q   
 
> 
> =
> []s
> Ricardo Miranda
> [EMAIL PROTECTED]
> http://rm2.hpg.ig.com.br/
> 
>
___
> Yahoo! Encontros
> O lugar certo para você encontrar aquela pessoa que
> falta na sua vida. Cadastre-se hoje mesmo!
> http://br.encontros.yahoo.com/
>
=
> Instruções para entrar na lista, sair da lista e
> usar a lista em
> http://www.mat.puc-rio.br/~nicolau/olimp/obm-l.html
> O administrador desta lista é
> <[EMAIL PROTECTED]>
>
= 

___
Yahoo! Encontros
O lugar certo para você encontrar aquela pessoa que falta na sua vida. Cadastre-se 
hoje mesmo!
http://br.encontros.yahoo.com/
=
Instruções para entrar na lista, sair da lista e usar a lista em
http://www.mat.puc-rio.br/~nicolau/olimp/obm-l.html
O administrador desta lista é <[EMAIL PROTECTED]>
=



Re: [obm-l] A Intuicao Matematica

2002-05-24 Por tôpico Carlos Maçaranduba

Segundo leibnitz(que desenvolveu em paralelo a newtow
o calculo integral e diferencial) existem verdades que
são inatas  ao espírito(o individuo já nasce com elas,
ver "Novos Ensaios sobre o Entendimento Humano" de sua
autoria ) o que explicaria porque a lógica "é lógica"
e porque individuos soubessem da verdade de certas
conjecturas sem prova-lás.Concordo com Paulo quando
fala do emocional , ele é uam das peças chaves do
sentir e vibrar com certas inferencias e
conclusoes(aliás em lógica existe uma técnica
interessante chamada dedução natural que tenta imitar
as regras de inferencia do homem).Essas questões são
interessantes também no âmbito da inteligencia
artificial.Criariamos máquinas com emoções e
intuições???Qual lógica trataria de tal coisa???Uma
das limitações da lógica é imposta pelo Teorema da
incompletude de Godel , máquinas nao seriam capazes de
lidar com isso mas nós humanos compreendemos e até
brincamos com estas loucuras(a lembrar  o paradoxo de
Russel do barbeiro) que me leva a conjecturar(que
lógica é essa?) que somos muito mais que meras
máquinas computacionais . 
Eu não posso ser provado.

 --- Paulo Santa Rita <[EMAIL PROTECTED]> escreveu: >
Ola Duda e demais
> colegas desta lista,
> 
> Eu nao sei responder a pergunta que voce fez abaixo,
> mas acredito que a 
> intuicao matematica esta para a mente de um
> matematico assim como a intuicao 
> sensivel esta para a mente humana ...
> 
> Quando voce olha para um objeto voce IMEDIATAMENTE
> deduz varias propriedades 
> SEM USAR NENHUM RACIOCINIO MEDIADOR. Por exemplo,
> voce tem uma IDEIA 
> INSTANTANEA ( INTUICAO SENSIVEL ) do tamanho, da
> cor, da forma, da 
> proximidade dele para com outros objetos, etc. Todos
> esses conhecimentos sao 
> validos e foram obtidos por INTUICAO SENSIVEL
> 9PERCEPCAO INSTANTANEA ), isto 
> e, sem que fosse necessario usar um RACIOCINIO
> LOGICO OU DISCURSSIVO para se 
> chegar a eles.
> 
> Me parece que a intuicao matematica se aproxima
> disso. Voce "olha" ( com o 
> olhar da mente matematica ) os objetos do mundo
> matematico E SENTE que eles 
> devem ter ou manter determinadas relacoes, sem que
> voce consiga, de 
> imediato, forjar uma demonstracao para estas
> relacoes ou propriedades.
> 
> Veja como Gauss fala em dois momentos :
> 
> 1) "Encontrei um maravilhoso teorema, mas,
> infelizmente, ainda nao consigo 
> demonstrar"
> 
> Ele fala sobre a lei da reciprocidade quadratica.
> Veja bem. Ele estava 
> convencido da correcao do teorema ANTES DE
> DEMONSTRA-LO. E comum voce 
> aprender em algumas escolas o seguinte : Voce so
> pode ter certeza de um 
> teorema depois de demonstra-lo !
> 
> 2) "Durante este outono, preocupei-me largamente COM
> AS CONSIDERACOES GERAIS 
> sobre as superficies curvas, o que conduz a um campo
> ilimitado ... Essas 
> pesquisas se ligam fortemente com a metafisica da
> geometria e nao e sem 
> ingentes esforcos que consigo me arracar das
> consequencias que dai advem. 
> Qual seria o verdadeiro sentido da raiz quadrada de
> -1 ? Nestes momentos, 
> sinto vibrar vivamente em mim o verdadeiro
> significado destas coisas mas 
> creio que sera terrivelmente dificil exprimir este
> significado em palavras"
> 
> Ve-se aqui que Gauss PRIMEIRO, SENTE A REALIDADE DO
> MUNDO MATEMATICO. E SO 
> DEPOIS ele vai exprimir o que sente atraves das
> teorias e formulas 
> matematicas.
> 
> Esses exemplos reforcam o que eu suspeito. A
> verdadeira inteligencia esta na 
> SENSIBILIDADE ou CAPACIDADE DE VIVENCIAR
> INTERNAMENTE E EMOTIVAMENTE os 
> objetos matematicos. Logica, demonstracao e teorias
> sao coisas que vem 
> depois. Primeiro voce pensa, vale dizer, vivenciar
> internamente e 
> emotivamente, atraves da imaginacao, os objetos e
> fatos matematicos, depois, 
> quanto mais viva for essa vivencia, mais profundas
> serao suas conclusoes e 
> mais facil ficara a posterior e necessaria
> demonstacao.
> 
> Ainda a esse respeito e interessante destacar que,
> conforme relata Voltaire, 
> uma vez perguntaram a Newton como ele havia
> descoberto a lei de gravitacao. 
> Newton respondeu : pensando continuamente sobre ela
> !
> 
> Quer dizer, imaginando e vivenciando os objetos voce
> desvela os misterios 
> que os encombrem ... Os exemplos acima mostram que e
> fundamental que o lado 
> emocional esteja presente : se o cara nao conseguir
> apreciar a beleza destas 
> coisas, nao tiver, por alguma razao, entusiasmo por
> elas, dificilmente ele 
> vai trocar os prazeres imediatos e efemeros da vida
> por elas ...
> 
> Como ultimo exemplo para reforcar minha tese, cito
> um exemplo contemporaneo.
> 
> Ha poucos meses atras eu assisti uma exposicao de um
> Matematico
> da area de equacoes diferenciais. Em verdade, de um
> Grande Matematico, com 
> varios premios internacionais e conhecido no mundo
> inteiro.
> 
> Ele explicava a ideia de DECOMPOSICAO FOCAL. Seja
> dada uma equacao 
> diferencial F(X, DX, ...)=0. A cada ponto (X,Y) do
> plano ele associava a 
> quantidade de solucoes desta equ

Re: [obm-l] Logica

2002-04-05 Por tôpico Carlos Maçaranduba

A afirmação é somente essa ???Se for não é tão dificil
de se enteder...
Imagine uma porta que pode estar aberta(A) ou fechada
e imagine que vc(V) ou outra pessoa pode abrir esta
porta..

Se eu nao abro a porta ,ela pode estar aberta??
Implicação V->A para V=0 e A=1.A resposta é sim pois
outra pessoa poderia abrir tal porta.
Se eu abrir a Porta (V=1) e a porta estiver
fechada(A=0) tal implicação é verdadeira(V->A)??
Ta na cara que nao , aliás essa é a única situação em
que a implicação é falsa.Portanto vc pode usa-la para
se basear em outros problemas no qual vc tenha
necessidade. 
--- Gustavo Martins <[EMAIL PROTECTED]> escreveu: >
Li algo que diz que, se p falso e q, verdadeiro,
> pode-se concluir que p -> q sera verdadeiro.
> Como e dito que isso vale sempre, tambem devera
> valer para este exemplo:
> 
> Vamos ver se JaP -> Pb e verdadeira, onde JaP
> significa "jogador do time a faz ponto" e Pb "ponto
> do time b". Ou seja, sempre que um jogador de A
> fizer ponto, B tambem  fara ponto.
> Entao, se num dado momento, JaP for falso e Pb,
> verdadeiro, podemos concluir que a afirmacao e
> verdadeira.
> 
> Mas sabemos que nao e.
> 
> Talvez o texto de logica que eu li esteja mal
> escrito. Alguem pode me dizer qual e a regra
> correta?
>  

___
Yahoo! Empregos
O trabalho dos seus sonhos pode estar aqui. Cadastre-se hoje mesmo no Yahoo! Empregos 
e tenha acesso a milhares de vagas abertas!
http://br.empregos.yahoo.com/
=
Instruções para entrar na lista, sair da lista e usar a lista em
http://www.mat.puc-rio.br/~nicolau/olimp/obm-l.html
O administrador desta lista é <[EMAIL PROTECTED]>
=



[obm-l] Mais Cardinalidade

2002-01-21 Por tôpico Carlos Maçaranduba

estou lendo algo sobre isso..gostaria de alguns
esclarecimentos Quais são os conjuntos de
cardinalidade alef zero??e alef mais c???Quer dizer
que temos 3 conjuntos infinitos com cardinalidades
diferentes (c , alef e alef mais c)???

No livro que eu estou olhando ele prova que a
cardinalidade do conjunto das partes de um conjunto x
é  maior que a cardinalidade do conjunto x.Entao se eu
conseguir uma função bijetora entre um conjunto y e o
conjunto das partes de x é a mesma coisa que dizer que
a cardinalidade de y é maior que a de x???è assim que
ele chega a alef???Qual o conjunto que originou o
conjunto das partes no qual é o contradominio da
função bijetora no qual tem os irracionais como
dominio???entendeu onde quero chegar??pode ser que eu
entendi errado é que o livro é em ingles e a notação é
muito complicadafico grato por quem puder
esclarecer sobre isso




 --- "Nicolau C. Saldanha"
<[EMAIL PROTECTED]> escreveu: > On Thu,
Dec 27, 2001 at 02:07:52PM -0200, Vinicius
> José Fortuna wrote:
> > Ué, eu sempre entendi que a cardinalidade de um
> conjunto fosse o número de
> > elemento do mesmo.
> > 
> > Se dois conjuntos possuem infinitos elementos, eu
> achava que a
> > cardinalidade fosse a mesma. Alguém tem um 
> conceito mais preciso de
> > cardinalidade?
> 
> Cantor. :-)
> 
> Cantor começou uma revolução na matemática ao
> descobrir que uns infinitos
> são maiores do que outros. Dois conjuntos A e B têm
> o mesmo cardinal
> (segundo Cantor) se existir uma bijeção entre A e B.
> O cardinal de A
> é menor do que o de B se existir uma função injetora
> de A para B mas
> não existir uma bijeção. Cantor demostrou que
> 
>  |N| = |Z| = |Q| = |A| < |R| = |C|
> 
> onde estes são os conjuntos de números naturais,
> inteiros, racionais,
> algébricos, reais e complexos. Em particular, isto
> demonstrava a
> existência de números transcendentes (não
> algébricos), novidade na época.
> 
> Tudo isto está em Naïve Set Theory de Halmos (e em
> um milhão de outros
> lugares).
> 
> []s, N. 

___
Yahoo! GeoCities
Tenha seu lugar na Web. Construa hoje mesmo sua home page no Yahoo! GeoCities. É fácil 
e grátis!
http://br.geocities.yahoo.com/
=
Instruções para entrar na lista, sair da lista e usar a lista em
http://www.mat.puc-rio.br/~nicolau/olimp/obm-l.html
O administrador desta lista é <[EMAIL PROTECTED]>
=



[obm-l] Mais Cardinalidade

2002-01-21 Por tôpico Carlos Maçaranduba

estou lendo algo sobre isso..gostaria de alguns
esclarecimentos Quais são os conjuntos de
cardinalidade alef zero??e alef mais c???Quer dizer
que temos 3 conjuntos infinitos com cardinalidades
diferentes (c , alef e alef mais c)???

No livro que eu estou olhando ele prova que a
cardinalidade do conjunto das partes de um conjunto x
é  maior que a cardinalidade do conjunto x.Entao se eu
conseguir uma função bijetora entre um conjunto y e o
conjunto das partes de x é a mesma coisa que dizer que
a cardinalidade de y é maior que a de x???è assim que
ele chega a alef???Qual o conjunto que originou o
conjunto das partes no qual é o contradominio da
função bijetora no qual tem os irracionais como
dominio???entendeu onde quero chegar??pode ser que eu
entendi errado é que o livro é em ingles e a notação é
muito complicadafico grato por quem puder
esclarecer sobre isso




 --- "Nicolau C. Saldanha"
<[EMAIL PROTECTED]> escreveu: > On Thu,
Dec 27, 2001 at 02:07:52PM -0200, Vinicius
> José Fortuna wrote:
> > Ué, eu sempre entendi que a cardinalidade de um
> conjunto fosse o número de
> > elemento do mesmo.
> > 
> > Se dois conjuntos possuem infinitos elementos, eu
> achava que a
> > cardinalidade fosse a mesma. Alguém tem um 
> conceito mais preciso de
> > cardinalidade?
> 
> Cantor. :-)
> 
> Cantor começou uma revolução na matemática ao
> descobrir que uns infinitos
> são maiores do que outros. Dois conjuntos A e B têm
> o mesmo cardinal
> (segundo Cantor) se existir uma bijeção entre A e B.
> O cardinal de A
> é menor do que o de B se existir uma função injetora
> de A para B mas
> não existir uma bijeção. Cantor demostrou que
> 
>  |N| = |Z| = |Q| = |A| < |R| = |C|
> 
> onde estes são os conjuntos de números naturais,
> inteiros, racionais,
> algébricos, reais e complexos. Em particular, isto
> demonstrava a
> existência de números transcendentes (não
> algébricos), novidade na época.
> 
> Tudo isto está em Naïve Set Theory de Halmos (e em
> um milhão de outros
> lugares).
> 
> []s, N. 

___
Yahoo! GeoCities
Tenha seu lugar na Web. Construa hoje mesmo sua home page no Yahoo! GeoCities. É fácil 
e grátis!
http://br.geocities.yahoo.com/
=
Instruções para entrar na lista, sair da lista e usar a lista em
http://www.mat.puc-rio.br/~nicolau/olimp/obm-l.html
O administrador desta lista é <[EMAIL PROTECTED]>
=



Re: Livro sobre Godel

2002-01-07 Por tôpico Carlos Maçaranduba

 Onde encontro tal livro???Ele é português

--- Paulo Santa Rita <[EMAIL PROTECTED]> escreveu: >
Ola Daniel e demais
> colegas desta,
> 
> Existem muitos livros sobre o Teorema de Godel. Com
> "Semelhantes ao Ultimo 
> Teorema de Fermat" talvez voce queira dizer um livro
> que nao exija 
> conhecimento previo sobre o assunto. Eu acho que e
> impossivel entender o 
> Teorema de Godel sem algum conhecimento previo sobre
> fundamentos da 
> Matematica e logica, desde que voce queira realmente
> entender a coisas e nao 
> somente saber o que falam sobre ela ...
> 
> O livro :
> 
> O Teorema de Godel e a hipotese do continuo
> Fundacao Calouste Gulbenkian
> Prof Manuel Lourenco
> 
> E uma antologia sensacional ... Tem traducao para o
> Portugues e nela voce 
> vai entender o Teorema de Godel como realmente e,
> demonstrado de mais de uma 
> maneira e discutido em seus aspectos matematicos e
> filosoficos pelo proprio 
> Godel.
> 
> TODOS OS PRE-REQUISITOS PARA ENTENDER A DEMONSTRACAO
> DE GODEL ESTAO NO 
> PROPRIO LIVRO, atraves de :
> 
> 1) Exposicao pelo Prof Manuel Lourenco de uma das
> formalizacoes da teoria 
> dos conjuntos ( Nao a de Zermelo-Frankel, mas a de
> Von Newmann-Bernays )
> 
> 2)Introducao a logica-matematica por Paul Cohen,
> curso ministrado na 
> Universidade de harvard em 1965.
> OBS : Paul Cohem diz explicitamente que o curso e
> para os matematicos nao 
> especialistas em logica-matematica adquirirem a base
> para o entendimento 
> desta area
> 
> Na antologia voce tambem vera as provas sobre a
> independencia da hipotese do 
> continuo e muitos outros resultados fundamentais
> sobre essa area. O livro 
> termina com um estudo sobre as implicacoes
> filosoficas que o teorema de 
> godel tem.
> 
> ABRE PARENTESES
> 
> OBS : Foi o proprio Paul Cohen, acima mencionado,
> que provou a independencia 
> da hipotese do continuo ... Dizer que a hipotese e
> independente significa 
> dizer que tanto a sua afirmacao ( Godel ) quanto a
> sua negacao ( Cohen ) 
> podem ser colocada ao lado dos demais axiomas da
> teoria dos conjuntos que 
> nao havera inconsistencia subsequente, isto e, ela e
> uma afirmacao 
> INDEPENDENTE : nao guarda relacao com os demais
> axiomas !
> 
> Este estado de coisas, evidentemente, nao e
> satisfatorio e aguarda que alg8m 
> de nos forneca um esclarecimento consistente.
> 
> FECHA PARENTESES
> 
> Enfim, esta antologia e um tesouro de valor
> incalculavel e serve para que 
> possamos realmente entender as coisas e falar e
> discutir sobre elas sem nos 
> fiarmos em textos meramente de divulgacao, que nao
> raro costumam explorar 
> aspectos fantasiosos ...
> 
> Um Grande abraco pra voce
> Paulo Santa Rita
> 2,1301,070102
> 
> 
> 
> 
> >From: Daniel <[EMAIL PROTECTED]>
> >Reply-To: [EMAIL PROTECTED]
> >To: Lista OBM <[EMAIL PROTECTED]>
> >Subject: Livro sobre Godel
> >Date: Fri, 04 Jan 2002 23:49:37 -0300
> >
> > Olá a todos, ocorreram alguns
> problemas no meu e-mail e perdi 
> >algumas das discussões.
> > Poderiam me dizer se existe algum
> livro sobre o Teorema de 
> >Godel, à venda, no estilo daquele "O último teorema
> de Fermat"?
> >
> >Obrigado
> > Daniel
> 
> 
> 
> 
>
_
> Converse com amigos on-line, experimente o MSN
> Messenger: 
> http://messenger.msn.com.br
>  

___
Yahoo! GeoCities
Tenha seu lugar na Web. Construa hoje mesmo sua home page no Yahoo! GeoCities. É fácil 
e grátis!
http://br.geocities.yahoo.com/



Re: Teorema de Godel

2002-01-05 Por tôpico Carlos Maçaranduba

Dá uma olhada neste endereço e explica-me por favor
que diagonal é essa.È a mesma usada por
Cantor???Ajuda-me a compreender o 1 teorema que esta
neste site.

http://www.cle.unicamp.br/prof/carnielli/teoremas_de_godel.htm


 --- Rogerio Fajardo <[EMAIL PROTECTED]>
escreveu: > 
> A idéia é criar uma sentença que diz: "eu não posso
> ser provada", ou seja, 
> uma sentença, cujo número de godel é x, que diz que
> não existe demonstração 
> para a fórmula cujo número de godel é x.
>Para entender a fórmula que godel criou, é
> necessário o conceito de 
> variável livre. A fórmula "x é primo" possui uma
> variável livre x, não 
> podemos deizer que ela é verdadeira ou falsa sem
> conhecer o valor de x. Para 
> eliminar essa variável livre, tem duas maneiras: uma
> é substituir x por um 
> número (p.ex. "7 é primo"), outra é colocar um
> quantificador ("existe x t.q. 
> x é primo"). Note que uma fórmula sem variável livre
> (que chamamos 
> "sentença") deve ser ou verdadeira ou falsa (i.e,
> sua negação verdadeira) em 
> um modelo matemático fixado (que precisa ser
> definido, mas, intuitivamente, 
> é uma interpretação para o significado das
> fórmulas). O sistema de axiomas 
> ideal deve provar ou a sentença ou sua negação. Pois
> bem, godel cria uma 
> sentença que não pode ser provada nem ela nem sua
> negação.
> 
> Para obter essa sentença, godel criou a fórmula
> PROVA(x,y,y) que significa: 
> "A sequência de fórmulas cujo número é x é uma
> demonstração da fórmula (de 
> número y) de uma variável livre, substituindo sua
> variável livre pelo valor 
> y". Por exemplo, se 1000 é o número da fórmula "x é
> primo", 
> PROVA(12345,1000,1000) diz: "12345 é o número da
> demonstração de "1000 é 
> primo".
> 
> A fórmula ¬ExPROVA(x,y,y) diz "a fórmula de número
> y, substituindo sua  
> variável livre por y, não póde ser provada". No
> nosso exemplo, 
> ¬ExPROVA(x,100,1000) diz "não existe demonstração de
> que 1000 é primo". Pois 
> bem, ¬ExPROVA(x,y,y) tem uma variável livre y, e tem
> um número (seja g esse 
> número). Portanto a fórmula ¬ExPROVA(x,g,g) é uma
> sentença (note que g não é 
> uma variável, mas um número conhecido, que eu já
> calculei). E essa sentença 
> diz: "A fórmula de número g, substituindo sua
> variável livre por g, não pode 
> ser provada". Mas quem é a fórmula de número g? É o
> próprio ¬ExPROVA(x,y,y). 
> E substituindo sua variável livre por g? É a propria
> sentença 
> ¬ExPROVA(x,g,g). Portanto, ¬ExPROVA(x,g,g)  diz
> "¬ExPROVA(x,g,g) não pode 
> ser provada", que gera o paradoxo que queríamos (uma
> sentença que diz "eu 
> não posso ser provada").
> 
> Observe que, se um sistema for consistente, eu de
> fato não consigo provar 
> ¬ExPROVA(x,g,g). Mas isso se o sistema for
> consistente (i.e., não provar uma 
> fórmula e sua negação). Caso contrário, tudo vira
> teorema, e tudo pode ser 
> provado (de uma contradição provamos qualquer
> coisa), inclusive 
> ¬ExPROVA(x,g,g). Mas se eu provar a consistência do
> sistema, eu acabei de 
> provar que ¬ExPROVA(x,g,g) não pode ser provada. Mas
> isso, como vimos, é o 
> próprio ¬ExPROVA(x,g,g), e chegamos numa
> contradição. Concluindo: a segunda 
> parte do Teorema de Godel (conhecido como segundo
> teorema de godel) diz que, 
> se um sistema for consistente, sua consistência não
> pode ser provada (dentro 
> do próprio sistema).
> 
> Uma observação importante é que, apesar de dar a
> idéia geral da 
> demonstração, a demonstração que está no site está
> longe de ser completa. 
> Fica a pergunta: como godel criou (ou provou que
> existe) a fórmula 
> PROVA(x,y,y) usando só o fato de que o sistema é
> capaz de exprimir a 
> aritmética e de que seus axiomas formam um conjunto
> recursivo (consigo 
> decidir, através de um algoritmo finito, se uma
> fórmula é axioma ou não). É 
> interessante olhar no trabalho original de godel
> ("On formally undecidable 
> propositions of principia mathematica and related
> systens") como ele 
> codifica cada axioma, e cada regra de inferência, em
> termos de relações 
> aritméticas. Repare que a fórmula indecidível
> ¬ExPROVA(x,g,g), no fundo é 
> uma gigantesca fórmula que só envolve números,
> conectivos lógicos, e as 
> operações + e *.
> 
> 
> >From: Carlos Maçaranduba <[EMAIL PROTECTED]>
> >Reply-To: [EMAIL PROTECTED]
> >To: [EMAIL PROTECTED],
> [EMAIL PROTECTED]
> >Subject: Teorema de Godel
> >Date: Wed, 2 Jan 2002 18:43:16 -0300 (ART)
> >
> >neste endereço há uma de

Teorema de Godel

2002-01-02 Por tôpico Carlos Maçaranduba

neste endereço há uma demonstração do teorema de godel
que aparentemente é simples de se entender.Alguem
poderia ver a parte que ele usa o predicado
PROVA(x,g,g) e explicar-me pq ele faz isso? 


http://www.pr.gov.br/celepar/celepar/batebyte/edicoes/2000/bb95/teorema.htm

___
Yahoo! GeoCities
Tenha seu lugar na Web. Construa hoje mesmo sua home page no Yahoo! GeoCities. É fácil 
e grátis!
http://br.geocities.yahoo.com/



Re: Completude da Geometria e Teorema de Godel

2001-12-14 Por tôpico Carlos Maçaranduba

vc disse sobre as propriedades do sistema formal e
sobre a consistencia e a completude.Como vc encara o
antagonismo das duas últimas???Vc apenas sabe o que
Godel provou ou ENTENDE BEM o que ele demonstrou???è
uma coisa de fácil entendimento como 2+2=4,ou ele
demonstrou de forma dificil de se entender e vc só
memorizou o resultado??Vc está entendendo o que quero
dizer??O que quero falar se isso é uma coisa clara
,lógica ,que está na cara ,ou um resultado avançado.  


--- Paulo Santa Rita <[EMAIL PROTECTED]> escreveu: >
Ola Rogerio e demais
> colegas desta lista,
> 
> E importante que se compreenda corretamente o que e
> um SISTEMA FORMAL e o 
> que vem a ser COMPLETUDE e CONSISTENCIA num tal
> sistema. Estes sistemas tem, 
> a grosso modo :
> 
> 1) Objetos indefinidos ( ou primitivos )
> 2) proposicoes primitivas ( ou axiomas, ou
> postulados )
> 
> NAO SE PODE ATRIBUIR AOS OBJETOS PRIMITIVOS NENHUMA
> PROPRIEDADE DITADA POR 
> UMA EVENTUAL REPRESENTACAO MENTAL E INTUITIVA QUE
> TENHAMOS DELES. Tudo que 
> se falar sobre os objetos deve ser uma consequencia
> logica dos axiomas e dos 
> teoremas que ja tenhamos demonstrado. Pode-se
> construir novos objetos em 
> estrita obediencia as regras de construcao.
> 
> 1)Um sistema formal e CONSISTENTE se nao for
> possivel provar uma afirmacao e 
> a sua negacao, isto e, exemplificando, se eu provar
> que "A e B" eu nao 
> poderei provar que "A e nao B"
> 
> 2)Um sistema formal e COMPLETO se todas as
> afirmacoes sobre os objetos puder 
> ser provada com os recursos de inferencia do proprio
> sistema, isto e, nao 
> pode haver uma propriedade usufruida por alguns
> objetos do sistema que seja 
> indemonstravel com os recursos de inferencia do
> sistema.
> 
> Em geral, criar uma sistema formal e, em geral, um
> dos objetivos perseguidos 
> para qualquer ramo da matematica, sobretudo quando
> ele ja esta 
> suficientemente maduro e ja deu bons resultados.
> 
> A grosso modo, o que Godel mostrou e que os dois
> conceitos acima, de 
> COMPLETUDE e INCONSISTENCIA, sao antagonicos para
> qualquer sistema formal 
> que use minimos recursos da Aritmetica, isto e :
> 
> "Se o sistema formal for COMPLETO, isto e, toda
> afirmacao sobre os objetos 
> do sistema puderem ser demonstradas com os recursos
> de inferencia do 
> sistema, entao ele sera INCONSISTENTE, vale dizer,
> nos seremos capazes de 
> provar uma teorema e a negacao dele; Se, por outro
> lado, o sistema formal 
> for CONSISTENTE, isto e, se nunca poder acontecer de
> provarmos um teorema e 
> a sua negacao, entao ele sera INCOMPLETO, vale
> dizer, haverao propriedades 
> validas dos objetos do sistema que nos nao seremos
> capazes de provar com os 
> recursos de inferencia do proprio sistema."
> 
> Nao existe Teorema da Completude na Geometria
> Euclidiana. Nao no sentido de 
> COMPLETUDE de um sistema formal. Hilbert mostrou que
> a geometria euclidiana 
> seria consistente, se a algebra tambem fosse. Mas a
> consistencia da Algebra 
> depende da Aritmetica e a prova da consistencia
> desta ultima parece muito 
> dificil de ser conseguida ...
> 
> Ate parece, numa primeira apreciacao, que o Teorema
> de Godel e algo ruim e 
> negativo... Ele sulapou o sonho de Hilbert e de
> todos os Matematicos 
> formalistas, que com seus sistemas formais, tiravam
> o sentido intuitivo que 
> damos aos objetos matematicos, reduzindo a
> Matematica a um jogo logico sem 
> graca, sem semantica e sem sentido.
> 
> Observe que COMPLETUDE e CONSISTENCIA sao
> propriedade DO SISTEMA FORMAL, nao 
> de um de seus objetos : sao portanto propriedades do
> TODO. Visto por este 
> angulo, Godel mostrou que o TODO tem propriedades (
> consistencia, completude 
> )  que sao inacessiveis ou inesplicaveis pela mera
> consideracao das partes 
> que o compoe O TODO, isto e, O TODO E MAIS QUE A
> MERA SOMA DAS PARTES. O 
> cara formalista pressupoe justamente o contrario.
> Ele pensa que conhecendo 
> bem as partes ( axiomas, teoremas, objetos
> indefinidos ) vai poder explicar 
> ( demonstrar ) tudo que aparecer ou ocorrer na
> frente dele. E o SONHO 
> EISNTENIANO de encontrar UM CONJUNTO DE EQUACOES QUE
> EXPLICAM TODO O 
> UNIVERSO.
> 
> Godel, nos permitiu comecar a pensar NO SENTIDO, NA
> SEMANTICA, NO FIM, NA 
> FUNCAO, NO PAPEL, NA INTERPRETACAO TELEOLOGICA, como
> algo mais que mera 
> filosofia barata. Se se retirar o sentido das
> coisas, as coisa perdem o 
> sentido. Agora, como articular de forma consistente
> e seria este sentido ?
> 
> Todos os danos que estamos causando ao mundo
> natural, que vem ha anos 
> preocupando ecologistas do mundo inteiro, promanam
> de nossa ignorancia com 
> respeito ao papel e o sentido dos fenomenos. O ideal
> seria que nos nos 
> relacionassemos com a natureza respeitando os seus
> acontecimentos ou o papel 
> que cada coisa tem. Todavia, quem tem que dar esta
> linguagem, como sempre, e 
> a Matematica, e muito provavelmente foi o Teorema de
> Godel o primeiro passo 
> neste sentido.
> 
> Um 

Re: 2_questões...

2001-12-14 Por tôpico Carlos Maçaranduba

 --- "henrique.vitorio" <[EMAIL PROTECTED]>
escreveu: > 
> Oi, 
> Saudações a todos,meu nome eh Henrique(sow de 
> Recife) e sow novo nessa lista.Entaum..aí vaum umas 
> questões que gostaria que me ajudassem. 
>   1- encontre todas soluções inteiras positivas de: 
>7^(x) + 1 = 5^(z) + 3^(y) (nessa questão soh
> consegui 
> mostrar que x,y e z têm que ser ímpar). 

gostaria de ver como vc provou que todos devem ser
ímpares pois,tirando a solução trivial(1,1,1),temos
três  tipos de padroes para x e y mod4 .Se isso for
verdade ,apenas quando xmod4=1 e ymod=1 é que é uma
caracteristica da solução.Veja só:


as potencias sucessivas de 7 ,terminam em 7 , 9 , 3 e
1 sempre nesta ordem ou seja pegue o x e divida por 4
e pegue o resto.Se resto =1,termina em 7 ,se resto
igual a 2 termina em 9 ,se resto=3 termina em 3 ,se
resto =0 termina em 1.OU SEJA NO 1° MEMBRO O ALGARISMO
DA UNIDADE SÓ PODE SER 8 ,0 ,4 E 2(SOMANDO-SE O
1),PORTANTO NO 2° MEMBRO, O ALGARISMO DA UNIDADE DEVE
TER UMA DESSAS TERMINAÇÕES.COMO 5 ELEVADO A QUALQUER
NATURAL TERMINA EM 5 ,ENTAO PARA QUE NO 2° MEMBRO
TENHA AS TERMINAÇOES DO 1°MEMBRO SOMADO AO 5 ,SO PODEM
SER 5 + '5'-> 0 , 5 + '9'-> 4 , 5 + '7'-> 2 e 5 +
'3'-> 8,ou seja a potencia de 3 deve terminar com um
desses números aspeados.Se fizer as potencias
sucessivas de 3 ,sempre obterá nas unidades 3, 9 ,7 e
1 ou seja: 

ymod4=1-> termina em 3
ymod4=2-> termina em 9
ymod4=3-> termina em 7
ymod4=0-> termina em 1
 
Perceba que a potencia de 3 nunca termina em 5.

entao as combinações só podem ser essas:

Se xmod4=3 -> ymod4=2 para todo z.
x é impar e y é par. 

Se xmod4=0 -> ymod4=3 para todo z.
x é par e y é impar.

Se xmod4=1 -> ymod4=1 para todo z.
x e y são impares.

Se sua prova estiver correta essa última seria a única
solução.Ou seja x e y divididos por 4 sempre tem que
deixar resto 1 .Gostaria que vc enviasse a prova.

è sempre bom ter um conterraneo.

"O plano decisivo entre a certeza e a incerteza é o
próprio eu" (Montaigne)


___
Yahoo! GeoCities
Tenha seu lugar na Web. Construa hoje mesmo sua home page no Yahoo! GeoCities. É fácil 
e grátis!
http://br.geocities.yahoo.com/



Re: Programa_para_achar_nºs_primos_...

2001-12-03 Por tôpico Carlos Maçaranduba

Rapaz existem algoritmos que podem ser implementados
para se achar primos , pelo fato de eles serem cada
vez mais raros quando se vai a direita na reta real ,é
muito  demorado vc achar primos cada vez
maiores.Desconheço um programa no mercado somente com
essa finalidade.Eu acho melhor pedir a alguem que
entenda melhor ,fazer um executável  para vc.



--- Eleu Lima Natalli <[EMAIL PROTECTED]> escreveu:
> Alguem sabe em q site posso baixar o prog q ''caça''
>  nº primos ?
> 
> 
> []s
>  

___
Yahoo! GeoCities
Tenha seu lugar na Web. Construa hoje mesmo sua home page no Yahoo! GeoCities. É fácil 
e grátis!
http://br.geocities.yahoo.com/



Re: Não_deveria_existir_multiplicação_por_0

2001-11-27 Por tôpico Carlos Maçaranduba

viajei na sua argumentação.Não entendi muita coisa.Mas
um n° elevado a zero é apenas convenção da propriedade
das potencias ,se vc tem 2^5 dividido por 2^5 é lógico
que dá 1 .Se vc aplica a propriedade que mantem a base
 e subtrai os expontes vc fica com 2^0 que é 1 por
isso.Faça tb 2 dividido por 0.2 , e depois por 2 por
0.02 e assim sucessivamente e compare os resultados
.Verá que quanto mais o denominador tende a zero maior
fica o resultado e ainda podemos pegar um número tão
pequeno quanto quisermos e só faremos aumentar o
resultado.Dá para achar a divisao por zero??Neste caso
fica indeterminada,mais há casos estudados em cálculo
1 que dá para saber o seu "limite".

Espero ter ajudado.   



 --- "Nicolau C. Saldanha"
<[EMAIL PROTECTED]> escreveu: > On Tue,
Nov 27, 2001 at 01:48:00PM -0200, Wassermam
> wrote:
> > Na minha opinião particular esta totalmente
> erronio multiplicação por 0, eu
> > acho errado acho que não deveria existir
> > Eu posso dar mil explicaçòes pq não mas vou dar
> poucas
> > 0x=1 agora de uma olhada nisto, vc não pode
> dividir os 2 termos por 0 e se vc
> > fazer o 0x=0 dai isto esta errado
> > e eu 5tb não concordo que 0^0=1 pois todo numero
> elevado a 0 =1
> > Deveria ser 0 ou infinito pois
> > 2.2.2= 2^3
> > 2.2=2^2
> > 2=2^1
> > 1=2^0
> > notem que esta noção deum saiu deste conceito ve
> que quando mais diminui o
> > elevado vai se dividindo por 0
> > Mas o 0 é um caso a parte
> > 0=0^x
> > 0.0.0=0^3
> > Dai como que podeira se dividir por 0 isto não tem
> lógica, então nunca deveria
> > multiplicação por 0 pois dai vc não tem o processo
> inverço em uma equação
> > algébrica, e pensando concretamente vc vai pegar
> uma pessoa e vai multiplicar
> > por 0, isso não deveria existir.
> > 
> > Desculpe pela falata de linearidade no pensamento
> mas acho que deu pra entender
> 
> Eu pelo menos não entendi nada. Acho que se você
> espera uma resposta
> você vai precisar argumentar melhor. []s, N. 

___
Yahoo! GeoCities
Tenha seu lugar na Web. Construa hoje mesmo sua home page no Yahoo! GeoCities. É fácil 
e grátis!
http://br.geocities.yahoo.com/



Re: RES: problema

2001-11-20 Por tôpico Carlos Maçaranduba

o que é o método das diferenças e qual a relação dele
com as equações de recorrencia??? 


--- Augusto César Morgado <[EMAIL PROTECTED]>
escreveu: > Uma coisa muito boa para por a cabeça da
gente no
> lugar diante de certos 
> problemas é reduzi-lo.
> Por exemplo, por que não pensar como as fg seriam
> usadas neste problema:
> Quantas sao as soluçoes em naturais de x+y = 8 com x
> entre 2 e 5 e y 
> entre 5 e 7?
> 
> Carlos Maçaranduba wrote:
> 
> >ok,mas poderia explicar -me como eu resolvo por
> >funções  geratrizes???fui procurar mais sobre isso
> e
> >encontrei alguns problemas de contagem sendo
> >resolvidos por estas tecnicas como abaixo:
> >
> >ache o números de soluções de x_1 + x_2 + x_3 = 17
> >tais que 2<= x_1 <=5 , 3 <= x_2 <= 6 e 4 <= x_3 <=
> 7 e
> >x_1 ,x_2 e x_3 são naturais.
> >
> >Simplesmente ele faz:
> >
> >
> >(x^2 + x^3 + x^4 + x^5)(x^3+ x^4 + x^5 + x^6)(x^4 +
> >x^5 + x^6 + x^7) e acha o coeficiente de x^17 que é
> >igual a 3 ,  que é a resposta do problema...
> >
> >   QUE MÁGICA É ESSA
> >
> >E O PIOR DE TUDO QUE AINDA HÁ GENTE QUE PREFERE
> >BIOLOGIA
> >
> >
> >--- "M. A. A. Cohen" <[EMAIL PROTECTED]> escreveu:
> >
> >Como vc nao disse nada sobre qtos tipos de cada
> >
> >>operador vc tem vou supor
> >>que os operadores e os operandos estao fixos, e
> seu
> >>objetivo eh descobrir de
> >>qtos maneiras pode colocar parenteses pra realizar
> >>essa operacao (se vc
> >>quiser mudar as ordens dos operadores ou inserir
> >>diferentes tipo de
> >>operando, o problema nao muda mto. o dificil acho
> >>que serah exatamente esse
> >>problema final).
> >>Esse, se nao me engano, eh um problema famoso. A
> >>resposta eh o q se costuma
> >>chamar de n-o numero de Catalan (vale
> >>[Binomial(2n,n)]/(n+1).
> >>Uma maneira de se provar isso eh considerar a
> funcao
> >>geratriz F cujo
> >>coeficiente de x^n eh a resposta do problema para
> >>cada n. Ai vc nota que
> >>sempre existe exatamente UM operando fora de todos
> >>os parenteses (que serve
> >>para ligar duas contas grandes). entao, C_n = C_0
> *
> >>C_n-1 + C_1 * C_n-2 +
> >>... + C_n-1 * C0  (q notacao horrivel!).
> >>Enfim, vc pediu uma dica neh :)
> >>Vc consegue achar uma equacao do segundo grau em
> >>F(x) e resolvendo e usando
> >>binomio de Newton vc encontra finalmente que o
> >>coeficiente de x_n eh sempre
> >>aquele numero la de cima.
> >>Tem um jeito de resolver esse problema sem usar
> >>funcoes geratrizes. Eu li
> >>uma vez no livro "Matematica Concreta", mas nao
> >>lembro agora como se faz...
> >>Abracos,
> >>Marcio
> >>
> >>-Mensagem original-
> >>De: [EMAIL PROTECTED]
> >>[mailto:[EMAIL PROTECTED]]Em nome
> de
> >>Carlos Maçaranduba
> >>Enviada em: quarta-feira, 14 de novembro de 2001
> >>17:44
> >>Para: [EMAIL PROTECTED]
> >>Assunto: problema
> >>
> >>
> >>Quem não conseguir fazer pelo menos diga uma
> >>idéia.Esta forma é a chamada forma infixa(forma no
> >>qual nós escrevemos) , mas existem as formas
> prefixa
> >>e
> >>posfixa(esta última usada em expressoes algébricas
> >>em
> >>compiladores pois se trata de uma forma mais
> >>eficiente
> >>de interpretar uma expressão algébrica).Depois
> digo
> >>como é a forma posfixa.Mas por favor tentem
> resolver
> >>essa questào para mim.
> >>
> >>
> >>Seja uma sequencia de operandos e operadores
> >>mostrados
> >>como abaixo:
> >>
> >>A+B.C;
> >>
> >>Separando por parenteses poderiamos obter duas
> >>expressões algébricas: (A+B).C ou A+(B.C);
> >>Repare que temos três operandos e dois
> >>operadores(multiplicação e soma).Dada uma
> sequencia
> >>de
> >>n operandos e n-1 operadores ,de quantas formas
> >>diferentes se pode formar expressões algébricas
> >>separadas por parenteses???
> >>
> >>obs:Obviamente que a sequencia começa por um
> >>operando
> >>e termina com outro operando.
> >>
> >>
>
>
> >
> >>___
> >>Yahoo! GeoCities
> >>Tenha seu lugar na Web. Construa hoje mesmo sua
> home
> >>page no Yahoo!
> >>GeoCities. É fácil e grátis!
> >>http://br.geocities.yahoo.com/
> >> 
> >>
> >
>
>___
> >Yahoo! GeoCities
> >Tenha seu lugar na Web. Construa hoje mesmo sua
> home page no Yahoo! GeoCities. É fácil e grátis!
> >http://br.geocities.yahoo.com/
> >
> >
> 
>  

___
Yahoo! GeoCities
Tenha seu lugar na Web. Construa hoje mesmo sua home page no Yahoo! GeoCities. É fácil 
e grátis!
http://br.geocities.yahoo.com/



Re: RES: problema

2001-11-17 Por tôpico Carlos Maçaranduba

poxa mas eu já estou perguntando 


--- Augusto César Morgado <[EMAIL PROTECTED]>
escreveu: > Uma coisa muito boa para por a cabeça da
gente no
> lugar diante de certos 
> problemas é reduzi-lo.
> Por exemplo, por que não pensar como as fg seriam
> usadas neste problema:
> Quantas sao as soluçoes em naturais de x+y = 8 com x
> entre 2 e 5 e y 
> entre 5 e 7?
> 
> Carlos Maçaranduba wrote:
> 
> >ok,mas poderia explicar -me como eu resolvo por
> >funções  geratrizes???fui procurar mais sobre isso
> e
> >encontrei alguns problemas de contagem sendo
> >resolvidos por estas tecnicas como abaixo:
> >
> >ache o números de soluções de x_1 + x_2 + x_3 = 17
> >tais que 2<= x_1 <=5 , 3 <= x_2 <= 6 e 4 <= x_3 <=
> 7 e
> >x_1 ,x_2 e x_3 são naturais.
> >
> >Simplesmente ele faz:
> >
> >
> >(x^2 + x^3 + x^4 + x^5)(x^3+ x^4 + x^5 + x^6)(x^4 +
> >x^5 + x^6 + x^7) e acha o coeficiente de x^17 que é
> >igual a 3 ,  que é a resposta do problema...
> >
> >   QUE MÁGICA É ESSA
> >
> >E O PIOR DE TUDO QUE AINDA HÁ GENTE QUE PREFERE
> >BIOLOGIA
> >
> >
> >--- "M. A. A. Cohen" <[EMAIL PROTECTED]> escreveu:
> >
> >Como vc nao disse nada sobre qtos tipos de cada
> >
> >>operador vc tem vou supor
> >>que os operadores e os operandos estao fixos, e
> seu
> >>objetivo eh descobrir de
> >>qtos maneiras pode colocar parenteses pra realizar
> >>essa operacao (se vc
> >>quiser mudar as ordens dos operadores ou inserir
> >>diferentes tipo de
> >>operando, o problema nao muda mto. o dificil acho
> >>que serah exatamente esse
> >>problema final).
> >>Esse, se nao me engano, eh um problema famoso. A
> >>resposta eh o q se costuma
> >>chamar de n-o numero de Catalan (vale
> >>[Binomial(2n,n)]/(n+1).
> >>Uma maneira de se provar isso eh considerar a
> funcao
> >>geratriz F cujo
> >>coeficiente de x^n eh a resposta do problema para
> >>cada n. Ai vc nota que
> >>sempre existe exatamente UM operando fora de todos
> >>os parenteses (que serve
> >>para ligar duas contas grandes). entao, C_n = C_0
> *
> >>C_n-1 + C_1 * C_n-2 +
> >>... + C_n-1 * C0  (q notacao horrivel!).
> >>Enfim, vc pediu uma dica neh :)
> >>Vc consegue achar uma equacao do segundo grau em
> >>F(x) e resolvendo e usando
> >>binomio de Newton vc encontra finalmente que o
> >>coeficiente de x_n eh sempre
> >>aquele numero la de cima.
> >>Tem um jeito de resolver esse problema sem usar
> >>funcoes geratrizes. Eu li
> >>uma vez no livro "Matematica Concreta", mas nao
> >>lembro agora como se faz...
> >>Abracos,
> >>Marcio
> >>
> >>-Mensagem original-
> >>De: [EMAIL PROTECTED]
> >>[mailto:[EMAIL PROTECTED]]Em nome
> de
> >>Carlos Maçaranduba
> >>Enviada em: quarta-feira, 14 de novembro de 2001
> >>17:44
> >>Para: [EMAIL PROTECTED]
> >>Assunto: problema
> >>
> >>
> >>Quem não conseguir fazer pelo menos diga uma
> >>idéia.Esta forma é a chamada forma infixa(forma no
> >>qual nós escrevemos) , mas existem as formas
> prefixa
> >>e
> >>posfixa(esta última usada em expressoes algébricas
> >>em
> >>compiladores pois se trata de uma forma mais
> >>eficiente
> >>de interpretar uma expressão algébrica).Depois
> digo
> >>como é a forma posfixa.Mas por favor tentem
> resolver
> >>essa questào para mim.
> >>
> >>
> >>Seja uma sequencia de operandos e operadores
> >>mostrados
> >>como abaixo:
> >>
> >>A+B.C;
> >>
> >>Separando por parenteses poderiamos obter duas
> >>expressões algébricas: (A+B).C ou A+(B.C);
> >>Repare que temos três operandos e dois
> >>operadores(multiplicação e soma).Dada uma
> sequencia
> >>de
> >>n operandos e n-1 operadores ,de quantas formas
> >>diferentes se pode formar expressões algébricas
> >>separadas por parenteses???
> >>
> >>obs:Obviamente que a sequencia começa por um
> >>operando
> >>e termina com outro operando.
> >>
> >>
>
>
> >
> >>___
> >>Yahoo! GeoCities
> >>Tenha seu lugar na Web. Construa hoje mesmo sua
> home
> >>page no Yahoo!
> >>GeoCities. É fácil e grátis!
> >>http://br.geocities.yahoo.com/
> >> 
> >>
> >
>
>___
> >Yahoo! GeoCities
> >Tenha seu lugar na Web. Construa hoje mesmo sua
> home page no Yahoo! GeoCities. É fácil e grátis!
> >http://br.geocities.yahoo.com/
> >
> >
> 
>  

___
Yahoo! GeoCities
Tenha seu lugar na Web. Construa hoje mesmo sua home page no Yahoo! GeoCities. É fácil 
e grátis!
http://br.geocities.yahoo.com/



Re: RES: problema

2001-11-17 Por tôpico Carlos Maçaranduba

ok,mas poderia explicar -me como eu resolvo por
funções  geratrizes???fui procurar mais sobre isso e
encontrei alguns problemas de contagem sendo
resolvidos por estas tecnicas como abaixo:

ache o números de soluções de x_1 + x_2 + x_3 = 17
tais que 2<= x_1 <=5 , 3 <= x_2 <= 6 e 4 <= x_3 <= 7 e
x_1 ,x_2 e x_3 são naturais.

Simplesmente ele faz:


(x^2 + x^3 + x^4 + x^5)(x^3+ x^4 + x^5 + x^6)(x^4 +
x^5 + x^6 + x^7) e acha o coeficiente de x^17 que é
igual a 3 ,  que é a resposta do problema...

   QUE MÁGICA É ESSA

E O PIOR DE TUDO QUE AINDA HÁ GENTE QUE PREFERE
BIOLOGIA


--- "M. A. A. Cohen" <[EMAIL PROTECTED]> escreveu: >
Como vc nao disse nada sobre qtos tipos de cada
> operador vc tem vou supor
> que os operadores e os operandos estao fixos, e seu
> objetivo eh descobrir de
> qtos maneiras pode colocar parenteses pra realizar
> essa operacao (se vc
> quiser mudar as ordens dos operadores ou inserir
> diferentes tipo de
> operando, o problema nao muda mto. o dificil acho
> que serah exatamente esse
> problema final).
> Esse, se nao me engano, eh um problema famoso. A
> resposta eh o q se costuma
> chamar de n-o numero de Catalan (vale
> [Binomial(2n,n)]/(n+1).
> Uma maneira de se provar isso eh considerar a funcao
> geratriz F cujo
> coeficiente de x^n eh a resposta do problema para
> cada n. Ai vc nota que
> sempre existe exatamente UM operando fora de todos
> os parenteses (que serve
> para ligar duas contas grandes). entao, C_n = C_0 *
> C_n-1 + C_1 * C_n-2 +
> ... + C_n-1 * C0  (q notacao horrivel!).
> Enfim, vc pediu uma dica neh :)
> Vc consegue achar uma equacao do segundo grau em
> F(x) e resolvendo e usando
> binomio de Newton vc encontra finalmente que o
> coeficiente de x_n eh sempre
> aquele numero la de cima.
> Tem um jeito de resolver esse problema sem usar
> funcoes geratrizes. Eu li
> uma vez no livro "Matematica Concreta", mas nao
> lembro agora como se faz...
> Abracos,
> Marcio
> 
> -Mensagem original-
> De: [EMAIL PROTECTED]
> [mailto:[EMAIL PROTECTED]]Em nome de
> Carlos Maçaranduba
> Enviada em: quarta-feira, 14 de novembro de 2001
> 17:44
> Para: [EMAIL PROTECTED]
> Assunto: problema
> 
> 
> Quem não conseguir fazer pelo menos diga uma
> idéia.Esta forma é a chamada forma infixa(forma no
> qual nós escrevemos) , mas existem as formas prefixa
> e
> posfixa(esta última usada em expressoes algébricas
> em
> compiladores pois se trata de uma forma mais
> eficiente
> de interpretar uma expressão algébrica).Depois digo
> como é a forma posfixa.Mas por favor tentem resolver
> essa questào para mim.
> 
> 
> Seja uma sequencia de operandos e operadores
> mostrados
> como abaixo:
> 
> A+B.C;
> 
> Separando por parenteses poderiamos obter duas
> expressões algébricas: (A+B).C ou A+(B.C);
> Repare que temos três operandos e dois
> operadores(multiplicação e soma).Dada uma sequencia
> de
> n operandos e n-1 operadores ,de quantas formas
> diferentes se pode formar expressões algébricas
> separadas por parenteses???
> 
> obs:Obviamente que a sequencia começa por um
> operando
> e termina com outro operando.
> 
>

> ___
> Yahoo! GeoCities
> Tenha seu lugar na Web. Construa hoje mesmo sua home
> page no Yahoo!
> GeoCities. É fácil e grátis!
> http://br.geocities.yahoo.com/
>  

___
Yahoo! GeoCities
Tenha seu lugar na Web. Construa hoje mesmo sua home page no Yahoo! GeoCities. É fácil 
e grátis!
http://br.geocities.yahoo.com/



problema

2001-11-14 Por tôpico Carlos Maçaranduba

Quem não conseguir fazer pelo menos diga uma
idéia.Esta forma é a chamada forma infixa(forma no
qual nós escrevemos) , mas existem as formas prefixa e
posfixa(esta última usada em expressoes algébricas em
compiladores pois se trata de uma forma mais eficiente
de interpretar uma expressão algébrica).Depois digo
como é a forma posfixa.Mas por favor tentem resolver
essa questào para mim.


Seja uma sequencia de operandos e operadores mostrados
como abaixo:

A+B.C;

Separando por parenteses poderiamos obter duas
expressões algébricas: (A+B).C ou A+(B.C);
Repare que temos três operandos e dois
operadores(multiplicação e soma).Dada uma sequencia de
n operandos e n-1 operadores ,de quantas formas
diferentes se pode formar expressões algébricas
separadas por parenteses???

obs:Obviamente que a sequencia começa por um operando
e termina com outro operando.

___
Yahoo! GeoCities
Tenha seu lugar na Web. Construa hoje mesmo sua home page no Yahoo! GeoCities. É fácil 
e grátis!
http://br.geocities.yahoo.com/



Re: Problema do IME-88/89 de analise combinatoria.

2001-11-02 Por tôpico Carlos Maçaranduba

 espero que eu não esteja errado. 

> Eis a setima questao da prova do IME-88/89.
> 
> Em cada uma das faces de um cubo constroi-se um
> circulo e, em cada circulo, 
> marcam-se n pontos. Unindo-se estes pontos,
> (a) quantas retas, nao contidas numa mesma face do
> cubo, podem ser formadas?
 
 (C_2n,2 - 2.C_n,2).C_6,2 . PEGUE DUAS FACES.EXISTEM
AO TOTAL 2n PONTOS.COMO A CADA DOIS PONTOS FORMA-SE
UMA RETA BASTA FAZER C_2n ,2.MAS EU ESTOU CONTANDO AS
RETAS DE MESMA FACE QUE SÃO C_n,2 E COMO SÃO DUAS
FACES FAÇO 2.C_n,2  E TIRO DO QUE OBTIVE.BASTA APLICAR
A TODOS OS CONJUNTOS DE 2 FACES PORTANTO MULTIPLICO
POR C_6,2.   



> (b) quantos triangulos, nao contidos numa mesma face
> do cubo, podem ser 
> formados?

(C_3n,3 - 3.C_n,3 ).C_6,3

PEGUE 3 FACES . SÃO 3n PONTOS .FAZENDO C_3n,3 OBTENHO
TODOS OS TRIANGULOS ,INCLUSIVE OS QUE ESTAO NA MESMA
FACE PORTANDO DEVO TIRA-LOS FAZENDO C_n,3 E POR SEREM
3 FACES MULTIPLICAR POR 3.BASTA APLICAR ESSA REGRA A
TODOS OS CONJUNTOS DE 3 FACES MULTIPLICANDO POR C_6,3.
PERCEBA QUE FAÇO 3n PQ INCLUO O CASO DE CADA PONTO
ESTA EM UMA FACE DISTINTA.

> (c) quantos tetraedros, com base numa das faces do
> cubo, podem ser formados?

(C_n,3 . C_n,1).2.C_6,2.

PEGUE DUAS FACES E ESCOLHA UMA PARA SER A BASE . A
CADA 1 PONTO DA FACE QUE NÃO É BASE TEMOS C_n,3 PONTOS
 DA BASE E TETRAEDROS .BASTA FAZER PARA TODOS OS
PONTOS DA FACE QUE NÃO É BASE .COMO A FACE QUE NÃO É
FACE PODE SÊ-LA BASTA MULTIPLICAR POR 2.APLICA-SE A
TODOS OS CONJUNTOS DE 2 FACES DO CUBO MULTIPLICANDO
POR C_6,2.  

> (d) quantos tetraedros, com todos os vertices em
> faces diferentes, podem ser 
> formados?

  ((C_n,1)^4 ).C_6,4
 PEGUE 4 FACES E ESCOLHA UM PONTO DE CADA .TODAS AS
VARIAÇÕES DARÁ O N° DE TETRAEDROS COM ESSA CONDIÇAO AS
4 FACES .PORTANTO (C_n,1)^4.BASTA APLICAR A TODOS OS
CONJUNTOS DE 4 FACES POR ISSO C_6,4.




> OBS: Suponha que, se 4 pontos nao pertencem a uma
> mesma face, entao nao sao 
> coplanares.
> 
> Agora vai uma pergunta minha:
> A questao mudaria se a observacao final nao se
> aplicasse para os itens (a) e 
> (b)? Por que?
 VEJA SÓ.ESSA CONDIÇÃO FOI IMPOSTA SOMENTE POR CAUSA
DO TETRAEDRO (DA LETRA C PRINCIPALMENTE) POIS PEGANDO
A LETRA C , E NÃO APLICANDO TAL CONDIÇÃO ,NADA ME
GARANTIRIA QUE EU NÃO ESTARIA CONTANDO TETRAEDROS A
MAIS.PELO FATO DE UMA RETA E UM TRIANGULO SEREM
FIGURAS PLANARES E SEUS PONTOS ESTAREM DISPOSTOS SOBRE
CIRCULOS ,CASO 4 PONTOS ESTIVESSEM EM UM MESMO
PLANO(NÃO NECESSARIAMENTE NA MESMA FACE),COM ELES 
OBTERIA O MESMO N° DE RETAS E TRIANGULOS OBTIDOS PELO
CASO CONTRÁRIO.   

 

___
Yahoo! GeoCities
Tenha seu lugar na Web. Construa hoje mesmo sua home page no Yahoo! GeoCities. É fácil 
e grátis!
http://br.geocities.yahoo.com/



Re: Polinômios/ITA

2001-10-27 Por tôpico Carlos Maçaranduba

 --- [EMAIL PROTECTED] escreveu: >Olá,
>Gostaria de ajuda para questões de polinômios
> que estiveram em provas 
> do ITA:
> 1-) Seja a equação p(x) = 0 , onde p(x) é um
> polinômio de grau m. Se p(x) 
> admite uma raiz inteira, então p(-1).p(0).p(1) é
> divisível por 3. Prove.

p(x) pode ser escrito como:

p(x)= a_m (x-r_1).(x-r_2).(x-r_3)(x-r_m) , onde
r_i
 1<=i<=m representa uma das raizes de p(x). 

 suponha que a raiz inteira seja r_t (1<=t<=m).Fazendo
p(-1)= (-1 - r_1).(-1 - r_t)(-1 - r_m)
p(0) = (0 - r_1 ).(0 - r_t).(0 - r_m)
p(1) = (1 - r_1).(1 - r_t ).(1 - r_m)

Perceba que se eu fizer p(-1).p(0).p(1) eu terei o
produto imbutido (-1 - r_t).(0 - r_t).(1 - r_t ) que
são 3 inteiros consecutivos pois r_t é inteiro.E como
o produto de 3 inteiros consecutivos é divisível por 3
a questão fica demonstrada.

___
Yahoo! GeoCities
Tenha seu lugar na Web. Construa hoje mesmo sua home page no Yahoo! GeoCities. É fácil 
e grátis!
http://br.geocities.yahoo.com/



Re: 2 problemas..

2001-10-23 Por tôpico Carlos Maçaranduba

Pq seriam periódicos??Explique melhor.





 --- Carlos Gustavo Tamm de Araujo Moreira
<[EMAIL PROTECTED]> escreveu: >Oi,Stein,
>O primeiro o pessoal ja' discutiu,ne'?
>Vamos ao segundo:Nao e' dificil ver que g(x)=x
> deve ter duas raizes reais
> distintas.Como g(g(x))=f(f(f(f(x=x tem 4
> solucoes,duas das quais sao as
> solucoes de g(x)=x,sobram duas solucoes,que seriam
> dois pontos periodicos de
> periodo exatamente 4 de f,o que e' absurdo,pois se x
> e' ponto periodico de
> periodo exatamente 4 de f entao f tem pelo menos 4
> pontos periodicos de
> periodo exatamente 4 (x,f(x),f(f(x)) e f(f(f(x))),os
> quais sao todos
> distintos,o que nao e' dificil mostrar).
>Abracos,
>Gugu
> P.S.:Como foi a prova ?
> 
> >
> >Gostaria de ver soluções para esses probleminhas 
> que estão me entalando.
> >Valeu.
> >1-Sejam a, b e c reais tais que a^2 + b^2 +1 = c^2.
> Prove que |a/2| + |c/2|
> >é par. |x| é a parte inteira de x.
> >2-Seja g(x)=ax^2 + bx + c uma função quadrática com
> coeficientes reais(a não
> >nulo) tal que a equação g(g(x)) = x tem quatro
> raízes reais distintas.
> >Demontre que não existe nenhuma função f:R->R tal
> que f(f(x)) = g(x) para
> >todo x real. 
>  

___
Yahoo! GeoCities
Tenha seu lugar na Web. Construa hoje mesmo sua home page no Yahoo! GeoCities. É fácil 
e grátis!
http://br.geocities.yahoo.com/



pequeno problema

2001-10-19 Por tôpico Carlos Maçaranduba

 DESENVOLVENDO (x^2 + x - 1)^n OBTEM-SE  O POLINOMIO:

 p(x) = a_2n . x^2n + a_2n-1 . x^2n-1 +a_0 

QUANTO VALE A SOMA DOS COEFICIENTES DE ÍNDICE PAR a_2n
+ a_2n-2 + a_2n-4 + a_2n-6 +..a_2 + a_0 para 
n=1992 ??

___
Yahoo! GeoCities
Tenha seu lugar na Web. Construa hoje mesmo sua home page no Yahoo! GeoCities. É fácil 
e grátis!
http://br.geocities.yahoo.com/



Re: 2 problemas..

2001-10-19 Por tôpico Carlos Maçaranduba

tem certeza que o enunciado da 2° questão está
correto?? 




> - Original Message -
> From: "Carlos Stein Naves de Brito"
> <[EMAIL PROTECTED]>
> To: <[EMAIL PROTECTED]>
> Sent: Wednesday, October 17, 2001 7:39 PM
> Subject: 2 problemas..
> 
> 
> Gostaria de ver soluções para esses probleminhas 
> que estão me entalando.
> Valeu.
> 1-Sejam a, b e c reais tais que a^2 + b^2 +1 = c^2.
> Prove que |a/2| + |c/2|
> é par. |x| é a parte inteira de x.
> 2-Seja g(x)=ax^2 + bx + c uma função quadrática com
> coeficientes reais(a não
> nulo) tal que a equação g(g(x)) = x tem quatro
> raízes reais distintas.
> Demontre que não existe nenhuma função f:R->R tal
> que f(f(x)) = g(x) para
> todo x real.
> 
>  

___
Yahoo! GeoCities
Tenha seu lugar na Web. Construa hoje mesmo sua home page no Yahoo! GeoCities. É fácil 
e grátis!
http://br.geocities.yahoo.com/



Re: Problemas rídiculos.....

2001-10-14 Por tôpico Carlos Maçaranduba

 --- Carlos Maçaranduba <[EMAIL PROTECTED]>
escreveu: > Aí vão  uma avalanche de problemas
bastante fáceis  
> 
> para quem ousar tentar resolve-los.Luiz  Ferraz Neto
> ,
>  com os seus ectoplasmas hiperdimensionais vai achar
> essas questões  uma piada.Boa sorte e só
> lamento. 
>  
> 
> 
> >=  , <=  : quer dizer maior ou igual  e  menor ou
> igual.
> 
> 
> Considere um polígono convexo de n lados e suponha
> que
> não há duas de suas diagonais que sejam paralelas
> nem
> três que concorram em um mesmo ponto que não seja
> vértice.Quantos desses pontos de intersecção são
> interiores ao polígono??quantos são exteriores??
   
  Os interiores são Combinaçao n tomados 4 a 4...pq
seria?? 

> Escrevem-se números de cinco dígitos(inclusive os
> começados por zero) em cartões .Como 0,1 e 8 não se
> alteram de cabeça para baixo e como 6 de cabeça para
> baixo se transforma em 9 , um só cartão pode
> representar dois números(por exemplo ,06198 e
> 86190).Qual é o número mínimo de cartões para
> representar todos os números de cinco dígitos ?
> 
> 
> 
> Se a e b são duas das raízes da equação x^4 + x^3 –
> 1
> = 0 , prove que ab é uma raiz de x^6 + x^4 + x^3 –
> x^2
> – 1 = 0.
> 
> 
> 
> Sejam p e q reais positivos tais que 1/p +1/q = 1.
> Prove que  xy <= (x^p) / p  +  (y^q) / q  , para
> todo
> x , y >=0.  
> 
>   
> 
> Seja H a altura de um tetraedro regular e h1 , h2
> ,h3 
> e h4 ,as distancias entre desde um ponto P em seu
> interior ás faces do tetraedro . Prove que 
> 
> 3 > S >= 12/5
> 
> Sendo S:
> 
> S = (H – h1) / (H + h1)  +  (H - h2) / ( H + h2 )  +
> (H- h3) / (H+h3) + (H- h4) / (H+ h4) .
> 
> 
> 
> 
>
___
> Yahoo! GeoCities
> Tenha seu lugar na Web. Construa hoje mesmo sua home
> page no Yahoo! GeoCities. É fácil e grátis!
> http://br.geocities.yahoo.com/ 

___
Yahoo! GeoCities
Tenha seu lugar na Web. Construa hoje mesmo sua home page no Yahoo! GeoCities. É fácil 
e grátis!
http://br.geocities.yahoo.com/



[no subject]

2001-10-14 Por tôpico Carlos Maçaranduba

ou seria (Problema do anagrama) 4! vezes 3??

___
Yahoo! GeoCities
Tenha seu lugar na Web. Construa hoje mesmo sua home page no Yahoo! GeoCities. É fácil 
e grátis!
http://br.geocities.yahoo.com/



Re: probleminhas

2001-10-14 Por tôpico Carlos Maçaranduba

corrijam-me caso eu esteja errado. 


--- pichurin pichurin <[EMAIL PROTECTED]>
escreveu: > 1)quantos anagramas da palavra CASACO
apresentam as
> três vogais juntas?

Observe que tem-se um A repetido.A quantidade de
anagramas da palavra é Permutação repetida de 6
elementos com dois repetidos ou seja P[6]_2=360. 
Observe tb que as três vogais juntas têm-se dois
"As".Aplicando a mesma fórmula para as vogais juntas
"AAO" temos P[3]_2=3.Basta dividir 360 por 3 = 120
anagramas.


> 2)determine o número de soluções da equação:
  x + y + z = 15
 Pode ser feito assim:
   
 Coloca-se 15 pontos e e duas barras:

 .||..

 os pontos representando o 15 e as barras
representando os sinais da soma.Observe que os espaços
deixados pelas barras representam soluções para o
problema.Observe que existem 15 repetidos e 2
repetidos.Entao basta fazer P[17]_15,2 = 136. 



3)se você tiver cinco moedas de valores diferentes,
> quantas gorjetas diferentes voc6e pode dar usando
> duas 
> delas?
 
  Esse por enquanto eu não sei pq quem me garante que
eu pegando dois elementos distintos dois a dois eu não
obtenho somas iguais??

> 4)calcular o número de maneiras de marcar 6 células
> de
> uma tabela 6 X 3, marcando exatamente 2 quadrardos
> em
> cada coluna e 1 quadrado em cada linha.
 
  Construa a tabela.Na primeira coluna marque dois
quadrados.Observe que a segunda coluna fica restrita
com quatro quadrados apenas ,e se marcar a segunda
coluna com uma possibilidade , a terceira só terá uma
solução. Refaça essa brincadeira de corno.Concentre-se
na primeira coluna.Quantas possibilidades eu
teria??Observe que depois que vc marca os dois
quadrados basta vc fazer a permutação dos quadrados
marcados e dos vazios .Neste caso tem-se 4 vazios e
dois marcados , basta fazer a permutação repetida de 6
com dois e quatro repetidos.No mesmo instante em que
faz isso , a segunda coluna fica apenas com 4
possibilidades 2 em branco e 2 marcados e a última
fica determinada.Ou seja seria P[6]_2,4 vezes 
P[4]_2,2 vezes P[2]_2 = 90.  
 

___
> Yahoo! GeoCities
> Tenha seu lugar na Web. Construa hoje mesmo sua home
> page no Yahoo! GeoCities. É fácil e grátis!
> http://br.geocities.yahoo.com/ 

___
Yahoo! GeoCities
Tenha seu lugar na Web. Construa hoje mesmo sua home page no Yahoo! GeoCities. É fácil 
e grátis!
http://br.geocities.yahoo.com/



  1   2   >